Transcript
Page 1: Hepatobiliary & pancreatic surgery

Hepatobiliary & pancreatic surgery1. Which of the following statements about the segmental anatomy of the liver are not true?A. Segments are subdivisions in both the French and American systems.B. Segments are determined primarily by the hepatic venous drainage.C. The French anatomic system is more applicable than the American system to clinical hepatic resection.D. Segments are important to the understanding of the topographic anatomy of the liver.Answer: D

DISCUSSION: Segments are the major subdivision of the right and left lobes of the liver. In either the classic lobar(American) or the segmental (French) system, the most variable aspect is the biliary system. Therefore the hepaticvenous or portal system defines most segments. The French system depicts eight segments, with the caudate lobe assegment I and the other seven segments defined primarily by the hepatic venous system. Segments are not well-depictedby topography.

2. Which of the following anatomic features of the biliary system are important considerations in operativecholangiography?A. The left hepatic duct comes off farther anterior than the right one.B. At the confluence there may be more than just a right and a left hepatic duct.C. Dissection of the triangle of Calot is more important than cholangiography in preventing bile duct injury.D. Segments V, VII, or VIII sometimes join the biliary system below the confluence.Answer: ABCD

DISCUSSION: All of these features are important. The angle of takeoff of the left hepatic duct may make operativevisualization difficult with the patient in the supine position. Because there may be more than two major hepatic ducts,visualization of two large ducts does not ensure that the system is normal. Ducts from any of the right-side segmentscan join below the confluence. Dissecting one superior edge of the gallbladder before it joins the cystic duct isparticularly important in preventing injury.

3. The hepatic artery:A. Supplies the same amount of blood to the liver as the portal vein.B. Provides more blood to the bile ducts than the portal vein.C. Is autoregulated just as the portal vein is.D. Supplies most of the blood to hepatic metastases.Answer: BD

DISCUSSION: The portal vein provides two thirds to three quarters of the total hepatic blood flow. The portal vein isincapable of direct autoregulation. The hepatic artery after transplantation classically infarcts portions of the biliarysystem, whereas hepatic metastases often arrive there via the portal vein. Most of their blood supply comes from thehepatic artery.

4. Bile formation is:A. An active secretory process.B. Determined at two sites principally.C. Regulated physiologically by hormones.D. Largely determined by the intactness of the enterohepatic circulation (EHC).Answer: ABCD

DISCUSSION: Bile formation is an active process at both the canalicular and ductular sites. The paracellular pathwayprobably plays a minor physiologic role. Secretin and glucagon are likely physiologic regulators of biliary secretion.Bile salts are extremely important and are probably the most important agent in the changes that occur whenenterohepatic circulation is interrupted.

5. Generally, the two most important hepatic functions to consider after hepatic resection are:A. Hepatic synthetic function.B. Glucose metabolism.C. The liver's role in lipid metabolism.D. The liver's role in vitamin metabolism.

Page 2: Hepatobiliary & pancreatic surgery

Answer: AB

Asir Surgery MCQs Bank. © 1422H-2002- first impression ©

This project was raised after an idia by Dr. Gharama Al-Shehri (consultant surgeon). Developed and typed by Dr. Ghazi Al-Shumrani (intern).

Page 3: Hepatobiliary & pancreatic surgery

Hepatobiliary & pancreatic surgeryDISCUSSION: While other functions undoubtedly may be important postoperatively, the most common abnormalitiesoccurring after a major hepatic resection are related to loss of protein synthesis and consequences of glucosemetabolism. Therefore, it is usually advisable to administer supplemental amounts of protein and sugar postoperatively.

6. Which of the following statements about pyogenic abscess of the liver are true?A. The right lobe is more commonly involved than the left lobe.B. Appendicitis with perforation and abscess is the most common underlying cause of hepatic abscess.C. Mortality is largely determined by the underlying disease.D. Mortality from hepatic abscess is currently greater than 40%.Answer: AC

DISCUSSION: Involvement of the right lobe with abscess formation approximates 70% of pyogenic abscesses. This isthought to be due to the streaming effect of superior mesenteric venous inflow to the right lobe. In addition, the greatervolume of the right lobe predisposes more tissue to seeding by bacterial organisms. While appendicitis comprised 25%to 40% of cases in early series, early recognition and operative therapy for appendicitis have reduced its importancesignificantly. In current series, malignant or benign biliary obstruction is the underlying cause of 35% to 50% of cases.Recent studies have shown that the underlying disease or an immunocompromised host is more importantprognostically than solitary versus multiple abscesses.

7. Which of the following statements most accurately describes the current therapy for pyogenic hepatic abscess?A. Antibiotics alone are adequate for the treatment of most cases.B. All patients require open surgical drainage for optimal management.C. Optimal treatment involves treatment of not only the abscess but the underlying source as well.D. Percutaneous drainage is more successful for multiple lesions than for solitary ones.Answer: C

DISCUSSION: The development of ultrasonography and computed tomography (CT) in the past two decades hasenabled earlier diagnosis and advances in treatment of hepatic abscess. Formerly, open surgical drainage wasconsidered necessary in essentially all cases of pyogenic abscess. Numerous recent series, however, have reported highsuccess rates and low mortality from the percutaneous catheter drainage of abscesses under CT or ultrasonographicguidance. Optimal management of pyogenic abscess, however, involves not only treatment of the abscess, whether bypercutaneous or surgical methods, but correction of the underlying source as well. All modes of therapy are moresuccessful in treating solitary lesions than multiple ones.

8. Which of the following statements characterize amebic abscess?A. Mortality is higher than that for similarly located pyogenic abscesses.B. The diagnosis of amebic abscess may be based on serologic tests and resolution of symptoms.C. In contrast to pyogenic abscess, the treatment of amebic abscess is primarily medical.D. Patients with amebic abscess tend to be older than those with pyogenic abscess.BC

DISCUSSION: Mortality for uncomplicated amebic abscess should be less than 5%, in contrast to the 15% to 20% ratefor pyogenic abscess. After the demonstration by radiologic examination of an abscess, appropriate serologic tests andresolution of symptoms after a course of treatment with an antiamebic agent such as metronidazole constitutepresumptive diagnosis of amebic abscess. Aspiration of abscess contents rarely yields amebic organisms. In contrast topyogenic abscess, amebic abscess rarely requires surgical or percutaneous drainage, except in the case of an extremelylarge abscess or bacterial superinfection. Amebic abscess affects males in a 9:1 to 10:1 ratio and generally affects ayounger population than pyogenic abscess. Additionally, in the United States the populations most affected areimmigrants from endemic areas such as Mexico or Latin America and American tourists to those regions.

9. Which of the following statement(s) is/are true about benign lesions of the liver?A. Adenomas are true neoplasms with a predisposition for complications and should usually be resected.B. Focal nodular hyperplasia (FNH) is a neoplasm related to birth control pills (BCPs) and usually requires resection.C. Hemangiomas are the most common benign lesions of the liver that come to the surgeon's attention.D. Nodular regenerative hyperplasia does not usually accompany cirrhosis.Answer: A

Page 4: Hepatobiliary & pancreatic surgery

DISCUSSION: Adenomas typically enlarge and cause symptoms, may rupture, and have a definite malignant potential.Therefore they should generally be resected when found. FNH is not a true neoplasm and generally has an uneventfulcourse. Both are related to BCPs, although the relationship of adenoma is more firmly established. While small bile ductAsir Surgery MCQs Bank. © 1422H-2002- first impression ©

This project was raised after an idia by Dr. Gharama Al-Shehri (consultant surgeon). Developed and typed by Dr. Ghazi Al-Shumrani (intern).

Page 5: Hepatobiliary & pancreatic surgery

Hepatobiliary & pancreatic surgeryhamartomas are much more common, hemangiomas are the most common lesion to come to the attention of surgeons.They should not generally be biopsied because of possible hemorrhage. By definition, nodular regenerative hyperplasiaoccurs in the absence of cirrhosis.

10. Which of the following statement(s) about malignant neoplasms of the liver is/are true?A. Hepatocellular carcinoma is probably the number 1 cause of death from cancers worldwide.B. The most common resectable hepatic malignant neoplasm in the United States is colorectal metastasis.C. Hepatoma has at least one variant that has a much more benign course than hepatomas in general.D. Hepatomas are generally slower growing than was formerly believed.Answer: ABCD

DISCUSSION: Although exact comparisons are impossible, hepatoma seems to be the most common cause of cancerdeath worldwide, despite its relative infrequency in the United States. Colorectal metastasis is a more commonindication for surgical treatment in the United States. The fibrolamellar variant and possibly the very well-differentiatedtumor probably have a better prognosis than hepatomas in general. Previous studies from Africa in which there was ahigh incidence of rupture account for the poor prognosis that was generally attributed to hepatoma. Recent studies fromEurope and the United States have shown that survival after presentation is usually measured in years.

11. Which of the following statement(s) is/are true about bile duct cancers?A. If resected, proximal lesions are usually curable.B. The more proximal the lesion, the more likely is resection to be curative.C. Radiation clearly prolongs survival.D. Transplantation is usually successful if the lesion seems confined to the liver.E. None of the above is true.Answer: E

DISCUSSION: Most bile duct cancers are discovered after they are incurable, and only a tiny subset of resectedproximal lesions are cured. The more distal the lesion, the more likely is resection to achieve cure (e.g., approximately30% 5-year survival for periampullary lesions as compared with 0% to 10% for hilar lesions). The use of adjuvant orprimary radiation remains controversial because of the heterogeneity of the patient populations on which this modalityhas been used. Because of the localized nature of this disease it would seem that transplantation would producefavorable results; however, this has not been the case.

12. Echinococcosis liver disease caused by Echinococcus granulosus:A. Is not a neoplasm.B. Is endemic to parts of Europe, but not the United States.C. Is usually curable by resection.D. Is more deadly than in its Echinococcus multilocularis form.Answer: ABC

DISCUSSION: The parasitic infection is fairly common in certain parts of Europe but very rare in the United States.Resection without peritoneal soilage is the treatment of choice. The E. multilocularis form, which is endemic to parts ofthe United States, is more likely to be fatal because it is rarely resectable. This form is more likely to resemble amalignancy than E. granulosus, although the natural course of the disease usually spans many years.

13. Which of the following statements about hemobilia are true?A. Tumors are the most common cause.B. The primary treatment of severe hemobilia is an operation.C. Percutaneous cholangiographic hemobilia is usually minor.D. Ultrasonography usually reveals a specific diagnosis.Answer: C

DISCUSSION: By far the most common cause of hemobilia is trauma. Tumors also may cause the syndrome but arerelatively uncommon causes. For severe hemobilia the best therapy is arteriographic embolization. Usually the site ofbleeding or a false aneurysm can be identified. Operation should be reserved as a last resort or when the condition isrecognized intraoperatively. Percutaneous cholangiography–associated intrabiliary hemorrhage is usually, but notalways, minor and self-limiting. Ultrasonography is a very nonspecific diagnostic technique for hemobilia.

Page 6: Hepatobiliary & pancreatic surgery

Arteriography remains the best diagnostic method.

Asir Surgery MCQs Bank. © 1422H-2002- first impression ©

This project was raised after an idia by Dr. Gharama Al-Shehri (consultant surgeon). Developed and typed by Dr. Ghazi Al-Shumrani (intern).

Page 7: Hepatobiliary & pancreatic surgery

Hepatobiliary & pancreatic surgery14. Ligation of all of the following arteries usually causes significant hepatic enzyme abnormalities except:A. Ligation of the right hepatic artery.B. Ligation of the left hepatic artery.C. Ligation of the hepatic artery distal to the gastroduodenal branch.D. Ligation of the hepatic artery proximal to the gastroduodenal artery.Answer: D

DISCUSSION: Ligation of the right or left hepatic artery frequently causes enzyme elevation but is usually tolerated bythe patient, particularly when this is a life-saving maneuver. Ligation of the hepatic artery distal to the gastroduodenalbranch is more risky but is also usually tolerated. Ligation of the hepatic artery proximal to the gastroduodenal one doesnot normally cause enzyme abnormalities because of abundant collateral flow through that branch.

15. Which of the following is the most common acid-base disturbance in patients with cirrhosis and portalhypertension?A. Metabolic acidosis.B. Respiratory alkalosis.C. Metabolic alkalosis.D. Respiratory acidosis.Answer: C

DISCUSSION: Metabolic alkalosis and hypokalemia are common in patients with cirrhosis because they often haveassociated secondary hyperaldosteronism (especially those with ascites), diarrhea, and frequent emesis.Hyperaldosteronism enhances H+ and K+ exchange for Na+ in the distal tubule of the kidney. The cause of diarrhea inpatients with cirrhosis is unknown, but malabsorption secondary to splanchnic venous hypertension may be acontributing factor. Emesis is common in alcoholic cirrhotics and patients with tense ascites. Deleterious effects ofmetabolic alkalosis include impaired tissue oxygen delivery secondary to shift of the oxyhemoglobin dissociation curveto the left and conversion of ammonium chloride to ammonia, which may contribute to encephalopathy.

16. A portal venous pressure of 30 mm. Hg (elevated) and a hepatic venous wedge pressure of 5 mm. Hg (normal) maybe associated with which of the following causes of portal hypertension?A. Portal vein thrombosis.B. Alcoholic cirrhosis.C. Schistosomiasis.D. Alcoholic hepatitis.Answer: AC

DISCUSSION: Pressure measured by wedging a catheter into a hepatic vein (hepatic venous wedge pressure) closelycorrelates with directly measured portal venous pressure in patients with portal hypertension when the site of elevatedresistance is at the sinusoidal or postsinusoidal level. Such is the case in alcoholic cirrhosis and alcoholic hepatitis.When the site of increased resistance is at the presinusoidal level, either within (schistosomiasis) or outside (portal veinthrombosis) the liver, the hepatic venous wedge pressure is normal despite markedly elevated portal vein pressure.Although schistosomiasis is one of the more frequent causes of portal hypertension worldwide, in North Americapresinusoidal portal hypertension is considerably less common than alcoholic liver disease. A normal hepatic venouswedge pressure in a patient who has bled from varices should lead one to suspect a presinusoidal cause. A specificdiagnosis can often be made by visceral angiography or liver biopsy.

17. Which of the following is the most effective definitive therapy for both prevention of recurrent variceal hemorrhageand control of ascites?A. Endoscopic sclerotherapy.B. Distal splenorenal shunt.C. Esophagogastric devascularization (Sugiura procedure).D. Side-to-side portacaval shunt.E. End-to-side portacaval shunt.Answer: D

DISCUSSION: Shunt operations are the most effective means of preventing recurrent variceal hemorrhage. Rebleedingrates after endoscopic sclerotherapy range from 40% to 60%. Although extensive esophagogastric devascularization haseffectively prevented recurrent bleeding in Japanese series, these operations have been followed by rebleeding rates in

Page 8: Hepatobiliary & pancreatic surgery

excess of 25% in most Western series. Although one controlled trial has shown more frequent recurrent hemorrhagefollowing the distal splenorenal shunt than after the portacaval shunt, most series have reported rebleeding rates of lessthan 10% for both of these operations. Both the liver and the splanchnic viscera are important sites of ascites formation.Asir Surgery MCQs Bank. © 1422H-2002- first impression ©

This project was raised after an idia by Dr. Gharama Al-Shehri (consultant surgeon). Developed and typed by Dr. Ghazi Al-Shumrani (intern).

Page 9: Hepatobiliary & pancreatic surgery

Hepatobiliary & pancreatic surgerySince the distal splenorenal shunt maintains sinusoidal and mesenteric venous hypertension and requires interruption ofimportant retroperitoneal lymphatics, it tends to aggravate rather than relieve ascites. Hepatic sinusoidal pressure maybe unchanged or even increased after an end-to-side portacaval shunt. Only side-to-side portal-systemic shunts, such asthe side-to-side portacaval shunt, reliably decompress both the liver and splanchnic viscera, thus preventing ascitesformation.

18. Which of the following treatments most effectively preserves hepatic portal perfusion?A. Distal splenorenal shunt.B. Conventional splenorenal shunt.C. Endoscopic sclerotherapy.D. Side-to-side portacaval shunt.Answer: C

DISCUSSION: The conventional splenorenal shunt and side-to-side portacaval shunts completely divert portal flowaway from the liver (nonselective shunts). The distal splenorenal shunt is a selective shunt that preserves hepatic portalperfusion in the majority of patients; however, the magnitude of portal flow is decreased because the gastrospleniccomponent is diverted into the renal vein. Additionally, many patients (especially alcoholic cirrhotics) developcollaterals between the mesenteric venous circulation and the shunt, resulting in gradual attrition of the remaining portalflow. Although there have been anecdotal reports of portal vein thrombosis after endoscopic sclerotherapy, twocontrolled trials have demonstrated better preservation of hepatic portal perfusion in sclerotherapy patients than inpersons who receive the distal splenorenal shunt.

19. Which of the following veins is preserved in performing the extensive esophagogastric devascularization proceduredescribed by Sugiura?A. Left gastric (coronary) vein.B. Short gastric vein.C. Splenic vein.D. Left gastroepiploic vein.Answer: A

DISCUSSION: The Sugiura procedure consists of devascularization of the esophagus to the inferior pulmonary veinand the proximal two thirds of the stomach, splenectomy, and distal esophageal transection. The devascularizationcomponent should be done as close to the esophagus and stomach as possible. The coronary vein and paraesophagealcollaterals are preserved to maintain an effective portal-systemic collateral pathway and thereby discourage reformationof varices.

20. Which of the following complications of portal hypertension often require surgical intervention (for more than 25%of patients)?A. Hypersplenism.B. Variceal hemorrhage.C. Ascites.D. Encephalopathy.Answer: B

DISCUSSION: While many patients with portal hypertension develop hypersplenism, it is rarely clinically significant.A splenectomy should not be performed unless platelet counts are persistently less than 20,000 per cu. mm. or whiteblood cell counts are less than 1200 per cu. mm. Unfortunately, splenectomy is sometimes done for clinicallyinsignificant hypersplenism, thus obviating a distal splenorenal shunt if the patient should subsequently bleed fromvarices. The initial treatment for most patients with bleeding esophageal varices should be endoscopic sclerotherapy;however, operation is required for the approximately one third of patients who fail sclerotherapy and for noncompliantpersons, those living in remote geographic locations, and patients bleeding from gastric varices. Ascites can becontrolled by a medical regimen of dietary salt restriction and diuretic therapy in more than 95% of patients. Whenascites is intractable to medical management, either intermittent large-volume paracenteses or a surgicalperitoneovenous shunt should be done. With rare exceptions, encephalopathy should be treated medically. Mostimportant is elimination of any precipitating factors that led to the neuropsychological disturbance. Lactulose,neomycin, and dietary protein restriction may also be components of the medical treatment regimen.

Page 10: Hepatobiliary & pancreatic surgery

21. Which of the following effects are advantages of combined vasopressin and nitroglycerin intravenous infusion, ascompared with vasopressin infusion alone, in controlling acute variceal bleeding?A. Lower frequency of encephalopathy.Asir Surgery MCQs Bank. © 1422H-2002- first impression ©

This project was raised after an idia by Dr. Gharama Al-Shehri (consultant surgeon). Developed and typed by Dr. Ghazi Al-Shumrani (intern).

Page 11: Hepatobiliary & pancreatic surgery

B. Lower incidence of vasopressin side effects.C. More effective control of bleeding.D. Less “rebound effect” when discontinuing the infusion.Answer: BC

Hepatobiliary & pancreatic surgery

DISCUSSION: Vasopressin acts through vasoconstriction of splanchnic arterioles. Both portal venous inflow and portalvenous pressure are reduced, resulting in control of acute variceal bleeding in approximately 50% of patients. However,the adverse side effects of systemic hypertension, bradycardia, decreased cardiac output, and coronary vasoconstrictionare quite common during vasopressin infusion. Simultaneous administration of nitroglycerin or nitroprusside eliminatesthese side effects—and in one controlled trial enhanced therapeutic effectiveness. Although the mechanism of action ofthis combined infusion is not clear, vasodilation of portal-systemic collaterals, resulting in a further reduction in portalpressure, may be responsible.

22. Which of the following statements about the peritoneovenous shunt (PVS) is/are correct?A. For cirrhotic patients with intractable ascites, the LeVeen shunt is an effective “bridge” to liver transplantation.B. Replacement of ascites with saline or lactated Ringer's solution reduces the coagulopathy following PVS.C. For patients with cirrhotic ascites, the survival using repeated paracentesis with 5% albumin infusion is equivalent tothat with the PVS.D. Oliguria (less than 25 ml. per hour) in the immediate postoperative period following PVS should be treated with a5% albumin infusion.E. The transjugular intrahepatic portacaval shunt with stent (TIPSS) works on the same principle as the PVS.Answer: BC

DISCUSSION: The PVS is a palliative procedure that does not prolong life. In comparing the early risks of theprocedure with those of repeated paracentesis, the shunt cannot be justified as a temporizing procedure to facilitateascites control in the patient awaiting liver transplantation. Oliguria is common in the first 24 hours after shuntinsertion. A correctly placed PVS (patency confirmed using an intraoperative “shuntogram”) expands the intravascularvolume with a continuous reinfusion of ascites. Inspection should identify elevation of the jugular venous pressure, anda diuretic (usually furosemide) is needed. The mechanisms of action of the two shunts are very different. TIPSS reducesportal pressure and controls ascites by reducing the rate of ascites formation. PVS reinfuses the ascites fluid, therebyreducing the prerenal stimulus to sodium retention and making the patient more responsive to diuretic therapy.

23. Which of the following clinical situations are considered good indications for PVS?A. A 50-year-old cirrhotic man had an emergency portacaval shunt for bleeding varices and postoperatively had anascites leak and mild superficial wound infection.B. A 57-year-old woman with primary biliary cirrhosis (PBC) has difficult to control ascites and diuretic-inducedencephalopathy.C. A 46-year-old resistant alcoholic has chronic ascites uncontrolled by diuretics combined with repeat paracentesis.D. A 34-year-old woman taking BCPs had rapid onset of ascites and is found to have hepatic vein thrombosis causingthe Budd-Chiari syndrome.Answer: C

DISCUSSION: Because of the high complication rate and the long-term failure rate, the PVS is used only when other,more lasting options for therapy either are not available or are contraindicated. The chronic alcoholic patient maybenefit from a peritoneovenous shunt because his ascites is the dominant problem related to his chronic liver disease,and persistent alcoholism is a contraindication to liver replacement in most centers. PVS may be quite effective for thetemporary management of acute intractable postoperative ascites, such as in patient A; however, it is absolutelycontraindicated in the presence of infection. Patient B has ascites as her dominant problem as well; however, with PBCas the underlying liver disease, she is an excellent candidate for transplantation. Patient D also has ascites as the majorproblem; however, the side-to-side portosystemic shunt is a far better long-term treatment option than PVS.

24. Which of the following explanations account(s) for the fact that hepatitis C is the most common cause ofposttransfusion hepatitis?A. There are more carriers of hepatitis C virus (HCV) in the normal population who serve as blood donors.B. Blood infected with hepatitis B virus (HBV) is eliminated through routine testing, leaving only HCV as the otherblood-borne pathogen.C. Current serologic tests for HCV antigen do not exclude carriers.D. Questions designed to eliminate risk groups for HCV from the normal donor population may not be as specific as

Page 12: Hepatobiliary & pancreatic surgery

would be desirable.E. Hepatitis C is a more virulent form of viral hepatitis, so it is expected that more cases of posttransfusion hepatitiswould occur.Asir Surgery MCQs Bank. © 1422H-2002- first impression ©

This project was raised after an idia by Dr. Gharama Al-Shehri (consultant surgeon). Developed and typed by Dr. Ghazi Al-Shumrani (intern).

Page 13: Hepatobiliary & pancreatic surgery

Answer: BDHepatobiliary & pancreatic surgery

DISCUSSION: The ability to specifically identify persons infected with HCV has only recently becomeavailable.Therefore, data about epidemiology are less than complete. It is very likely not true that more blood donors carry HCVbecause of the large preponderance of HBV in the United States. It is true, however, that successful elimination of mostof the HBV carriers occurs through routine testing. Although serologic tests are available for HCV, they are tests, not ofantigen, but of antibody. Therefore, this test alone may not screen out persons who are infected but have not yetdeveloped or may never develop antibody. Risk groups for the relatively newly defined HCV may well not becomprehensively established, and therefore this explanation may be a contributor. There are no differences in virulencebetween these classes of hepatitis virus.

25. True or false: HBV infections:A. Are usually asymptomatic.B. May not be clinically recognized but may lead to chronic hepatitis.C. Reliably protect against subsequent HBV infection regardless of the measured antibody titer to hepatitis B surfaceantigen (HBsAg).D. Are completely prevented by postexposure administration of HBIg hepatitis B immunoglobulin (HBIg).E. Preclude subsequent infection with HDV.Answer: TRUE: BC, FALSE ADE

DISCUSSION: Although some types of hepatitis are more often asymptomatic than symptomatic, that is not the casefor hepatitis B. Further, even if the HBV infection is asymptomatic, serious long-term side effects may occur. A priorinfection with hepatitis B confers lifelong immunity even if the antibody titer wanes below the protective level of 10mIU. HBIg is useful in reducing the incidence of postexposure HBV infection from around 30% with no intervention,to 15% with standard immune globulin, to about 5% to 7% with HBIg. HBV infection is required for infection withHDV and is therefore an essential step toward, rather than preventive of, HBV infection.

26. Which of the following statements about choledocholithiasis are correct?A. Common duct stones can originate in the gallbladder and migrate to the common duct, and stones can form de novoin the duct system.B. Calcium bilirubinate stones are associated with the presence of bacteria in the duct system.C. Common duct stones discovered at laparoscopic cholecystectomy should be treated by postoperative endoscopicextraction.D. The serum bilirubin value is usually greater than 15 mg. per dl. in the patient with a symptomatic common ductstone.Answer: ABC

DISCUSSION: Most common duct stones originate in the gallbladder and migrate to the common duct, where they maybecome larger. These stones tend to consist predominantly of cholesterol (about 80% of gallbladder stones arepredominantly cholesterol). Stones found in the bile ducts after cholecystectomy may have been overlooked, but denovo stone formation does occur. Arbitrarily, stones found 2 years after cholecystectomy are assumed to have formedwithin the duct system. Calcium bilirubinate stones are thought to result from precipitation of insoluble bilirubinmonoglucuronide formed by deconjugation of bilirubin diglucuronide, a reaction promoted by the enzyme beta-glucuronidase, which is produced by bacteria in the biliary tract. Calcium bilirubinate stones are found almostexclusively in patients who have some form of biliary tract lesion that causes partial obstruction, and these patients tendto have bactibilia. Stones smaller than approximately 5 mm. often can be extracted through a dilated cystic duct orpushed into the duodenum. Larger stones are best left for postoperative endoscopic sphincterotomy and extraction.Patients with more than five stones or stones larger than 1.5 cm. should be treated by open choledocholithotomy or,when indicated, a biliary-enteric anastomosis. Not all patients with symptomatic common duct stones have elevatedserum bilirubin, but when jaundice is present the bilirubin is only rarely greater than 15 mg. per dl.

27. A benign biliary duct stricture:A. Need not be treated unless it causes clinical jaundice.B. Should always be treated by percutaneous balloon drainage.C. Is prone to recur after treatment with biliary-enteric anastomosis.D. When due to chronic pancreatitis should be treated by side-to-side choledochoduodenostomy.Answer: CD

Page 14: Hepatobiliary & pancreatic surgery

DISCUSSION: Even a minor obstructing lesion in the extrahepatic duct system can produce cirrhosis over time, and thedevelopment of portal hypertension, ascites, and esophageal varices. Therefore, all biliary strictures should be treatedunless this is not possible or there is no chance for success. The presence or absence of jaundice is of no significance.Asir Surgery MCQs Bank. © 1422H-2002- first impression ©

This project was raised after an idia by Dr. Gharama Al-Shehri (consultant surgeon). Developed and typed by Dr. Ghazi Al-Shumrani (intern).

Page 15: Hepatobiliary & pancreatic surgery

Hepatobiliary & pancreatic surgeryOften, the only biochemical abnormality is mild elevation of alkaline phosphatase. The long-term results ofpercutaneous balloon dilatation are not yet known, but short-term results are good. Although some argue that balloondilatation should be the initial treatment, its role is ill-defined, and it should not be viewed as standard therapy at thistime. Biliary-enteric anastomoses are predisposed to stricture, for reasons that are ill-understood. A mucosa-to-mucosaanastomosis, large size of the anastomosis, a normal duct at the point of anastomosis, and stenting appear to be elementsthat work against stricture. About 70% of anastomoses are not complicated by strictures. Common duct stricturescaused by chronic pancreatitis are located in the distal portion of the duct and are easily treated by side-to-sidecholedochoduodenostomy. A wide anastomosis is usually possible, and because of this stenting often is not necessary.Although a Roux-en-Y biliary-enteric reconstruction is acceptable treatment, no advantage overcholedochoduodenostomy has been demonstrated.

28. Which statements about extrahepatic bile duct cancer are correct?A. Cholangiography is essential in evaluating patients for resectability.B. The prognosis is excellent when appropriate surgical and adjuvant therapy are given.C. The location of the tumor determines the type of surgical procedure.D. The disease usually becomes manifest by moderate to severe right-side upper quadrant pain.Answer: AC

DISCUSSION: Cholangiography is essential for both diagnosis and evaluation of resectability. Brushings of the lesionfor diagnosis and temporary stenting, done percutaneously or endoscopically, are often done at the time ofcholangiography. Angiography and CT are helpful, but in the absence of hepatic artery or portal vein occlusion thesetests are not accurate predictors of resectability. The primary obstacles to complete resection are invasion of the portalvein or the hepatic artery and proximal extension of the tumor into the liver. The long-range prognosis for patients whoundergo treatment for extrahepatic bile duct cancer is poor, even when the lesion is surgically resectable and adjuvanttherapy is given. Only about 10% of patients are alive without disease at 10 years. Nevertheless, bile duct cancer tendsnot to metastasize to distant sites, so resection and radiation therapy are useful in prolonging symptom-free life. Tumorsin the proximal third of the extrahepatic bile duct system are treated by a Roux-en-Y biliary-enteric anastomosis. Toensure excision of the entire tumor this anastomosis usually must be made to the individual hepatic ducts, which mustbe stented individually. Tumors of the middle third usually require anastomosis to the proximal hepatic duct. Incontrast, lesions of the distal third require Whipple's procedure with appropriate reconstruction. Thus, the treatment ofextrahepatic bile duct cancer depends on the location of the tumor. Pain is not a prominent feature of bile duct cancer.Most cases become manifest by the insidious development of jaundice.

29. Which of the following statements about biliary tract problems are correct?A. Choledochal cyst should be treated by Roux-en-Y cystojejunostomy.B. Sclerosing cholangitis is characterized by long, narrow strictures in the extrahepatic biliary duct system.C. Operative (needle) cholangiography is indicated in patients who at operation appear to have no gallbladder.D. The long cystic duct, which appears to be fused with the common duct and enters it distally, should be dissected freeand ligated at its entrance into the common duct.Answer: C

DISCUSSION: In the past, choledochal cyst was treated by Roux-en-Y cystojejunostomy, but long-term results werepoor. Excision of the cyst is essential to prevent recurrent pancreatitis. In addition, the development of carcinoma inabout 25% of patients mandates cyst excision. Accordingly, excision of the cyst with biliary reconstruction by Roux-en-Y hepaticojejunostomy and diversion of the flow of pancreatic juice through the ampulla of Vater is currently thestandard treatment. Sclerosing cholangitis causes fibrosis of bile ducts both within and outside the liver. This process,which is poorly understood, causes strictures in the duct system, characteristically with normal or dilated segmentsbetween strictures. Unfortunately, this anatomic arrangement does not lend itself to biliary reconstructive procedures.Each case must be analyzed, however, because in some patients the anatomic situation may lend itself to balloondilatation or reconstruction. When the gallbladder appears to be absent, a search should be made for an ectopicallylocated organ in the retroduodenal area, within the falciform ligament, and within the substance of the right lobe of theliver. With true gallbladder agenesis the common duct may be dilated, and choledocholithiasis is present in about onefourth of those who undergo operation. Therefore, operative needle cholangiography should always be done. Dissectionof a long, fused cystic duct is fraught with hazard because the cystic and common ducts may share a common wall andserious duct damage may occur. The cystic duct should be ligated and divided immediately proximal to the area offusion.

30. Which of the following statements about the diagnosis of acute calculous cholecystitis are true?

Page 16: Hepatobiliary & pancreatic surgery

A. Pain is so frequent that its absence almost precludes the diagnosis.B. Jaundice is present in a majority of patients.C. Ultrasonography is the definitive diagnostic test.Asir Surgery MCQs Bank. © 1422H-2002- first impression ©

This project was raised after an idia by Dr. Gharama Al-Shehri (consultant surgeon). Developed and typed by Dr. Ghazi Al-Shumrani (intern).

Page 17: Hepatobiliary & pancreatic surgery

D. Cholescintigraphy is the definitive diagnostic test.Answer: AD

Hepatobiliary & pancreatic surgery

DISCUSSION: The presence of pain is the sine qua non of acute calculous cholecystitis. Chronic cholecystitisassociated with cholelithiasis may develop in the absence of pain, and in critically ill patients pain may not be aprominent feature of acute acalculous cholecystitis. Only about 10% of patients with acute cholecystitis are jaundiced.Although an occasional patient may have concomitant bile duct obstruction, the jaundice associated with acutecholecystitis is probably due to absorption of bile pigments from the diseased gallbladder. The presence of jaundice in apatient with right-side upper quadrant pain should also suggest the possibility of acute cholangitis secondary to bile ductobstruction. Ultrasonography is very accurate in the detection of gallstones, but stones may be present in the absence ofacute cholecystitis. Thickening of the gallbladder wall and a collection of fluid around the gallbladder areultrasonographic findings in some patients with acute cholecystitis, but they are not always present and are not specific.Ultrasonography may be useful when the diagnosis is obscure because other conditions in the liver, pancreas, andkidney can be detected; however, it is not the definitive test for acute cholecystitis. Cholescintigraphy is specific for thediagnosis of acute calculous cholecystitis (accuracy over 95% in experienced hands). The rapidity, simplicity, andaccuracy make cholescintigraphy the definitive diagnostic test in acute calculous cholecystitis; however, it must beinterpreted cautiously in the context of another critical illness or recent surgery or trauma, because false-positives arenot unusual in these situations.

31. Which statements about acute acalculous cholecystitis are correct?A. The disease is often accompanied by or associated with other conditions.B. The diagnosis is often difficult.C. The mortality rate is higher than that for acute calculous cholecystitis.D. The disease has been treated successfully by percutaneous cholecystostomy.Answer: ABCD

DISCUSSION: About half of the cases of acute acalculous cholecystitis are associated with other conditions, includingsepsis, sarcoidosis, polyarteritis nodosa, and systemic lupus erythematosus. A majority of cases occur after trauma,burns, or major surgical procedures performed for other conditions. The precise pathogenesis has not been determined.The diagnosis of acute acalculous cholecystitis is often difficult because symptoms may be masked by another illness,injury, or the postoperative state. Unlike acute calculous cholecystitis, in which pain is always present, pain occurs inonly about 70% of cases. In addition, cholescintigraphy is sometimes inaccurate. These factors make the diagnosisdifficult, and a high index of suspicion is necessary, especially in patients who have had operations or trauma.Unexplained abdominal pain, sepsis, and ileus should prompt a thorough investigation. The mortality rate for acuteacalculous cholecystitis is higher than that of the calculous type. The incidence of gangrene and perforation of thegallbladder is higher. The accompanying illnesses and conditions and the frequent delays in diagnosis undoubtedlycontribute to the higher death rate. Percutaneous cholecystostomy has been used as a diagnostic and therapeuticmaneuver in patients who are thought to have acute acalculous cholecystitis. Aspiration and culture of bile assist inconfirming the diagnosis, and continuous drainage successfully treats the acute condition. Surprisingly, persistentgangrene and subsequent complications have been infrequent. Immediate cholecystectomy should be done if significantimprovement does not take place within 12 hours of percutaneous cholecystostomy. Long-term management of the tubeand the need for elective cholecystectomy must be individualized. The experience with percutaneous cholecystostomyis too small to determine whether this technique reduces the mortality rate.

32. True statements about the surgical management of patients with acute calculous cholecystitis include:A. Operation should be performed in all patients as soon as the diagnosis is made.B. Antibiotic therapy should be initiated as soon as the diagnosis is made.C. Dissection of the gallbladder is facilitated by decompression of the organ with the use of a trocar.D. An operative cholangiogram should be done in every patient.Answer: BC

DISCUSSION: Cholecystectomy should be done in an otherwise healthy patient as soon as the diagnosis is made andthe patient is properly prepared for surgery. However, patients who have one or more significant risk factors such as arecent myocardial infarction, unstable angina, clinically significant coronary artery disease, or cirrhosis should not haveimmediate cholecystectomy unless they do not improve within 24 to 36 hours in response to antibiotic administrationand supportive care. Antibiotic administration should commence as soon as the diagnosis is made and should becontinued for 24 hours postoperatively—or for 7 days if significant peritonitis is present. This use of antibiotics hassignificantly reduced septic complications after cholecystectomy for acute cholecystitis. In most cases the gallbladder istensely distended, making visualization and dissection of the cystic duct area difficult and perhaps dangerous.

Page 18: Hepatobiliary & pancreatic surgery

Decompression of the gallbladder by insertion of a needle facilitates retraction and dissection of the gallbladder.Although some advise that operative cholangiography be done only on a selective basis, its routine use helps todelineate anatomy and facilitates detection of an occasionally unsuspected bile duct stone. Accordingly, it is usedAsir Surgery MCQs Bank. © 1422H-2002- first impression ©

This project was raised after an idia by Dr. Gharama Al-Shehri (consultant surgeon). Developed and typed by Dr. Ghazi Al-Shumrani (intern).

Page 19: Hepatobiliary & pancreatic surgery

Hepatobiliary & pancreatic surgeryroutinely in elective cases. In acute cholecystitis, however, the biliary duct system may be very friable, and operativecholangiography should be done only when it is safe to do so.

33. Which of the following are indications for cholecystectomy?A. The presence of gallstones in a patient with intermittent episodes of right-side upper quadrant pain.B. The presence of gallstones in an asymptomatic patient.C. The presence of symptomatic gallstones in a patient with angina pectoris.D. The presence of asymptomatic gallstones in a patient who has insulin-dependent diabetes.Answer: A

DISCUSSION: Cholecystectomy (and concomitant operative cholangiography) are indicated for symptomatic patientsto relieve pain and to prevent the development of acute cholecystitis and its complications. Morbidity and expense arenot as great for elective cholecystectomy as they are for cholecystectomy for acute cholelithiasis. The risk of thedevelopment of symptoms in patients who have asymptomatic stones is approximately 2% per year, a rate associatedwith mortality and morbidity that do not exceed those of elective cholecystectomy. Therefore, cholecystectomy is notindicated for asymptomatic patients. Patients who have angina pectoris should not have cholecystectomy until theircoronary artery disease has been treated adequately, even if this requires a coronary artery bypass procedure. Heartdisease is the most frequent cause of death after cholecystectomy. Prophylactic cholecystectomy, formerlyrecommended for insulin-dependent diabetics, is not indicated because several studies have shown that the mortalityrate from acute cholecystitis is no higher for diabetics than for nondiabetics.

34. Which of the following statements about laparoscopic cholecystectomy are correct?A. The procedure is associated with less postoperative pain and earlier return to normal activity.B. The incidence of bile duct injury is higher than for open cholecystectomy.C. Laparoscopic cholecystectomy should be used in asymptomatic patients because it is safer than opencholecystectomy.D. Pregnancy is a contraindication.Answer: AB

DISCUSSION: Studies have clearly documented that postoperative pain following laparoscopic cholecystectomy is lessthan that experienced after open cholecystectomy and that patients can resume normal activity sooner. This appears tobe related to the reduced trauma to the abdominal wall by virtue of the very small incisions used in laparoscopicprocedures. The best evidence is that the bile duct injury rate (0.4%) is approximately double that for opencholecystectomy. The incidence of this serious complication will probably decrease with improved techniques, bettertraining, and more advanced instrumentation. Only symptomatic patients should have cholecystectomy. Prophylacticremoval of the gallbladder is not cost effective. All elective operations are contraindicated in the first trimester, so as toprevent fetal anomalies and spontaneous abortion. The laparoscopic technique is not contraindicated thereafter except inpatients in whom peritoneal access cannot safely be established. This is rarely a problem. Premature labor is a risk inthe third trimester. Thus, unless cholecystectomy can be avoided altogether during pregnancy, the second trimester isthe most propitious time.

35. Which of the following statements about cholangitis are correct?A. Charcot's triad is always present.B. Associated biliary tract disease is always present.C. Chills and fever are due to the presence of bacteria in the bile duct system.D. The most common cause of cholangitis is choledocholithiasis.Answer: BCD

DISCUSSION: Although Charcot's triad (pain, chills and fever, jaundice) is diagnostic of cholangitis, the complete triadoccurs only in 50% to 70% of patients. Fever is the most common symptom; therefore, cholangitis should be consideredin all patients who have unexplained fever. Episodes of pain, chills, and fever are often so brief as not to concern thepatient. Cholangitis does not occur in the absence of partial or complete bile duct obstruction. All patients diagnosed ashaving cholangitis should have appropriate diagnostic studies to determine the cause. This usually involvescholangiography. The presence of bacteria in bile does not produce symptoms in the absence of partial or completeobstruction of the bile duct system. When obstruction is present, pressure within the system increases, giving rise toreflux of bacteria or their toxic products into the hepatic venous circulation. This cholangiovenous reflux produceschills, fever, and the hemodynamic changes of sepsis. Death may ensue if treatment is not instituted promptly.Choledocholithiasis, the most commonly associated problem, may produce partial or complete obstruction. When

Page 20: Hepatobiliary & pancreatic surgery

bacteria are not present in the bile duct system, choledocholithiasis may go undetected unless the degree of obstructionis sufficient to cause jaundice. Other causes of cholangitis are benign and malignant strictures, biliary-entericanastomoses, invasive procedures, foreign bodies, and parasitic infestation of the bile ducts.Asir Surgery MCQs Bank. © 1422H-2002- first impression ©

This project was raised after an idia by Dr. Gharama Al-Shehri (consultant surgeon). Developed and typed by Dr. Ghazi Al-Shumrani (intern).

Page 21: Hepatobiliary & pancreatic surgery

36. Recurrent episodes of cholangitis:A. Suggest the presence of undetected or overlooked bile duct pathology.B. Occur frequently in patients who have indwelling biliary tubes or stents.C. May be ameliorated by long-term administration of antibiotics.D. May be associated with the development of secondary biliary cirrhosis.Answer: ABCD

Hepatobiliary & pancreatic surgery

DISCUSSION: Cholangitis does not occur in the presence of a normal bile duct system, and all patients with cholangitishave an abnormality. Thus, recurrent episodes of cholangitis signal the need for diagnostic studies. Cholangiographyusually will be necessary. The presence of any foreign body in the biliary tract is frequently associated with bactibiliaand recurrent episodes of cholangitis. Even a silk suture exposed to the lumen of a bile duct has been known to causecholangitis. Pigment stone and sludge formation may result from the bacterial deconjugation of bilirubin diglucuronideto bilirubin monoglucuronide, which precipitates as calcium bilirubinate. This material can occlude indwelling tubesand predispose to more frequent episodes of cholangitis. Long-term administration of an oral antibiotic may reduce thefrequency and severity of attacks of cholangitis; however, this method of management should not be routine. Correctionof the underlying problem is essential. Chronic obstruction and recurrent infection eventually lead to secondary biliarycirrhosis and its complications of portal hypertension, ascites, and bleeding esophageal varices. Once this stage of thedisease is reached, correction of the underlying biliary tract problem does not reverse the changes in the liver. Onceagain, every effort should be made to eliminate the cause of the cholangitis early in the course of disease. The onlyeffective treatment for end-stage liver disease is hepatic transplantation.

37. The initial goal of therapy for acute toxic cholangitis is to:A. Prevent cholangiovenous reflux by decompressing the duct system.B. Remove the obstructing stone, if one is present.C. Alleviate jaundice and prevent permanent liver damage.D. Prevent the development of gallstone pancreatitis.Answer: A

DISCUSSION: Uncontrolled sepsis and the consequent multisystem organ failure are the life-threatening sequelae ofacute toxic cholangitis. Thus, the initial goal of treatment is to decompress the biliary duct system to prevent reflux ofbacteria and their toxic products into the circulation. This can be done by intubating the duct system through thepercutaneous, transhepatic, or the endoscopic route or by insertion of a T tube in the common duct at operation.Removal of the stone causing the obstruction is not necessary to stabilize the patient. Only after the duct isdecompressed should the cause of the obstruction be addressed. When transhepatic biliary drainage has been used,endoscopic or surgical removal of the stone can be carried out after the patient has recovered completely. When initialtherapy is sphincterotomy, the stone should be removed as part of the procedure. Often the stone falls out withoutmanipulation. If surgical placement of a T tube is the initial treatment, the stone should be removed only if it isconvenient to do so. The long-range goal of treatment of patients with bile duct obstruction is to prevent cirrhosis,ascites, portal hypertension, and hemorrhage from esophageal varices; however, death from sepsis is the immediatethreat in acute toxic cholangitis. Gallstone pancreatitis may occur in patients who have an impacted stone in the distalduct, independent of the presence or absence of acute toxic cholangitis; however, gallstone pancreatitis is more oftenassociated with the passage of a stone into the duodenum.

38. The clinical picture of gallstone ileus includes which of the following?A. Air in the biliary tree.B. Small bowel obstruction.C. A stone at the site of obstruction.D. Acholic stools.E. Associated bouts of cholangitis.Answer: ABCE

DISCUSSION: An antecedent biliary-enteric fistula is necessary to allow stone migration into the intestinal tract, andthis results in air entering the biliary tree (pneumobilia). It also allows contamination of the bile ducts with intestinalbacteria, which in fact occurs in only a minority of such cases. The stone obstructs the narrower distal bowel, producingsmall bowel obstruction. Such a stone, if opaque, can be seen on plain radiography and, if not, can be appreciated bysonography. Stools are not acholic, since the cholecystoenteric fistula allows bile access to the intestinal lumen.

Page 22: Hepatobiliary & pancreatic surgery

39. Which of the following statement(s) about gallstone ileus is/are not true?A. The condition is seen most frequently in women older than 70.Asir Surgery MCQs Bank. © 1422H-2002- first impression ©

This project was raised after an idia by Dr. Gharama Al-Shehri (consultant surgeon). Developed and typed by Dr. Ghazi Al-Shumrani (intern).

Page 23: Hepatobiliary & pancreatic surgery

B. Concomitant with the bowel obstruction, air is seen in the biliary tree.Hepatobiliary & pancreatic surgery

C. The usual fistula underlying the problem is between the gallbladder and the ileum.D. When possible, relief of small bowel obstruction should be accompanied by definitive repair of the fistula sincethere is a significant incidence of recurrence if the fistula is left in place.E. Ultrasound studies may be of help in identifying a gallstone as the obstructing agent.Answer: C

DISCUSSION: It is true that gallstone ileus occurs mostly in elderly women and should always be suspect when smallbowel obstruction presents in this age group. The great majority of cases of gallstone ileus are preceded by aspontaneous fistula occurring between the gallbladder and duodenum, allowing gallstones to enter the intestinal tract,which can potentially block the terminal ileum. Finding air within the biliary tree should always arouse suspicion of thepossibility of this diagnosis when it is associated with a radiographic pattern of small bowel obstruction. The initial partof the operative approach to this disease is to relieve the bowel obstruction by performing an enterotomy just proximalto the point of obstruction to remove the stone. Where possible, definitive repair of the fistula should be undertaken toavoid recurrent obstruction and to obviate the possible recurring complications of cholangitis. Percutaneous drainage ofbile collections combined with endoscopic papillotomy may be sufficient treatment for external and internal biliaryfistulas but is never an allowable approach in the presence of gallstone ileus with small bowel obstruction. Relief of theobstruction is mandated in this setting.

40. Which of the following lesions are believed to be associated with the development of carcinoma of the gallbladder?A. Cholecystoenteric fistula.B. A calcified gallbladder.C. Adenoma of the gallbladder.D. Xanthogranulomatous cholecystitis.E. All of the above.Answer: E

DISCUSSION: The prevalence of carcinoma of the gallbladder in patients who have or have had a cholecystoentericfistula is believed to be 15%. The prevalence of carcinoma in a calcified, or “porcelain,” gallbladder is reported to rangefrom 12.5% to 61%. It is generally accepted that adenoma of the gallbladder is a precancerous lesion that presents as apolypoid lesion. Xanthogranulomatous cholecystitis is a rare form of chronic cholecystitis believed to be associatedwith a higher incidence of cancer. This form of cholecystitis is also important because, grossly, it may mimic cancer ofthe gallbladder.

41. The preferred treatment for carcinoma of the gallbladder is:A. Radical resection that includes gallbladder in continuity with the right hepatic lobe and regional lymph nodedissection.B. Radiation therapy.C. Chemotherapy.D. Combined treatment involving surgical therapy, chemotherapy, and radiation.E. None of the above.Answer: E

DISCUSSION: Radical resection, radiation therapy, and chemotherapy have been effective only anecdotally. Mostbelieve that the dismal prognosis of carcinoma of the gallbladder does not justify anything more than palliativetreatment. About 88% of patients are dead within a year of diagnosis, and only about 4% are alive after 5 years,regardless of the type of treatment. Those whose surgeon was unaware of the presence of the tumor at the time ofcholecystectomy (approximately 12% of cases) are most likely to survive long term. There are insufficient data tosupport conclusively the proposition that the patient with unexpected carcinoma found on histologic examination shouldundergo reoperation with intent for radical excision. There also are indirect suggestions that the prognosis of gallbladdercarcinoma may be improving, but it is not clear if this is spontaneous or due to either earlier diagnosis or surgicalmanagement.

42. Which of the following statement(s) about pancreatic embryonic malformations is/are correct?A. Pancreas divisum can be a cause of gastrointestinal bleeding.B. Heterotopic pancreatic tissue predisposes to pancreatic adenocarcinoma.C. Annular pancreas may cause gastrointestinal obstruction in children or in adults.D. Relative obstruction to the flow of pancreatic juice through the minor papilla appears to be the cause of pancreatitis

Page 24: Hepatobiliary & pancreatic surgery

in some patients with pancreas divisum.Answer: CD

Asir Surgery MCQs Bank. © 1422H-2002- first impression ©

This project was raised after an idia by Dr. Gharama Al-Shehri (consultant surgeon). Developed and typed by Dr. Ghazi Al-Shumrani (intern).

Page 25: Hepatobiliary & pancreatic surgery

Hepatobiliary & pancreatic surgeryDISCUSSION: The clinically recognized embryonic malformations of the pancreas include heterotopic pancreas,pancreas divisum, and annular pancreas. Heterotopic pancreatic tissue most often takes the form of a firm nodule ofvariable size in the stomach, duodenum, small bowel, or Meckel's diverticulum. The typical complications ofheterotopic pancreas include intestinal obstruction, ulceration, or hemorrhage. Pancreas divisum is an anatomic variantthat results from failure of fusion of the two primordial pancreatic duct systems. In pancreas divisum the major portionof the pancreas is drained via the duct of Santorini through the minor duodenal papilla. Relative stenosis of the minorduodenal papilla can cause pancreatitis. Pancreas divisum is not associated with gastrointestinal bleeding. Annularpancreas results when histologically normal pancreatic tissue completely or partially encircles the second portion of theduodenum. Varying degrees of duodenal obstructive symptoms may be observed in both children and adults with thiscondition.

43. The pancreas occupies a retroperitoneal position in the upper abdomen. Which statement(s) is/are correct?A. The superior mesenteric vein and the splenic vein join to form the portal vein posterior to the neck of the pancreas.B. The uncinate process of the pancreas extends posterior to the inferior vena cava.C. The tail of the pancreas extends to the left of the aorta, toward the splenic hilum.D. The head of the pancreas is jointly supplied by arterial blood from the celiac axis and the superior mesenteric artery.Answer: ACD

DISCUSSION: The pancreas occupies a retroperitoneal position in the upper abdomen, extending obliquely from theduodenal C loop to a more cephalad position where the pancreatic tail abuts the hilum of the spleen. The portion of thepancreas anterior to the confluence of the superior mesenteric vein, splenic vein, and portal vein is designated the neckof the gland. The uncinate process extends posterior to the superior mesenteric vein and approaches the superiormesenteric artery. The head of the pancreas is intimately associated with the second portion of the duodenum, and thesetwo structures are jointly supplied by two arterial arcades known as the anterior and posterior pancreaticoduodenalarteries, which originate as branches of the celiac axis and superior mesenteric artery.

44. Both endocrine and exocrine tissue comprise the pancreas. Which statement(s) is/are true?A. The islets of Langerhans total 1 million per gland and drain their secretions via intercalated duct cells through theampulla of Vater.B. Islet alpha cells produce glucagon.C. Islet sigma cells produce somatostatin.D. The acini and ductal systems constitute the exocrine portion of the pancreas.Answer: BD

DISCUSSION: The endocrine portion of the pancreas is served by the islets of Langerhans, which number 1 millionislets per gland. The islets of Langerhans drain their endocrine secretions into the bloodstream. Insulin-producing betacells comprise the majority of the islet population. Alpha cells produce glucagon and constitute approximately 20% to25% of the total islet cell number. Delta cells of the islets produce somatostatin. The acini and ductal systems constitutethe exocrine portion of the pancreas. The acinar cells contain zymogen granules in their narrow, centrally located apicalportion. The pancreatic duct system includes intercalated duct cells along the ductal pathway, terminating in the mainexcretory duct of the pancreas.

45. Pancreatic exocrine secretory products include a bicarbonate-rich electrolyte solution as well as digestive enzymes.Which of the following statement(s) is/are true?A. Cholecystokinin (CCK) is the most potent endogenous stimulant of pancreatic enzyme secretion.B. The chloride and bicarbonate concentrations of pancreatic juice vary and depend on the secretory flow rate.C. Secretin is the most potent endogenous stimulant of pancreatic water and electrolyte secretion.D. The peptidases synthesized by acinar cells are released into the pancreatic duct system in active form.Answer: ABC

DISCUSSION: CCK is the most potent endogenous stimulant of pancreatic enzyme secretion. The pancreatic acinarcells respond to CCK with release of their zymogen granules into the ductal system. Peptidases are released in inactiveform, later to be activated by contact with duodenal enterokinase and activated trypsin. Secretin is the most potentendogenous stimulant of pancreatic water and electrolyte secretion. The concentrations of the anions bicarbonate andchloride vary and are largely dependent on the secretory flow rate stimulated by secretin.

46. Which of the following parameters is/are not included in the Ranson's prognostic signs useful in the early

Page 26: Hepatobiliary & pancreatic surgery

evaluation of a patient with acute pancreatitis?A. Elevated blood glucose.B. Leukocytosis.Asir Surgery MCQs Bank. © 1422H-2002- first impression ©

This project was raised after an idia by Dr. Gharama Al-Shehri (consultant surgeon). Developed and typed by Dr. Ghazi Al-Shumrani (intern).

Page 27: Hepatobiliary & pancreatic surgery

C. Amylase value greater than 1000 U per dl.D. Serum lactic dehydrogenase (LDH) greater than 350 IU per dl.E. Alanine aminotransferase greater than 250 U per dl.Answer: CE

Hepatobiliary & pancreatic surgery

DISCUSSION: Several prognostic systems have been demonstrated to predict the severity of pancreatitis accurately.Two Ranson prognostic criteria have been developed: one each, for pancreatitis that is not due to gallstones andpancreatitis that is. The systems have minor differences. In both of the Ranson systems elevated blood glucose,leukocytosis, and elevations of serum LDH have proved to have prognostic importance. The degree of amylaseelevation is not one of the parameters, nor is the degree of ALT elevation.

47. Standard supportive measures for patients with mild pancreatitis include the following:A. Intravenous fluid and electrolyte therapy.B. Withholding of analgesics to allow serial abdominal examinations.C. Subcutaneous octreotide therapy.D. Nasogastric decompression.E. Prophylactic antibiotics.Answer: A

DISCUSSION: Standard therapy for all patients with mild acute pancreatitis should include intravenous fluidresuscitation, electrolyte replacement, and analgesics. Nasogastric decompression is typically reserved for patients withsignificant ileus who are at risk for emesis and aspiration. Subcutaneous therapy with octreotide, the octapeptide analogof somatostatin, has not been proven to influence the outcome in patients with mild pancreatitis. Prophylactic antibioticsare not used for mild pancreatitis. Antibiotics are reserved for patients with severe pancreatitis (defined as greater thanthree Ranson prognostic signs with associated CT evidence of pancreatic or peripancreatic necrosis).

48. Which of the following statements about chronic pancreatitis is/are correct?A. Chronic pancreatitis is the inevitable result after repeated episodes of acute pancreatitis.B. Patients with chronic pancreatitis commonly present with jaundice, pruritus, and fever.C. Mesenteric angiography is useful in the evaluation of many patients with chronic pancreatitis.D. Total pancreatectomy usually offers the best outcome in patients with chronic pancreatitis.E. For patients with disabling chronic pancreatitis and a dilated pancreatic duct with associated stricture formation, alongitudinal pancreaticojejunostomy (Peustow procedure) is an appropriate surgical option.Answer: E

DISCUSSION: Chronic pancreatitis is a clinical entity that includes recurrent or persistent abdominal pain withevidence of exocrine and endocrine pancreatic insufficiency. While chronic pancreatitis may result from repeatedepisodes of acute pancreatitis, not all patients with recurring acute pancreatitis progress to chronic pancreatitis. Themost common causes of chronic pancreatitis include alcohol abuse, hyperparathyroidism, congenital anomalies of thepancreatic duct, pancreatic trauma, and cystic fibrosis. The most useful radiographic tests in patients with suspectedchronic pancreatitis are CT and endoscopic retrograde cholangiopancreatography (ERCP). Mesenteric angiography hasno role in the evaluation of most patients with chronic pancreatitis. Patients with disabling chronic pancreatitis whorequire operative intervention are candidates for a longitudinal pancreaticojejunostomy (Peustow procedure) ifpancreatography demonstrates a dilated pancreatic duct. Total pancreatectomy is rarely performed because of thesignificant problems associated with labile insulin sensitivity, steatorrhea, and weight loss.

49. Which of the following statements about pancreatic ascites is/are correct?A. Patients typically present with painful ascites, reflecting the release of toxic pancreatic enzymes into the peritonealcavity.B. The standard evaluation of a patient with new-onset ascites includes abdominal paracentesis. In cases of pancreaticascites, the peritoneal fluid contains high concentrations of both amylase and protein.C. Pancreatic ascites can follow an episode of acute pancreatitis.D. Patients with pancreatic ascites may fail to improve with nonoperative therapy and require surgical procedures. Atabdominal exploration an acceptable approach to the pancreatic duct disruption involves suture ligation with omentalpatching.Answer: BC

DISCUSSION: Pancreatic ascites typically occurs because of a pancreatic duct disruption, most commonly involving

Page 28: Hepatobiliary & pancreatic surgery

alcohol abuse and resultant acute pancreatitis. In pancreatic ascites, pancreatic exocrine secretions exit a pancreatic ductdisruption and drain anteriorly into the peritoneal cavity. Patients typically present with painless massive ascites, as thepancreatic enzymes that extravasate into the peritoneal cavity are typically nonactivated. The diagnosis of pancreatic

Asir Surgery MCQs Bank. © 1422H-2002- first impression ©

This project was raised after an idia by Dr. Gharama Al-Shehri (consultant surgeon). Developed and typed by Dr. Ghazi Al-Shumrani (intern).

Page 29: Hepatobiliary & pancreatic surgery

Hepatobiliary & pancreatic surgeryascites is best made by paracentesis, in which the analysis of the ascites fluid reveals it to be high in amylase (more than1000 U. per dl.) and high in albumin (more than 3 gm. per dl.). Nonoperative treatment is initially indicated in mostpatients with pancreatic ascites. Should nonoperative therapy fail, surgical therapy is directed to closure of thepancreatic duct disruption. Preoperative pancreatography is useful in directing surgical therapy. Distal pancreatic ductdisruption may be treated with distal pancreatectomy or with Roux-en-Y pancreaticojejunostomy. Pancreatic leaks inthe more proximal aspects of the gland are treated with Roux-en-Y pancreaticojejunostomy. Suture ligation of thepancreatic duct with omental patching is not considered appropriate therapy for pancreatic duct disruptions.

50. Which of the following statements about adenocarcinoma of the pancreas is/are correct?A. It is the fifth most common cause of cancer death in the U.S.B. Most cases occur in the body and tail of the pancreas, making distal pancreatectomy the most commonly performedresectional therapy.C. For cancers of the head of the pancreas resected by pancreaticoduodenectomy, prognosis appears to be independentof nodal status, margin status, or tumor diameter.D. The most accurate screening test involves surveillance of stool for carbohydrate antigen (CA 19–9).Answer: A

DISCUSSION: Adenocarcinoma of the pancreas is newly diagnosed in approximately 28,000 patients in the UnitedStates every year. It is the fifth most common cause of cancer death in the United States, exceeded only by lung,colorectal, breast, and prostate cancer. The majority of cases of adenocarcinoma of the pancreas occur in the head of thegland, and if resectable, are treated via pancreaticoduodenectomy. Recent studies have shown that factors favoringlong-term survival after pancreaticoduodenectomy for adenocarcinoma of the head of the pancreas include negativenodal status, negative margin status, small tumor diameter, and diploid DNA content. No accurate screening tests foradenocarcinoma of the pancreas are currently available. The best serologic test appears to be the CA 19–9, which iselevated in the majority of patients with adenocarcinoma of the head of the pancreas. Unfortunately, the test is notsufficiently sensitive or specific, and further screening tests are needed.

51. A 35-year-old woman presents with episodes of obtundation, somnolence, and tachycardia. An insulinoma issuspected based on a random serum glucose test value of 38 mg. per dl. Which of the following statements is/are true?A. The most important diagnostic study for insulinoma is an oral glucose tolerance test.B. It may be helpful to perform ERCP in an effort to localize the tumor.C. Most patients with insulinoma present with extensive disease, rendering them only rarely resectable or curable.D. An important component of the preoperative evaluation in patients with presumed insulinoma involves confirmingelevated C-peptide or proinsulin levels and screening for anti-insulin antibodies.Answer: D

DISCUSSION: Insulinoma is the most common endocrine tumor of the pancreas. Insulinoma is associated withWhipple's triad, which consists of (1) symptoms of hypoglycemia at fasting; (2) documentation of blood glucose levelsof less than 50 mg. per dl.; and (3) relief of symptoms following administration of glucose. The most reliable methodfor diagnosing insulinomas is a monitored fast. Neither an oral or an intravenous glucose tolerance test is indicated inthe majority of patients being evaluated for insulinoma. Support for the diagnosis of insulinoma can come fromdocumenting elevated C peptide and proinsulin levels. Screening for anti-insulin antibodies is indicated to rule out thepossibility of surreptitious insulin administration. Tumor localization is typically performed with CT, endoscopicultrasonography, or angiography. ERCP is not indicated for evaluation of most pancreatic endocrine tumors, as thetumors only rarely communicate with the main pancreatic duct system. As many as 90% of patients with insulinomahave benign solitary pancreatic adenomas amenable to surgical cure.

52. Which of the following statements about gastrinoma (Zollinger-Ellison syndrome) is/are correct?A. As many as 75% of gastrinoma patients have sporadic disease; 25% have gastrinoma associated with multipleendocrine neoplasia type 1 (MEN 1).B. Extrapancreatic gastrinomas are common, and exploration should include careful assessment of the duodenum andperipancreatic lymph nodes.C. Diarrhea may be a prominent presenting feature of some patients with gastrinoma.D. Before elective operation acid-reducing medications such as omeprazole should be administered.Answer: ABCD

DISCUSSION: Gastrinoma patients typically present with peptic ulceration of the upper gastrointestinal tract andabdominal pain. As many as 50% of patients may have diarrhea, which may be a prominent feature in some cases.

Page 30: Hepatobiliary & pancreatic surgery

Approximately 25% of gastrinoma patients have the disease associated with the MEN-1 syndrome, whereas 75% have asporadic variety of the disease. Recent evidence indicates that extrapancreatic gastrinomas are common. Carefulattention must be paid to the duodenum and peripancreatic lymph nodes at the time of abdominal exploration. BeforeAsir Surgery MCQs Bank. © 1422H-2002- first impression ©

This project was raised after an idia by Dr. Gharama Al-Shehri (consultant surgeon). Developed and typed by Dr. Ghazi Al-Shumrani (intern).

Page 31: Hepatobiliary & pancreatic surgery

Hepatobiliary & pancreatic surgeryelective operation it is imperative that the gastric acid hypersecretion be controlled. The control of gastrichypersecretion is best performed by the administration of one of the substituted benzimidazoles, such as omeprazole orlansoprazole.

53. With regard to the control of pancreatic exocrine function, which of the following statement(s) is/are correct?

a. Cholecystokinin, a hormone released from the duodenal mucosa, is the predominant stimulus for pancreaticenzyme secretion

b.Gastrin is a major stimulant for pancreatic bicarbonate secretionc. Secretin is released from the duodenum upon mucosal acidification and stimulates pancreatic bicarbonate

secretiond.Acetylcholine, released from pancreatic nerves, stimulates enzyme secretion

Answer: a, c, d

Enzyme secretion is regulated primarily through hormonal and neural factors. The enteric hormone cholecystokinin,released from endocrine cells in the duodenal mucosa, is the predominant regulator and stimulates acinar cells throughspecific membrane-bound receptors. Acetylcholine strongly stimulates acinar cells when released from postganglionicfibers of the pancreatic plexus and acts in synergy with CCK to potentiate enzyme secretion. Secretin weakly stimulatesacinar cell secretion and potentiates the effect of cholecystokinin on the acinar cells.Bicarbonate is formed from carbonic acid by the enzyme carbonic anhydrase. Secretin, the major stimulant forbicarbonate secretion, is released from the duodenal mucosa in response to a duodenal luminal pH of less than 3.0.Cholecystokinin only weakly stimulates bicarbonate secretion, whereas it potentiates secretin-stimulated bicarbonatesecretion. Gastrin and acetylcholine are weak stimulants of bicarbonate secretion.

54. In the performance of a pancreaticoduodenectomy (Whipple procedure), the superior mesenteric vein is animportant landmark. Which of the following statements is/are true with regard to the superior mesenteric vein?

a. Small venous branches enter the superior mesenteric vein anteriorly as it courses beneath the neck of thepancreas

b.The superior mesenteric vein joins the splenic vein at the superior border of the pancreas to form the portalvein

c. Small venous branches enter the superior mesenteric vein laterally as it courses beneath the neck of thepancreas

d.The superior mesenteric vein courses anterior to the neck of the pancreasAnswer: b, c

The venous drainage of the pancreas and duodenum follows the arterial supply. The anterior and posterior venousarcades drain the head; the body and tail drain into the splenic vein. All venous effluent from the pancreas ultimatelydrains into the portal vein which is formed by the confluence of the superior mesenteric vein and the splenic vein at thesuperior border of the pancreas. The anterior and posterior venous arcades in the head of the pancreas drain directly intothe suprapancreatic portal vein. The anteroinferior pancreaticoduodenal arcades drain with the right gastroepiploic veinto form a common venous trunk with the right colic vein. This trunk is known as the gastrocolic trunk and enters thesuperior mesenteric vein at the inferior border of the neck of the pancreas. The posteroinferior venous arcade emptiesdirectly into the superior mesenteric vein. The veins of the head drain laterally into the superior mesenteric and portalveins. There are no venous tributaries entering the superior mesenteric vein anteriorly. For this reason, it is safe todissect the neck of the pancreas directly anterior to the superior mesenteric and portal veins when performing apancreaticoduodenectomy.

55. Pancreas divisum results from incomplete fusion of the ventral pancreatic duct with the dorsal pancreatic ductduring embryologic development. Which of the following statements correctly describes pancreas divisum?

a. The body and tail of the pancreas drain via an accessory ampulla distal to the ampulla of Vater. The uncinateprocess drains via the ampulla of Vater

b.The entire pancreatic ductal system drains via the ampulla of Vaterc. The entire pancreatic ductal system drains via an accessory ampulla proximal to the ampulla of Vaterd.The body and tail of the pancreas are absent. The uncinate process drains via the ampulla of Vater

Answer: c

Page 32: Hepatobiliary & pancreatic surgery

In 90% of individuals, the main pancreatic duct, or duct of Wirsung, runs the entire length of the pancreas and joins thecommon bile duct to empty into the duodenum at the ampulla of Vater. The pancreatic duct is 2 to 3.5 mm in diameterand contains 20 secondary branches, which drain the tail, body, and uncinate process. The drainage of the lesser duct, orAsir Surgery MCQs Bank. © 1422H-2002- first impression ©

This project was raised after an idia by Dr. Gharama Al-Shehri (consultant surgeon). Developed and typed by Dr. Ghazi Al-Shumrani (intern).

Page 33: Hepatobiliary & pancreatic surgery

Hepatobiliary & pancreatic surgeryduct of Santorini, is variable. The lesser duct commonly drains the superior portion of the head of the pancreas. Itempties separately into the second portion of the duodenum through the lesser papilla located 2 cm proximal to theampulla of Vater. Pancreas divisum results from an incomplete fusion of the ventral pancreatic duct with the dorsal ductduring fetal development and is present in 5% of patients. In this anomaly, the lesser duct drains the entire pancreas viaan accessory ampulla located proximal to the ampulla of Vater. Inadequacy of this pattern of drainage can result inchronic pain.

56. Which of the following statements is/are correct with regard to the blood supply of the pancreas?

a. The inferior pancreaticoduodenal artery, a branch of the celiac artery, divides into anterior and posteriorbranches to supply the pancreatic head

b.The body and tail of the pancreas are supplied by branches of the splenic arteryc. The superior pancreaticoduodenal artery is a branch of the gastroduodenal arteryd.The body and tail of the pancreas are supplied by branches derived from the left renal artery

Answer: b, c

The pancreas receives its blood supply from a variety of major arterial sources. In the head of the pancreas, there arearcades in the anterior and posterior surfaces, which generally collateralize. These arcades arise from branches of thegastroduodenal and the superior mesenteric arteries. Just distal to the first portion of the duodenum, the gastroduodenalartery becomes the superior pancreaticoduodenal artery, which divides into anterior and posterior branches. The inferiorpancreaticoduodenal artery is the first branch of the superior mesenteric artery and divides into anterior and posteriorbranches.The body and tail of the pancreas are supplied by the splenic artery. The splenic artery arises from the celiac trunk andcourses along the superior surface of the pancreas to the spleen. Approximately ten branches of the splenic artery supplythe body and tail of the pancreas.

57. Orally administered glucose provokes a greater insulin response than an equivalent amount of intravenouslyadministered glucose. The incremental response to ingested glucose is due to the effects of which of thefollowing hormones?

a. Gastric inhibitory peptideb.Somatostatinc. Pancreatic polypeptided.Secretin

Answer: a

Orally administered glucose stimulates a greater insulin response than an equivalent amount of intravenous glucosethrough the release of enteric hormones that potentiate insulin secretion. This effect is known as the enteroinsular axis.Gastric inhibitory polypeptide (GIP) appears to be an important regulator of this effect, although other gut peptides,such as glucagon-like peptide I (GLP-1), may contribute to this effect as well. Nutrients that regulate insulin secretioninclude amino acids, such as arginine, lysine, and leucine, and free fatty acids. Hormones that stimulate insulinsecretion include glucagon, GIP, and cholecystokinin, whereas somatostatin, amylin, and pancreastatin are inhibitory.Insulin is also stimulated by sulfonylurea compounds, which act independently of the glucose concentration and formthe basis of treatment of type II, or insulin-independent, diabetes.

58. The islets of Langerhans contain four major endocrine cell types that secrete which of the followinghormones?

a. Insulin, somatostatin, glucagon, secretinb.Insulin, somatostatin, cholecystokinin, pancreatic polypeptidec. Insulin, somatostatin, glucagon, pancreatic polypeptided.Insulin, secretin, glucagon, cholecystokinin

Answer: c

Within the pancreas are small nests of cells that are responsible for the secretion of hormones that control glucosehomeostasis. These nests are called islets of Langerhans and constitute 2% of the pancreatic mass. The islets contain anaverage of 3000 cells and range in diameter from 40 to 900 mm. The islets are composed of four major cell types—alpha (A), beta (B), delta (D), and PP or F cells, which secrete glucagon, insulin, somatostatin, and pancreatic

Page 34: Hepatobiliary & pancreatic surgery

polypeptide, respectively. The B cells are centrally located within the islet and constitute 70% of the islet mass, whereasthe PP, A, and D cells are located at the periphery of the islet. They constitute roughly 15%, 10%, and 5% of the isletcell mass, respectively.Asir Surgery MCQs Bank. © 1422H-2002- first impression ©

This project was raised after an idia by Dr. Gharama Al-Shehri (consultant surgeon). Developed and typed by Dr. Ghazi Al-Shumrani (intern).

Page 35: Hepatobiliary & pancreatic surgery

Hepatobiliary & pancreatic surgery

59. A 50-year-old man develops acute pancreatitis due to alcohol abuse. Hyperamylasemia resolves by the thirdday after admission. By the eighth hospital day, the patient is noted to have recurrent fever (38.5°C),progressive leukocytosis (18,500 WBC/mm3), and tachypnea. The most appropriate management includeswhich as the next step?

a. Laparotomy with pancreatic debridementb.CT guided aspiration of peripancreatic fluid collectionsc. ERCP with sphincterotomy and placement of biliary stentd.Intravenous amphotericin B

Answer: b

The common causes of pancreatic abscesses are infected pancreatic pseudocysts and necrotizing pancreatitis. Thediagnosis is suggested by persistent fever, leukocytosis, and a palpable abdominal mass. Bacteremia and systemictoxicity are late clinical features. Percutaneous aspiration with positive cultures is the definitive preoperative test,facilitated by CT scanning or ultrasound-guidance to suspicious peripancreatic fluid collections. When diagnosed, thetreatment of choice is wide surgical débridement with removal of all infected and revitalized tissues. Generous drainageis mandatory. One of the major sources of morbidity and mortality in this situation is the late development of mycoticvisceral pseudoaneurysms, particularly involving the splenic circulation. These may be complex management problems,requiring angiographic embolization or other innovative treatment strategies.

60. The patient in the above question is treated by observation for 8 weeks. He continues to be symptomatic withepigastric pain. A repeat abdominal CT scan reveals a persistent 6 cm pseudocyst in the region of the body ofthe pancreas. The pseudocyst is unilocular and demonstrates a well-defined rim of fibrous tissue. The gastricantrum is displaced anteriorly. Using CT guidance, 300 ml of fluid is aspirated from the lesion which is shownto be collapsed radiographically. No further intervention is performed. What is the risk of pseudocystrecurrence after simple aspiration?

a. 80–85%b.60–65%c. 40–45%d.20–25%

Answer: d

Generally, a pancreatic pseudocyst can be observed for a period of weeks or months in an effort to allow forspontaneous resolution. Percutaneous ultrasound-or CT-directed aspiration or drainage catheter placement is an initialtreatment option. Simple aspiration is performed if the initial aspirate is sterile; if the aspirate is infected, a catheter oropen drainage procedure is appropriate. Determination of pancreatic ductal anatomy is important. Contrast injectioninto the pseudocyst at the time of aspiration should be considered to assess the possibility of pancreatic ductalcommunication and obstruction, or multiple cysts. The pseudocyst recurrence rate after simple aspiration is about 20%to 25%.

61. In prospective, randomized trials which of the following agents or therapeutic measures has/have beendemonstrated to accelerate recovery from acute pancreatitis?

a. Peritoneal lavageb.Anticholinergic blockadec. Octreotided.H2 receptor blockadee. None of the above

Answer: e

A variety of pharmacologic agents that directly or indirectly reduce acinar cell enzyme release or ductal secretion haveundergone clinical evaluation for the treatment of acute pancreatitis—generally with unimpressive results. Among thefirst were anticholinergic drugs. Despite extensive experience over many years, no objective data have emerged tosupport their use. Clinical trials of glucagon and calcitonin based on the same principle have produced a similar lack ofsupportive data. More recently, a somatostatin analog has been subjected to clinical trials for patients with acutepancreatitis. Somatostatin inhibits pancreatic enzyme and bicarbonate secretion by preventing the normal release of

Page 36: Hepatobiliary & pancreatic surgery

cholecystokinin, secretin, and other gut peptides. Despite the theoretical appeal, it has not been possible to demonstratethat somatostatin alters the natural history or prognosis of simple acute pancreatitis, although it does diminish pancreaticsecretion.Asir Surgery MCQs Bank. © 1422H-2002- first impression ©

This project was raised after an idia by Dr. Gharama Al-Shehri (consultant surgeon). Developed and typed by Dr. Ghazi Al-Shumrani (intern).

Page 37: Hepatobiliary & pancreatic surgery

Hepatobiliary & pancreatic surgeryPeritoneal lavage as a specific therapy for acute pancreatitis was proposed after experimental studies demonstratedimproved survival in animals with fulminant pancreatitis. The concept was appealing in that activated proteases andother vasoactive substances identifiable in peritoneal aspirates from patients with pancreatitis would be removed, ratherthan systemically absorbed. Unfortunately, clinical trials using this approach have produced disappointing results, andthe eventual overall mortality rate appears unchanged.

62. Which of the following medical procedures has/have been associated with an increased risk of post-procedureacute pancreatitis?

a. Common bile duct explorationb.Endoscopic retrograde cholangiopancreatographyc. Coronary bypass graftingd.Distal gastrectomy

Answer: a, b, c, d

Many surgical procedures in the upper abdomen are associated with postoperative pancreatitis. The incidence of acutepancreatitis after gastric resection ranges from 0.6% to 1.23%. After biliary tract surgery, particularly after common bileduct exploration itself, acute pancreatitis occurs with an incidence of 0.5% to 3%. Direct manipulation or retraction ofthe pancreas or pancreatic duct appears to be the most common cause. About 1% of patients develop acute pancreatitisafter endoscopic retrograde cholangiopancreatography (ERCP). This is a predictable event, and the risk can beminimized by limiting the pressure used for contrast injection of the pancreatic duct. Acute pancreatitis also occurs inpatients after coronary artery bypass surgery and a variety of other procedures remote from the pancreas. Althoughpancreatitis in this circumstance is thought to result from ischemia, hypotension is not always noted. The systemicconsequences of activation of the inflammatory system may contribute to changes in microvascular blood flow.

63. A 42-year-old male develops acute pancreatitis in the setting of acute alcohol abuse. One week after onset ofsymptoms, computed tomography of the abdomen reveals a pancreatic phlegmon and associated pseudocyst.Which of the following factors, if present, would decrease the likelihood of spontaneous resolution of thepseudocyst?

a. Size greater than 5 cmb.Diffuse calcification of the pancreatic glandc. Multilocularityd.Location in the pancreatic tail

Answer: a, b, c

Initial management of pancreatic pseudocysts is based on symptoms. If the patient is asymptomatic and the cyst is small(< 5.0 cm) it can be safely observed as many of these will resolve over a period of weeks. Concurrent chronic alcoholicpancreatitis (by history or as indicated by pancreatic calcification), pseudocyst size greater than 5 cm, the presence of amultilocular or debris-filled pseudocyst cavity, and chronicity (longer than 6 weeks) are all factors that are associatedwith a lower probability of spontaneous resolution.

64. Which of the following is/are prognostic signs reported by Ranson to predict outcomes associated with acutepancreatitis?

a. Age greater than 60 yearsb.Hematocrit decrease of 105 within 48 hours of hospital admissionc. Serum amylase value greater than 4 times upper limit of normald.Serum glucose greater than 200 mg/dL on admissione. Ca2+ level less than 8 mg/dL within 48 hours of hospital admission

Answer: b, d, e

Ranson prognostic signs include:

ON ADMISSIONAge above 55 yearsWhite blood cell count above 16,000/µLGlucose level above 200 mg/dL

Page 38: Hepatobiliary & pancreatic surgery

Lactase dehydrogenate level above 350 IU/LSGOT value above 250 IU/L

Asir Surgery MCQs Bank. © 1422H-2002- first impression ©

This project was raised after an idia by Dr. Gharama Al-Shehri (consultant surgeon). Developed and typed by Dr. Ghazi Al-Shumrani (intern).

Page 39: Hepatobiliary & pancreatic surgery

AFTER 48 HOURSHematocrit decrease of 10%Blood urea nitrogen level increase of 5 mg/dLCa2+ level below 8 mg/dLPaO2 level below 60 mmHgBase deficit value above 4 mEq/LFluid sequestration greater than 6 L

Hepatobiliary & pancreatic surgery

65. A 36-year-old woman is admitted to a the hospital with upper abdominal pain, hyperamylasemia, elevation ofserum alkaline phosphatase and ultrasound evidence of cholelithiasis. With intravenous hydration andanalgesia, symptoms rapidly resolved. After 48 hours, serum amylase and alkaline phosphatase values hadreturned to normal and physical examination revealed lessening tenderness in the right upper quadrant of theabdomen. Appropriate management consists of which of the following as the next step?

a. Cholecystectomy and intraoperative cholangiography before hospital dischargeb.Elective cholecystectomy at approximately 8 weeksc. Endoscopic sphincterotomy before discharge followed by cholecystectomy at approximately 8 weeksd.Observation

Answer: a

A patient who has simple cholelithiasis and an episode of acute pancreatitis is usually treated nonoperatively untilclinical resolution of the pancreatitis occurs. The rate of recurrent biliary pancreatitis is as high as 34% to 56% within 6weeks; therefore, an aggressive operative approach is appropriate. Cholecystectomy is often performed after theresolution of acute pancreatitis but before hospital discharge. Common bile duct instrumentation in this setting has asubstantially increased risk of recurrent acute pancreatitis.

66. For the patient in the preceding question, symptomatic recurrence at 3 weeks after aspiration is confirmedultrasonographically. Endoscopic retrograde pancreatography does not demonstrate communication of a majorpancreatic duct with the pseudocyst. Appropriate management includes which of the following?

a. Pancreatectomy to include the pseudocystb.Cystgastrostomyc. Repeat aspiration followed by injection of sodium morrhuate into the pseudocyst cavityd.Pancreatic debridement followed by external drainage

Answer: b

The operative treatment for pseudocysts depends on the underlying cause of the cyst, as well as the size, location, andmaturity of the pseudocyst wall. Whenever possible, the status of the pancreatic duct should be assessed preoperatively,preferably by ERCP. Operative drainage can be either external or internal. External drainage is chosen in the presenceof infection or an immature capsule. The disadvantages of external drainage include the risk of pancreatic fistulaformation and a pseudocyst recurrence. External drainage has been associated with a higher mortality rate, probablybecause it is used in patients at higher risk, especially those with sepsis, pancreatic abscesses, or ruptured pseudocysts.The type of internal drainage procedure selected depends on the location of the pseudocyst and whether or not there isassociated pancreatic ductal pathology. Cystogastrostomy is the simplest and safest alternative if the pseudocyst isappropriately adjacent to the posterior wall of the stomach. Cystojejunostomy using a Roux-en-Y or loop jejunostomymay also be appropriate, depending on the location and specific anatomy of the pseudocyst. Pancreatic resection isassociated with the lowest recurrence rate (3%), but is limited to pseudocysts occurring in the tail of the pancreas.

67. With regard to acute pancreatitis: which of the following statements is/are correct?

a. The majority of patients presenting with acute pancreatitis of biliary type are femaleb.The majority of patients presenting with acute pancreatitis of alcoholic type are femalec. The most common cause of acute pancreatitis in the United States is alcohol used.Patients with alcohol-induced pancreatitis tend to be older than those with biliary-induced disease

Answer: a, c

In autopsy series, the evidence for past acute pancreatitis averages 0.31%. Variations among populations are highlydependent on social factors such as ethanol use and on environmental and hereditary determinants such as the incidence

Page 40: Hepatobiliary & pancreatic surgery

of gallstones. Acute pancreatitis may occur at any age but is most common in adults between 30 and 70 years of age. Ingeneral, patients with gallstone-induced pancreatitis are older (age 40 to 60 years), whereas those with alcohol-associated pancreatitis are younger (age 30 to 40 years). The sex distribution of acute pancreatitis depends on the

Asir Surgery MCQs Bank. © 1422H-2002- first impression ©

This project was raised after an idia by Dr. Gharama Al-Shehri (consultant surgeon). Developed and typed by Dr. Ghazi Al-Shumrani (intern).

Page 41: Hepatobiliary & pancreatic surgery

Hepatobiliary & pancreatic surgeryclinical cause of the disease, with women representing 68% of patients with gallstone-associated pancreatitis.Conversely, when alcohol is the primary association, men account for most patients.Clinical associations with acute pancreatitis can be divided into three broad categories-biliary stones, ethanol, andothers. Biliary tract stone disease and ethanol-induced pancreatitis account for most cases of acute pancreatitis reportedworldwide. The particular distribution of causes reflects the source of the patient population evaluated. In a summary of18 different reports of acute pancreatitis in the United States with a combined total of 7147 patients, 53% of patientswere believed to have ethanol-induced disease, whereas 28% had proven biliary stones. In contrast, of 1539 patientsreported on from Great Britain, 52% had gallstones, 7% were ethanol related, and 34% had no identifiable cause.

68. Which of the following statement(s) relating to chronic pancreatitis is/are correct?

a. In the United States, the most common cause of chronic pancreatitis is alcohol abuseb.Approximately 50% of chronic alcoholics develop chronic pancreatitisc. Clinically significant chronic pancreatitis develops on average after five years of alcohol abuse in mend.The risk of alcohol-induced chronic pancreatitis can be decreased by consumption of a high-protein diet

Answer: a

In the United States, alcohol consumption is the major cause of chronic pancreatitis: with approximately 70% of casesattributable to this factor. Most patients with symptomatic chronic pancreatitis have consumed large volumes of alcoholdaily for a prolonged period of time. The average daily intake of alcohol is 150 to 175 g with the mean duration ofalcoholism before recognition of pancreatitis being 18 years for men and 11 years for women. The incidence of chronicpancreatitis on autopsy studies of chronic alcoholics is 50 times the rate of non-drinking controls. Only 10% ofalcoholics develop chronic pancreatitis—suggesting that factors other than long-term alcohol exposure may alsoinfluence susceptibility. In both experimental and clinical studies, the risk of alcohol-induced chronic pancreatitis isincreased by a high-protein, high-fat diet.

69. The most appropriate test to confirm a clinical diagnosis of early chronic pancreatitis is which of thefollowing?

a. Serum amylase determinationb.Calculation of urinary amylase clearancec. Measurement of para-aminobenzoic acid absorptiond.Endoscopic retrograde cholangiopancreatography

Answer: d

Routine tests of blood or serum are not helpful in making a diagnosis of chronic pancreatitis. Although serum amylaselevels are almost always elevated in acute pancreatitis—amylase levels may be normal, elevated, or subnormal inchronic pancreatitis. Determination of urinary amylase secretion and calculation of urinary amylase clearance does notimprove sensitivity or specificity. Indirect tests of pancreatic function which measure absorption of nutrients that firstrequire pancreatic digestion are not helpful in early cases of chronic pancreatitis. Clinically detectable malabsorption isabsent until 90% of exocrine function is lost. Because of this, indirect tests of pancreatic function do not detect earlydisease. In addition, false positive tests may occur in other disease states associated with malabsorption (Crohn’sdisease, sprue, postgastrectomy states, or in association with diabetes mellitus, cirrhosis, or renal disease. ERCP hasbecome widely recognized as the most sensitive and reliable method for diagnosing chronic pancreatitis. Sensitivityapproaches 90% with equal specificity.

70. A 52-year-old male, known to be alcoholic, is evaluated because of chronic abdominal pain. The clinicaldiagnosis of chronic pancreatitis is supported by ERCP findings of pancreatic ductal ectasia with alternatingareas of stricture and dilatation. Several pancreatic ductal stones are also noted. With chronic pain as theoperative indication, the most appropriate procedure would be:

a. 80% distal pancreatectomy with splenectomyb.Longitudinal pancreaticojejunostomyc. Distal pancreatectomy with end pancreaticojejunostomyd.Total pancreatectomy

Answer: b

When patients with chronic pancreatitis have pancreatic ductal dilatation (greater than 8 mm. ductal decompression

Page 42: Hepatobiliary & pancreatic surgery

using longitudinal pancreaticojejunostomy may be employed for relief of pain. The finding that pancreatic ductalhypertension exists in patients with painful chronic pancreatitis and that surgical decompression reduces intrapancreaticpressure to normal provides the rationale for this operation. The anterior surface of the pancreas is exposed through theAsir Surgery MCQs Bank. © 1422H-2002- first impression ©

This project was raised after an idia by Dr. Gharama Al-Shehri (consultant surgeon). Developed and typed by Dr. Ghazi Al-Shumrani (intern).

Page 43: Hepatobiliary & pancreatic surgery

Hepatobiliary & pancreatic surgerylesser sac. The entire pancreatic duct is opened from the pancreatic tail to a point 1 cm from the duodenum. A side-to-side anastomosis is then performed between the opened pancreatic duct and a loop of jejunum. Splenectomy is notnecessary. In-hospital mortality rates of less than 5% have been widely reported. Approximately 80% of patients reportcomplete or substantial improvement of pain following longitudinal pancreaticojejunostomy.

71. For the patient in the preceding question, the most appropriate long-term management is which of thefollowing?

a. Endoscopic stenting of the distal common bile ductb.Choledochoduodenostomyc. Pancreaticoduodenectomy (Whipple procedure)d.Percutaneous transhepatic drainage of the common hepatic duct

Answer: b

Operative management of patients with stricture of the common bile duct associated with chronic pancreatitis isjustified to treat symptoms and to prevent development of biliary cirrhosis. Operative indications include progressivejaundice, cholangitis, liver biopsy evidence of biliary cirrhosis, persistent elevation of alkaline phosphatase at greaterthan three times normal, and progressive stricture demonstrated by radiologically progressive dilatation of extrahepaticand intrahepatic biliary ducts. Both choledochoduodenostomy and choledochojejunostomy are excellent operativechoices.

72. Which of the following is the most common cause of obstructive jaundice in patients with chronic pancreatitis?

a. Adenocarcinoma of the head of the pancreasb.Choledocholithiasisc. Fibrotic stricture of the common bile ductd.Pancreatic pseudocyst formation

Answer: c

Biliary complications involving the common bile duct can occur in chronic pancreatitis because of the intimateassociation of that structure with the head of the pancreas. In two-thirds of individuals, the common bile duct traversesthe pancreatic parenchyma and in an additional 25%, the common bile duct lies within a groove along the posteriorsurface of the pancreas. Fibrosis associated with chronic pancreatitis can encase and compress the common bile duct.Common bile duct stenosis is relatively common in chronic pancreatitis, occurring in approximately 10% of casesobserved long-term. Cholangiography typically reveals a long, gradually tapering stricture conforming to theintrapancreatic portion of the common bile duct. In contrast, malignant strictures usually result in abrupt termination ofthe biliary duct. The proximal suprapancreatic portion of the bile duct is variably dilated.

73. Alcohol-induced and hereditary chronic pancreatitis are the two most common etiologies observed in NorthAmerican patients. Most of the remaining patients fall into which of the following categories?

a. Chronic pancreatitis secondary to hyperparathyroidismb.Chronic pancreatitis caused by protein-calorie malnutritionc. Chronic pancreatitis secondary to congenital pancreatic ductal obstructiond.Idiopathic chronic pancreatitis

Answer: d

After alcohol-induced and hereditary disease, idiopathic chronic pancreatitis is the most common cause of calcifyingpancreatitis in North American patients. This designation is given to those cases without a recognizable cause.Idiopathic pancreatitis accounts for about 15% of the cases and has two peaks in incidence, suggesting that differingunderlying causes may exist. The first peak occurs in young adulthood and the second has an occurrence atapproximately 60 years of age.

74. Which of the following statements regarding prognosis in chronic pancreatitis is/are correct?

a. Patients with chronic pancreatitis have decreased long-term survival compared with the general populationb.Patients with chronic pancreatitis exhibit no excess mortality relative to the general population

Page 44: Hepatobiliary & pancreatic surgery

c. Excess mortality in patients with chronic pancreatitis is related to cancers of the aerodigestive system,complications of diabetes, and complications of cirrhosis

Asir Surgery MCQs Bank. © 1422H-2002- first impression ©

This project was raised after an idia by Dr. Gharama Al-Shehri (consultant surgeon). Developed and typed by Dr. Ghazi Al-Shumrani (intern).

Page 45: Hepatobiliary & pancreatic surgery

Hepatobiliary & pancreatic surgeryd.Excess mortality in patients with chronic pancreatitis is due to development of adenocarcinoma of the pancreas

and to the complications of recurrent pancreatitisAnswer: a, c

Patients with chronic pancreatitis have decreased long-term survival compared to the general population. An excess ofmortality of 30% over 20 years has been estimated. Less than 20% of deaths are directly attributable to pancreatitis orits complications. Excess mortality is related to extrapancreatic complications of alcoholism and smoking, includingcancers of the aerodigestive system, complications of diabetes, and complications of cirrhosis.

75. Which of the following is the most common clinical manifestation of chronic pancreatitis?

a. Epigastric pain with radiation to the hypogastriumb.Diabetes mellitusc. Steatorrhead.Epigastric pain with radiation to the upper lumbar vertebrae

Answer: d

Pain is a predominant symptom complex in most patients with chronic pancreatitis. Chronic pancreatic pain is usuallylocalized to the epigastrium with radiation to the back in the region of the upper lumbar vertebrae. Discomfort may beexacerbated by eating and is usually alleviated by abstinence from food and by bending forward. Malabsorption andweight loss, clinical manifestations of steatorrhea, are only observed when greater than 90% of exocrine tissue has beendestroyed. Clinical signs of malabsorption are a late manifestation of chronic pancreatitis. Although abnormal glucosetests can be demonstrated in 50% to 70% of patients with chronic Pancreatitis: overt diabetes mellitus is present in only30% to 40%. Endocrine deficits are usually progressive. If individual patients are repetitively tested, progressivedeterioration is often observed.

76. For the patient in the preceding question, appropriate management includes which of the following?

a. Distal pancreatectomyb.Cystjejunostomyc. Percutaneous drainaged.Primary radiotherapy and chemotherapy

Answer: a

The proper treatment is surgical removal of the tumor; aggressive pancreatic resection is appropriate. It is crucial toavoid mistaking a mucinous cystic tumor for a pancreatic pseudocyst. Internal drainage of a malignant mucinous cystictumor results in catastrophic tumor dissemination and should never be performed. With appropriate treatment, allpatients with histologically benign tumors should be cured; for tumors demonstrating malignant change, 5-year survivalafter surgery is about 60%.

77. A 72-year-old man develops jaundice and is demonstrated to have a 2.5 mass in the pancreatic head bycomputed tomography. There are no signs of unresectability on CT examination. Fine needle aspirationcytology is positive for adenocarcinoma. Which of the following intraoperative findings would indicateunresectability?

a. Fibrotic reaction in the body and tail of the pancreasb.Microscopic tumor cells in perigastric lymph nodes on frozen sectionc. Inability to develop an avascular plane anterior to the superior mesenteric veind.Cholelithiasis

Answer: b, c

During performance of pancreaticoduodenectomy, the lesser sac is opened widely through the gastrocolic omentum.This maneuver allows inspection of the body and tail of the gland to determine the extent of the tumor involvement andallows examination of lymph nodes along the superior and inferior body of the pancreas and around the celiac axis.Enlarged nodes in these areas should undergo biopsy and be submitted for frozen-section examination, since tumor inthese areas is beyond the bounds of standard pancreaticoduodenectomy and constitutes a contraindication to resection.If there is no evidence of lymphadenopathy, a dissection between the anterior surface of the portal vein and theposterior surface of the neck of the pancreas is performed. Ordinarily, only thin areolar tissue lies between the pancreas

Page 46: Hepatobiliary & pancreatic surgery

and the portal vein, and a communication behind the neck of the pancreas can be established. If there is hard tissueintervening and such communication cannot be established, this implies invasion of the anterior surface of the portalvein and signals unresectability by standard methods.Asir Surgery MCQs Bank. © 1422H-2002- first impression ©

This project was raised after an idia by Dr. Gharama Al-Shehri (consultant surgeon). Developed and typed by Dr. Ghazi Al-Shumrani (intern).

Page 47: Hepatobiliary & pancreatic surgery

Hepatobiliary & pancreatic surgery

78. A 67-year-old male presents with complaints of itching, dark urine, and epigastric pain. Physical examinationreveals jaundice. Initial laboratory tests show total bilirubin of 6.5 mg/dL, alkaline phosphatase elevated at 3the upper limit of normal, and mild elevations in serum transaminases. Appropriate management includeswhich diagnostic test next?

a. Abdominal ultrasonographyb.Computed tomography of the abdomenc. Magnetic resonance imaging of the abdomend.Endoscopic retrograde cholangiography

Answer: a

Standard transcutaneous ultrasonography is the appropriate first test in the evaluation of the patient with jaundice,because the presence of a dilated common bile duct or intrahepatic bile ducts is essentially diagnostic of extrahepaticbiliary obstruction. This finding directs the physician to a search for the cause of the obstruction. If the bile ducts are notdilated, mechanical obstruction is unlikely and the diagnostic thrust should move toward hepatocellular disease.Ultrasonography is also the best test to determine whether gallstones are present; this is extremely important becausecholedocholithiasis is one of the conditions most likely to cause jaundice in the elderly population.

79. Which of the following statements regarding ductal adenocarcinoma of the pancreas is/are correct?

a. For ductal adenocarcinoma, 60–70% of tumors arise in the head of the gland, 15% in the body, and 10% in thetail, the remainder are diffuse

b.Fifty percent of pancreatic adenocarcinomas involve the gland diffusely at the time of diagnosisc. For ductal adenocarcinomas, tumors of the body and tail are usually larger at the time of diagnosis than those

arising in the head of the glandd.Pancreatic adenocarcinomas occur with equal frequency within the head, body, and tail of the gland

Answer: a, c

Sixty to 70% of pancreatic ductal adenocarcinomas occur in the head of the gland. About 15% reside in the body of thegland, another 10% are in the tail, and the remaining 5% to 15% are diffuse. The predilection of pancreatic cancer todevelop in the head of the gland is unexplained, but has the practical consequence that tumors in the head are diagnosedearlier because they cause obstructive jaundice whereas tumors in the body and tail tend to be more advanced at thetime of symptomatic presentation. Tumors in the body and tail are typically larger at the time of diagnosis (average, 7 to8 cm) than in the head (average, 4 to 5 cm).

80. The most common cause of death in the postoperative period following pancreaticoduodenectomy is which ofthe following?

a. Myocardial infarctionb.Intraperitoneal hemorrhagec. Pulmonary embolismd.Pneumonia

Answer: b

Pancreaticoduodenectomy is a formidable operation, and until recently, average operative mortality was reported toapproximate 20%. In the past few years, several centers have reported large series with operative mortalities lower than5%.The most dreaded complication of pancreaticoduodenectomy is disruption of the pancreaticojejunostomy, which occursin about 10% of patients. Anastomotic breakdown may lead to the development of an upper abdominal abscess or maypresent as a external pancreatic fistula. In its most virulent form, disruption leads to necrotizing retroperitoneal infectionwhich may erode major arteries and veins of the upper abdomen, including the portal vein or its branches or the stumpof the gastroduodenal artery. Impending catastrophe is often preceded by a small herald bleed from the drain site. Suchan event is an indication to return to the operating room to widely drain the pancreaticojejunostomy and to repair theinvolved blood vessel. Open packing of the wound may be necessary in controlling diffuse necrosis and infection. Onrare occasions, completion pancreatectomy is required to control sepsis. Intraperitoneal hemorrhage is the mostcommon cause of death from pancreaticoduodenectomy.

Page 48: Hepatobiliary & pancreatic surgery

Asir Surgery MCQs Bank. © 1422H-2002- first impression ©

This project was raised after an idia by Dr. Gharama Al-Shehri (consultant surgeon). Developed and typed by Dr. Ghazi Al-Shumrani (intern).

Page 49: Hepatobiliary & pancreatic surgery

Hepatobiliary & pancreatic surgery81. For the above patient, ultrasonography reveals dilated extrahepatic and intrahepatic bile ducts and the absence

of gallstones within the gallbladder. A 2 cm mass within the pancreatic head is visualized. Computedtomography of the abdomen is performed. Which of following CT findings indicate probable unresectability?

a. Common bile duct diameter of 2 cmb.Ascitesc. Infiltration of the transverse colonic mesenteryd.Dilatation of the main pancreatic duct to 1.5 cm

Answer: b, c

CT scans provide the best available radiologic information to determine whether or not a pancreatic neoplasm isresectable, but they cannot be considered absolutely definitive in this regard. Only about half of pancreatic tumors thatappear to be confined to the pancreas on CT scan are found to be resectable in the operating room. CT scanning is moreaccurate in the diagnosis of unresectability. CT findings that indicate that the tumor is unlikely to be surgically curableinclude vascular invasion, enlarged lymph nodes outside the boundaries of resection, ascites, distant metastases (usuallyliver), and distant organ invasion (usually colon). When a CT scan shows distant metastases or extensive local invasion,the positive predictive value of the technique is high; some 90% of such patients have unresectable disease atlaparotomy.

82. Which of the following have been shown to be risk factors for development of adenocarcinoma of thepancreas?

a. Cigarette smokingb.Coffee drinkingc. Adult-onset diabetes mellitusd.Chronic coumadin usagee. Prior gastrectomy

Answer: a, e

Most cases of pancreatic cancer have no obvious predisposing host factors. The most consistently observed risk factorfor pancreatic cancer is cigarette smoking. Most studies estimate that smoking results in a two-to three-fold increase inrisk of developing pancreatic cancer. Alcohol consumption has been implicated in some case-control studies ofpancreatic cancer, but the overall evidence is inconsistent and alcohol is not likely to be a major factor in thedevelopment of the disease. Although considerable public interest was focused in the past few years on coffeeconsumption as a risk factor for pancreatic cancer, evidence linking coffee consumption to pancreatic cancer is notcompelling.Abnormal glucose tolerance is present in about 80% of patients with pancreatic cancer, if carefully sought. Althoughdiabetes and pancreatic cancer occur together far more frequently than would be expected by chance, persons with long-standing diabetes are not at increased risk of developing pancreatic cancer. Chronic pancreatitis is a significant riskfactor for the development of pancreatic cancer. It appears that all forms of chronic pancreatitis are associated with anincreased risk of pancreatic cancer, suggesting that it is the pancreatitis and not the injuring agent which is responsiblefor the augmented cancer risk. Studies indicate that patients who have previously undergone gastric resection may befrom three to seven times more likely to develop pancreatic cancer than a control population.There have been several reports of familial clustering of pancreatic cancer. Recent epidemiological studies suggest thatabout 7% of pancreatic cancer patients have a positive family history of the disease. For most cases, however, nohereditary basis for pancreatic cancer has been identified.

83. Which of the following surgical procedures has the lowest incidence of recurrent jaundice when used in thecontext of unresectable carcinoma of the head of the pancreas?

a. Choledochoduodenostomyb.Cholecystojejunostomyc. Cholecystoduodenostomyd.Choledochojejunostomy

Answer: d

When jaundiced patients undergo exploration in the hopes of resection but unresectable disease is found, biliary bypassshould be performed. The jejunum is typically chosen as a conduit in preference to the duodenum because duodenalobstruction may occur as the tumor becomes more advanced. There has been much discussion over the use of the bile

Page 50: Hepatobiliary & pancreatic surgery

duct or the gallbladder for biliary decompression. Operative mortality and mean survival (about 6 months) do not differbetween patients with cholecystojejunostomy and choledochojejunostomy. Recurrent jaundice is more common aftercholecystojejunostomy. Because recurrent jaundice constitutes a failure of palliation, the use of the common duct for

Asir Surgery MCQs Bank. © 1422H-2002- first impression ©

This project was raised after an idia by Dr. Gharama Al-Shehri (consultant surgeon). Developed and typed by Dr. Ghazi Al-Shumrani (intern).

Page 51: Hepatobiliary & pancreatic surgery

Hepatobiliary & pancreatic surgerybiliary bypass is preferable in most patients. There are circumstances, however, in which it may be more appropriate touse the gallbladder. Such instances include patients with poor performance status, cases in which the tumor is bulky andinvades the porta hepatis, or when periductal varices have developed as a result of portal vein thrombosis. Thesuitability of the gallbladder as a biliary conduit must be proven intraoperatively. If, on aspiration, the gallbladdercontains colorless fluid, the cystic duct may be assumed to be obstructed, and the gallbladder should be removed andnot used for bypass. If there is green bile in the gallbladder, patency of the cystic duct should be proved bycholangiography before a bypass is performed.

84. A 45-year-old woman is evaluated for epigastric and back pain. Physical examination is normal. Computedtomography of the abdomen reveals a 8 cm cystic lesion in the region of the tail of the pancreas. The cystdemonstrates internal septations and papillary projections from its walls. Which of the following diagnoses ismost likely in this patient?

a. Pancreatic lymphomab.Retroperitoneal liposarcomac. Pancreatic pseudocystd.Pancreatic mucinous cystadenoma

Answer: d

Mucinous cystic neoplasms account for about 2% of pancreatic exocrine tumors. Most patients with mucinous cystictumors present with abdominal pain or an abdominal mass. There may be associated weight loss, steatorrhea, ordiabetes. The diagnosis is best made by CT scanning and ultrasonography, which demonstrate a mass containing fluid-filled structures and internal septations. Occasionally, it is possible to see the papillary tumor excrescences on the cystwalls.The tumor occurs six times as often in females as in males. About 80% of the tumors are located in the body and tail ofthe pancreas. They present as large (average, 10 cm), soft, and somewhat irregular tumors. Microscopically, the cystsare lined by columnar epithelium which contains mucin. Although most of the cells may appear benign histologically,most tumors larger than 3 cm contain areas of premalignant or malignant change and all mucinous cystic tumors shouldbe considered to have malignant potential.

85. A 45-year-old woman develops upper gastrointestinal hemorrhage. Evaluation by upper endoscopy revealsthree ulcers in the second portion of the duodenum. Bleeding is controlled using an endoscopic heat probe.Further investigation reveals a serum gastrin value of 240 pg/mL. Which of the following would support thepresumptive diagnosis of gastrinoma?

a. An increase of 320 pg/mL in serum gastrin upon intravenous infusion of secretinb.Gastric acid analysis demonstrating fasting acid secretion of 3 mEq/hc. Enlarged gastric rugae on upper gastrointestinal contrast studyd.An increase of 150 pg/mL in serum gastrin upon intravenous infusion of cholecystokinin

Answer: a, c

The indications for the measurement of gastrin include the presence of peptic ulcer disease, patients with prolongedundiagnosed diarrhea, patients within MEN-1 families and patients with prominent gastric rugal folds on upper GIseries. In most patients with gastrinoma, the fasting serum gastrin level is elevated above 200 pg/ml. Gastrin values over1000 pg/ml are virtually diagnostic of gastrinoma. However, fasting hypergastrinemia alone is not sufficient for thediagnosis of gastrinoma. Gastric acid analysis is an important test in the evaluation of patients with suspectedgastrinoma, as it can differentiate between ulcerogenic causes of hypergastrinemia and nonulcerogenic causes ofhypergastrinemia. The diagnosis of gastrinoma is supported by a basal acid output above 15 mEq/hour in nonoperatedpatients.Following documentation that hypergastrinemia is associated with excessive acid secretion, provocative testing usingsecretin should be performed to differentiate between gastrinoma, antral G cell hyperplasia/hyperfunction, and the othercauses of ulcerogenic hypergastrinemia. The secretin stimulation test is carried out in the fasting state by obtainingperipheral serum samples for gastrin in the basal period, administering secretin (2 units/kg body weight) as anintravenous bolus, and obtaining serum samples for gastrin at five minute intervals for 30 minutes. An increase in thegastrin level of more than 200 pg/ml above the basal level is supportive of the diagnosis of gastrinoma.

86. For the patient in the preceding question, an insulin/glucose ratio of 0.5 was documented at 28 hours of fasting.Symptoms of mental obtundation developed concurrently and were reversed by oral glucose administration.

Page 52: Hepatobiliary & pancreatic surgery

Endoscopic ultrasonography demonstrated a 1.2 cm mass in the head of the pancreas. Appropriatemanagement consists of which of the following?

Asir Surgery MCQs Bank. © 1422H-2002- first impression ©

This project was raised after an idia by Dr. Gharama Al-Shehri (consultant surgeon). Developed and typed by Dr. Ghazi Al-Shumrani (intern).

Page 53: Hepatobiliary & pancreatic surgery

a. Surgical enucleation of the tumorb.Total pancreatectomyc. Long-term octreotide administrationd.Primary radiotherapy

Answer: a

Hepatobiliary & pancreatic surgery

The treatment of insulinoma is surgical in nearly all cases. Insulinomas are found evenly distributed within thepancreas, with approximately one-third being located in the head and uncinate process, one-third in the body of thegland, and one-third in the tail of the gland. Ninety percent of patients will be found to have benign solitary adenomasamenable to surgical cure. Small benign insulinomas not in close proximity to the main pancreatic duct may be removedby enucleation, independent of their location within the gland. In the body and tail of the Pancreas: insulinomas greaterthan 2 cm in diameter, and those in close proximity to the pancreatic duct are most commonly excised by distalpancreatectomy. Large insulinomas deep in the head or uncinate process of the pancreas may not be amenable to localexcision, and may require pancreaticoduodenectomy.

87. A 35-year-old woman is evaluated for seizure disorder, mental obtundation, and personality change. Physicalexamination is normal. Fasting serum glucose is 44 mg/dL. Other serum values are normal. Subsequentinvestigations should include which of the following?

a. Oral glucose tolerance testb.Determination of fasting insulin/glucose ratiosc. Assay of serum C-peptide levelsd.Determination of serum prolactin levels

Answer: b, c

A common mistake made in the evaluation of a patient with suspected insulinoma is to commence the evaluation withan oral glucose tolerance test. Instead, insulinoma is most reliably diagnosed using the technique of a monitored fast.During a monitored fast, blood for glucose and insulin determinations is sampled every four to six hours, and at the timeof symptom occurrence. Hypoglycemic symptoms typically occur when glucose levels are less than 50 mg/dl, withconcurrent serum insulin levels often being greater than 25 µU/ml. Additional support for the diagnosis of insulinomacomes from the calculation of the insulin to glucose ratio (I:G ratio) at different time points during the monitored fast.Normal individuals will have I:G ratios less than 0.3, while patients with insulinoma typically demonstrate I:G ratiosgreater than 0.4 after a prolonged fast. Other measurable beta cell products synthesized in excess in patients withinsulinoma include C peptide and proinsulin. Elevated levels of C peptide and proinsulin are typically found in theperipheral blood in patients with insulinoma. The possibility of surreptitious insulin or oral hypoglycemic agentadministration should be considered in all patients with suspected insulinoma. C peptide and proinsulin levels will notbe elevated in patients self-administering insulin. Additionally, patients self-administering either bovine or porcineinsulin may demonstrate anti-insulin antibodies in circulating blood.

88. The most common location(s) for development of gastrinoma is/are which of the following?

a. Pancreas to the right of the superior mesenteric veinb.Pancreatic body and tailc. Gastric antrumd.Duodenum

Answer: a, d

The majority of gastrinomas have been identified to the right of the superior mesenteric vessels within the head of thepancreas or the duodenum. Intraoperative ultrasonography should be available to assist in tumor localization. Inaddition, intraoperative upper endoscopy may be helpful by allowing transillumination of the duodenal wall andidentification of small duodenal gastrinomas. At exploration, any suspicious peripancreatic lymph nodes are excisedand submitted for frozen section. Primary tumors located within the substance of the pancreas that are small (< 2 cm)and well encapsulated may be carefully enucleated. Pancreatic tumors without defined capsules or situated deep in thepancreatic parenchyma may require partial pancreatic resection. In the absence of an identifiable pancreatic or duodenaltumor, a longitudinal duodenotomy may be performed at the level of the second portion of the duodenum to allow foreversion of the duodenum in a search for duodenal microgastrinomas. Primary gastrinomas identified within theduodenal wall are resected locally, with primary closure of the duodenal defect.

Page 54: Hepatobiliary & pancreatic surgery

89. Neoplastic hypersecretion of the hormone vasoactive intestinal peptide is associated with which of thefollowing features?

Asir Surgery MCQs Bank. © 1422H-2002- first impression ©

This project was raised after an idia by Dr. Gharama Al-Shehri (consultant surgeon). Developed and typed by Dr. Ghazi Al-Shumrani (intern).

Page 55: Hepatobiliary & pancreatic surgery

a. Hypokalemia, hypochlorhydria, diarrheab.Hyperglycemia, necrolytic rash, hypoaminoacidemiac. Constipation, gallstones, hyperglycemiad.Hyperkalemia, necrolytic rash, diarrhea

Answer: a

Hepatobiliary & pancreatic surgery

Patients characteristically present with intermittent severe diarrhea, typically of a watery nature, averaging 5 liters/day.Malabsorption and steatorrhea are not common. Hypokalemia results from the fecal loss of large amounts of potassium(up to 400 meq/day), and low serum potassium levels may be associated with muscular weakness, lethargy, and nausea.Most patients are hypochlorhydric or achlorhydric. Half of the patients have some degree of hyperglycemia andhypercalcemia, while cutaneous flushing can be observed in a minority of patients. The diagnosis of VIPoma istypically made after excluding other more common causes of diarrhea. The active agent in the VIPoma syndrome isusually vasoactive intestinal polypeptide (VIP), with a minority of patients having elevations of other candidatemediators such as peptide histidine-isoleucine (PHI) or prostaglandins.

90. A patient with biochemically confirmed gastrinoma undergoes computed tomography for tumor localization.CT reveals a 2 cm mass in the head of the pancreas and multiple nodules within right and left lobes of the liver.Appropriate management includes which of the following?

a. Omeprazole administrationb.Radiotherapyc. Pancreaticoduodenectomyd.Proximal gastric vagotomy

Answer: a

Gastrinoma patients whose localization and staging studies are indicative of unresectable hepatic metastases shouldundergo percutaneous or laparoscopically-directed liver biopsy for histologic verification. If unresectable gastrinoma isconfirmed, then open surgical exploration is not performed and the patient is maintained on long-term omeprazoletherapy. Virtually all patients can be rendered achlorhydric with appropriate dose adjustment of omeprazole.Noncompliant patients who refuse to take appropriate doses of omeprazole and who develop complications related totheir ulcer diathesis may require total gastrectomy for management. Total gastrectomy removes the end organ (parietalcell mass) and was once the procedure of choice for gastrinoma. Today its use in gastrinoma patients has markedlydeclined.

91. The following statement(s) is/are true concerning the widely accepted French or Couinaud’s nomenclature forliver anatomy.

a. The liver is divided into eight discrete segments based on portal pedicle branches and hepatic venous drainageb.This anatomy is particularly useful in allowing less than lobar segmental anatomical resections that minimize

blood loss and loss of hepatic reservec. Enumeration of the system begins from right to leftd.Segments II and III are synonymous with the left lateral segment based on English nomenclature

Answer: a, b, d

In the now widely accepted French (Couinaud’s) nomenclature, the liver can be divided into eight discrete segmentsbased on portal pedicle branches and hepatic venous drainage. Enumeration of the segments begins left to right,beginning with segment I, the caudate lobe. The left lateral sector consists of a superior segment II and an inferiorsegment III and is synonymous with the left lateral segment in older terminology. The major advantage to this detailedsegmental anatomy, which is based on discrete portal pedicle branches, is to accurately locate individual lesions in thehepatic substance by preoperative imaging and intraoperative ultrasound and to allow the possibility of less than lobarsegmental anatomical resections that minimize blood loss and functional loss of hepatic reserve.

92. In the patient described above, which of the following are important operative steps in the performance of aright hepatic lobectomy?

a. The use of an ultrasonic dissector is essential for division of the hepatic parenchymab.If temporary portal inflow occlusion is used (Pringle maneuver), it is not necessary to reestablish blood flow

during the course of the parenchymal division

Page 56: Hepatobiliary & pancreatic surgery

c. The greater omentum may be used to buttress the transected liver edged.Control of the main right hepatic vein should eliminate all forms of venous drainage

Answer: cAsir Surgery MCQs Bank. © 1422H-2002- first impression ©

This project was raised after an idia by Dr. Gharama Al-Shehri (consultant surgeon). Developed and typed by Dr. Ghazi Al-Shumrani (intern).

Page 57: Hepatobiliary & pancreatic surgery

Hepatobiliary & pancreatic surgery

The steps involved in a right hepatic lobectomy involve adherence to the tenet of optimal operative exposure andcontrol of vascular inflow and outflow. In select circumstances, control of the vena cava may be desired. Either theindividual portal structures can be identified and ligated early in the course of the procedure, or simply the entire portaltriad can be circled with an umbilical tape tourniquet in preparation for the Pringle maneuver. If temporary portal inflowocclusion is used, intermittent 10 to 20 minute intervals of clamping with 3 to 5 minutes to reestablish blood flow isrecommended. The division of the hepatic parenchyma begins with scoring of Glisson’s capsule with cautery or knifeand proceeds with division of the hepatic surface using either blunt dissection by finger fracture, the blunt edge of aninstrument or suction tip, or using an ultrasonic dissector. Individual vessels and bile ducts are cauterized, sutured, orclipped in rapid succession from anterior to posterior. The hepatic veins are encountered in the hepatic substance nearthe vena cava and are carefully clamped and suture ligated to complete the resection. In addition, there are also severalposterior accessory veins (up to 10 in number) which drain the medial aspect of the right lobe and empty directly intothe right anterior surface of the IVC.

93. Intraoperative ultrasound is now commonly used by the hepatic surgeon. Which of the following statement(s)is/are true concerning intraoperative ultrasound and hepatic surgery?

a. An intraoperative ultrasound offers no advantage to conventional transcorporial ultrasound in detection ofhepatic lesions

b.Portal structures can be differentiated from hepatic veins by the extension of Glisson’s capsule surroundingthese structures

c. It is difficult on ultrasound to differentiate a vascular structure from a massd.The short hepatic veins are difficult to detect with intraoperative ultrasound

Answer: b

Over the past 10 years, detailed anatomic description of the hepatic veins, portal pedicles, and the inferior vena cavahave been possible through the use of intraoperative ultrasound. Cooperation between radiologists and hepatic surgeonwith the use of intraoperative ultrasound has allowed the identification of lesions during surgery that were not visible byconventional transcorporial ultrasound or CT scanning. Beginning superiorly at the inferior vena cava, the confluenceand course of each of the hepatic veins can easily be determined. More inferiorly, the main right and left portal pediclescan be seen coursing transversely in the transverse scissura. Portal structures can easily be differentiated from hepaticveins by the hyperechoic extensions of Glisson’s capsule which surround these structures. When a circular structure isencountered, a mass or metastasis may be suspected. Scanning away from the mass may reveal a tubulovascular shapewhich has been imaged and cross sectioned. Flattening of the circular mass by external compression with the ultrasoundprobe will also differentiate a vascular structure from a solid mass.

94. The following statement(s) is/are true concerning hepatic anatomical nomenclature.

a. In the traditional English system, the right lobe is divided into anterior and posterior segments by anintersegmental line with no topographic landmarks or interparenchymal septi

b.The caudate lobe in the French or Couinaud’s nomenclature is referred to as segment Ic. The right lobe of the liver by English nomenclature is subdivided in the French system into segments V–VIIId.In the English system, the left lobe of the liver is divided into the medial segment and lateral segment by the

falciform ligamentAnswer: a, b, c, d

Until recently, anatomic descriptions in the English literature began with the major divisions of liver into a right and leftlobe separated by a vertical line drawn from the gallbladder fossa to the inferior vena cava. The left lobe is furtherdivided by the falciform ligament into a medial segment and lateral segment. The right lobe is further divided into ananterior and posterior segment by an intersegmental line which has no reliable topographical landmarks and nointerparenchymal septi to allow easy identification. The French nomenclature also known as Couinaud’s nomenclatureenumerates the segments of the liver beginning with segment I or the caudate lobe. Segments II, III, and IV make upmost of the “English” left lobe, while segments V through VIII represent the English nomenclature right lobe.

95. A 57-year-old man with a history of Duke’s C colon cancer is being evaluated for a rising CEA. Which of thefollowing statement(s) is/are correct concerning the use of CT scanning for this indication?

a. Conventional CT scanning will detect lesions well below 1 cm in size

Page 58: Hepatobiliary & pancreatic surgery

b.CT arterio-portography involves immediate CT scanning after direct injection into both the common hepaticartery and superior mesenteric artery

c. A double helical (spiral) CT scan may eliminate the need for invasive angiographyAsir Surgery MCQs Bank. © 1422H-2002- first impression ©

This project was raised after an idia by Dr. Gharama Al-Shehri (consultant surgeon). Developed and typed by Dr. Ghazi Al-Shumrani (intern).

Page 59: Hepatobiliary & pancreatic surgery

Hepatobiliary & pancreatic surgeryd.Magnetic resonance imaging of the liver will add little to the workup of this patient

Answer: b, c, d

CT scanning has been used increasingly to screen for hepatic and other intra-abdominal or retroperitoneal lesions.Conventional CT scanning includes 0.5–1 cm axial images of the liver after oral administration of barium and bolusinjection of intravenous contrast. Although resolution has improved, hepatic lesions below 1 cm in size or lesions thatare isodense with hepatic parenchyma may be missed. Resolution of hepatic lesions has been greatly enhanced by thecombination of visceral angiography and CT scanning, known as CT arterio-portography (CTAP). Immediate CTscanning after injection of contrast directly into the common hepatic artery may identify small hepatic lesions whichusually show increased density relative to the surrounding hepatic parenchyma. CT arterio-portography also includesdirect injection of contrast into the splenic or superior mesenteric arteries, with CT imaging during the portal venousphase of this injection. Hepatic lesions supplied by the hepatic artery thus appear as discrete hypodense lesionssurrounded by normal hepatic parenchyma enhanced by portal venous contrast. Recently, double helical (spiral) CTscanning has become available and shows considerable promise to complement or replace CTAP for preoperativeimaging. This technique allows total hepatic imaging in both the arterial and arterial/venous phases after a single rapidbolus injection of intravenous contrast during a single breath hold by the patient. It is possible to visualize the portalstructures and hepatic veins on a single scan and give a high resolution of small hepatic lesions. In addition, three-dimensional reconstructions can be created to further delineate hepatic parenchyma and demonstrate a CT constructedhepatic arteriogram. This technique may completely replace the need for invasive arteriography to characterize theblood supply to the liver prior to hepatic resection or after hepatic transplantation. Magnetic resonance imaging of theliver has results similar to CT scanning, but to date has not demonstrated improvements sufficient to justify theincreased cost associated with the technique.

96. A solitary 6 cm lesion is identified in the right hepatic lobe in the patient described above. Which of thefollowing statement(s) is/are true concerning the initial operative management?

a. To facilitate mobilization and assessment with intraoperative ultrasound, complete mobilization includingdividing the left and right triangular ligaments would be necessary

b.In dividing the right triangular ligament, care must be taken to avoid injury to accessory right hepatic veinsdraining directly into the vena cava

c. Unless a considerable length of hepatic vein is found outside the hepatic parenchyma, early hepatic veinligation should be avoided

d.Ligation of the portal arterial structures is always necessary before proceeding with hepatic lobectomyAnswer: a, b, c

For major hepatic resections and for complete intraoperative ultrasound, complete mobilization of the liver will berequired. After detachment of the hepatic flexure of the colon and division of the falciform ligament, both the left andright triangular ligaments must be sharply taken down to fully mobilize the liver. During division of the right triangularligament, care must be taken to avoid injury to the right diaphragm, the right adrenal gland and adrenal vein, the rightphrenic vein, and several moderate-size accessory right hepatic veins draining directly into the vena cava. Aftermobilization, digital and bimanual palpation is performed and intraoperative ultrasound may be performed. Dissectionof the porta hepatis is performed by many hepatic surgeons to identify the main bifurcations of the hepatic artery, bileduct, and portal vein. This allows individual ligation of unilateral branches of each of these structures during hepaticlobectomy but prior to parenchymal dissection. An alternative approach has been recently described where the mainportal structures are left undisturbed and branches to a given lobe are ligated during parenchymal transection.Hemorrhage can be minimized by intermittent portal inflow occlusion by clamping or compression of the portal triad(Pringle maneuver). There has been considerable debate over early versus late isolation and ligation of a given hepaticvein during lobectomy since the extraparenchymal component of the hepatic vein may be quite short or absent. Sincehemorrhage in this location may be difficult to control, a safe strategy is to always avoid early isolation of a givenhepatic vein or to attempt isolation only when a considerable length of vein is found on mobilization of the respectivetriangular ligament.

97. Which of the following statement(s) is/are true concerning the arterial venous anatomy of the liver?

a. Most commonly, the right, left, and middle hepatic veins join the inferior vena cava as a separate trunkb.Most frequently, the entire length of each hepatic vein is within the parenchyma of the liverc. A replaced right hepatic artery may be placed in jeopardy during performance of a pancreaticoduodenectomyd.There is little collateral arterial circulation between the right and left hepatic lobes

Answer: b, c

Page 60: Hepatobiliary & pancreatic surgery

There are three major hepatic veins which carry blood from the central veins of the hepatic substance to the inferiorvena cava (IVC). In two-thirds of patients, there is a single large right hepatic vein which joins the right anterior wall ofAsir Surgery MCQs Bank. © 1422H-2002- first impression ©

This project was raised after an idia by Dr. Gharama Al-Shehri (consultant surgeon). Developed and typed by Dr. Ghazi Al-Shumrani (intern).

Page 61: Hepatobiliary & pancreatic surgery

Hepatobiliary & pancreatic surgerythe IVC and a middle and a left hepatic vein which converge one-to-two cm from the IVC and enter the left anteriorwall of the IVC as a single vessel. In one-third of patients, each major hepatic vein joins at the same horizontal level ofthe IVC as a separate trunk. In some patients, there is a short but definable extraparenchymal segment of one or more ofthe hepatic veins at the confluence with the IVC. More frequently, the entire length of the hepatic veins isintraparenchymal, which may preclude early, safe hepatic venous isolation during hepatic resection.There is considerable variability in the origin and course of the right and left hepatic arteries. The most common finding(55% of patients) is a transverse common hepatic artery from the celiac trunk which gives off the gastroduodenal, rightgastric, and supraduodenal arteries and courses obliquely in the left anterior aspect of the hepatoduodenal ligament as aproper hepatic artery. After giving off the cystic artery to the gallbladder, there is then a fairly low trifurcation into asingle right, middle, and left hepatic arteries. Knowledge of the most common variations is extremely importance sinceinadvertent division may occur during gastric, pancreatic, and hepatobiliary procedures. There may be a replaced oraccessory left hepatic artery which arises from the left gastric artery and courses transversely in the lesser omentum.With nearly equal frequency, there is a replaced or accessory right hepatic artery from the superior mesenteric arterynear its origin which courses posterior or through the head of the pancreas obliquely along the right posterior border ofthe hepatoduodenal ligament. Although original anatomic descriptions deny the existence of collateral vessels to theopposite hepatic lobe, image perfusion studies after ligation of main or replaced hepatic arteries have clearlydemonstrated the presence of collateral flow to the deprived lobe.

98. The liver plays a vital role in carbohydrate metabolism and regulation of blood glucose. The followingstatement(s) is/are true concerning carbohydrate metabolism by the liver.

a. Glycogen, a complex polymer of glucose, is synthesized by the hepatocyte in a remarkably energy efficientprocess

b.Glucagon stimulates glycogenesisc. Glycolysis, the process by which glucose is converted to two molecules of pyruvate, occurs in the liver

mitochondriad.If glycogen stores become depleted, the liver is capable of synthesizing new glucose by the process of

gluconeogenesis, which is stimulated by insulinAnswer: a

Serum glucose is tightly regulated by the liver despite wide fluctuations in dietary ingestion. The liver can take up asmuch as 100 g/day of glucose and convert it to glycogen by the process of glycogenesis. The liver can also releaseglucose into the blood by glycogenolysis, the breakdown of glycogen, or by gluconeogenesis, the formation of newglucose from substrates such as alanine, lactate, glycerol or dietary amino acids. Hormones play a key role in hepaticregulation of glucose metabolism. Insulin, for example, stimulates glycogenesis, and glucagon stimulatesglycogenolysis and gluconeogenesis. Gluconeogenesis is also enhanced by fasting, critical illness and periods ofanaerobic metabolism.Glycogen is a complex polymer of glucose. Liver cells can store up to 8% of their weight as glycogen. The first step inglycogen storage is the transport of glucose through the hepatocyte plasma membrane. About 90% of portal venousglucose is removed from the blood by liver cells through carrier-facilitated diffusion. The rate of glucose transport isenhanced by insulin. Once in the hepatocyte, glucose and ATP are converted by the enzyme glucokinase to glucose-6-phosphate (G6P), the first intermediate in the synthesis of glycogen. Because complete oxidation of one molecule ofG6P generates 37 molecules of ATP, and storage only uses one molecule of ATP, the overall efficiency of glucosestorage in glycogen is a remarkable 97%. Glycolysis is the pathway by which glucose is converted to two molecules ofpyruvate and occurs in the cytoplasm in contrast to the citric acid cycle which occurs in the mitochondria.

99. Transport of substances from the blood into the hepatocyte occurs through the sinusoidal membrane. Thefollowing statement(s) is/are true concerning this plasma membrane.

a. The high lipid content of this phospholipid bilayer allows lipid-soluble molecules to enter the cell by simplediffusion

b.Carrier proteins within the phospholipid bilayer bind to a solute in blood and by conformational change allowit to be transported into the cell

c. Large glycoprotein molecules of the sinusoidal membrane known as receptors always transport the bindingligand into the cell

d.The transmission of a signal to the interior of the cell by receptor-ligand binding which generates intracellularsecond messengers is known as signal transduction

Answer: a, b, d

Page 62: Hepatobiliary & pancreatic surgery

The hepatocyte plasma membrane consists of a phospholipid bilayer in which hydrophobic fatty acid tails are orientedto the interior membrane and hydrophilic phospholipid head groups are oriented to the exterior (sinusoidal orcytoplasmic) membrane. Within this phospholipid bilayer are proteins which serve either structural functions orAsir Surgery MCQs Bank. © 1422H-2002- first impression ©

This project was raised after an idia by Dr. Gharama Al-Shehri (consultant surgeon). Developed and typed by Dr. Ghazi Al-Shumrani (intern).

Page 63: Hepatobiliary & pancreatic surgery

Hepatobiliary & pancreatic surgerymetabolic functions. The hepatocyte sinusoidal plasma membrane is heavily studded with microvilli to increase theabsorptive area in contact with sinusoidal blood. The cell membrane, by virtue of its high lipid content, allows lipid-soluble molecules to enter the cell by simple diffusion. Polar molecules must enter cells via membrane transportproteins. Channel proteins allow molecules to diffuse simply into cells without binding, whereas carrier proteins firstbind the solute and, by conformational change, allow it to be transported into the cell. The glucose carrier inhepatocytes is an example of carrier-facilitated diffusion. The sinusoidal membrane is studded with receptors, which arelarge glycoprotein molecules that span the plasma membrane lipid bilayer. A ligand-binding site of this receptormolecule projects into the space of Disse. When appropriate ligand-receptor binding occurs, the entire ligand may beinternalized for intracellular degradation or biliary transport, or the ligand may transmit a signal to the interior of thehepatocyte by a number of intracellular second messenger systems, a process known as signal transduction. Suchsecond messengers include cAMP, inositol triphosphate, and diacylglycerol. Each of these structurally simple chemicalscan amplify cell membrane events and bring about major changes in cellular physiology.

100.The liver is an important site of protein metabolism. Which of the following statement(s) is/are true concerningprotein metabolism by the liver?

a. Amino acids are taken up by hepatocytes by active transport mechanisms and are generally stored long-termfor later synthetic activity

b.Under certain conditions the amine group is removed from the amino acids in the liver and the carbon chainused for carbohydrate, lipid, or nonessential amino acid synthesis

c. The most important route of detoxification of ammonia formed as the result of deamination of amino acids isvia excretion of ammonia into the urine

d.Proteins synthesized by the liver include albumin, transferrin, fibrinogen, and apolipoproteinse. Albumin is a sensitive indicator of hepatic synthetic function

Answer: b, d

Essentially all of the end products of dietary protein digestion are amino acids, which are absorbed by the enterocytesinto the portal circulation in ionized states. Amino acids are taken up by hepatocytes by one of several active transportmechanisms. Amino acids are not stored in the liver but are rapidly used in the production of plasma proteins, purines,heme proteins, and hormones. Under certain conditions, the amine group is removed from the amino acids, and thecarbon chain is used for carbohydrate, lipid, or nonessential amino acid synthesis. The ammonia formed as the result ofdeamination of amino acids is detoxified by one of two routes. The most important pathway involves conversion ofammonia to urea by enzymes of the Krebs-Henseleit cycle, found only in the liver. A second route of ammoniametabolism involves deamination of L-glutamine by the kidney, with excretion of ammonia into the urine.Essentially all albumin, fibrinogen, and apolipoproteins are derived from the liver which can add up to 50 grams ofprotein to the plasma per day. Of total hepatic protein synthesis, 75% is destined for export in plasma. Albumin, animportant plasma protein synthesized in the liver, has a long half-life in plasma of about 19 days. This long half-lifemakes albumin an insensitive indicator of hepatic synthetic function.

101.The following statement(s) is/are true concerning hepatic blood flow.

a. Although constituting only 2.5% of total body weight, the liver receives 25% of the cardiac outputb.Hepatic blood flow is equally derived from the portal vein and hepatic arteryc. The liver serves as a physiologic blood reservoir either releasing blood back into the systemic circulation at

times of acute blood loss or in situations of volume overload serving as a site of extra blood storaged.An important function of the liver is to filter particulate debris which is performed by phagocytic Kupffer cells

which line the hepatic sinusoidal endotheliumAnswer: a, c, d

The liver constitutes about 2.5% of the total body weight but receives 25% of the cardiac output. Total hepatic bloodflow is 100 to 130 ml/min/kg. About two-thirds of total hepatic blood flow is derived from the portal vein and one-thirdfrom the hepatic artery. The liver also serves as a physiologic blood reservoir. About 25% to 30% of the liver volume isaccounted for by blood, and in cases of acute blood loss up to 30%, or as much as 300 ml of the hepatic blood volumecan be released into the systemic circulation without adverse effects on liver function. Conversely, in the case of rightheart failure or other causes of systemic volume overload, as much as one liter of extra blood can be stored in the liverbefore passive congestion and liver injury occur. The hepatic sinusoids are lined by an endothelium punctuated withpores that allow proteins and other particles to diffuse out of the vascular tree and into proximity with hepatocytes. Thisextreme permeability of the liver allows rapid exchange of a diverse number of nutrients, hormones and environmentalagents between the blood and the hepatocyte. The liver also acts as a filter for particulate debris, which enters the portal

Page 64: Hepatobiliary & pancreatic surgery

circulation through intestinal capillaries. Particles such as bacteria are ingested by Kupffer cells by the process ofphagocytosis. Kupffer cells line the hepatic sinusoidal endothelium where formed blood elements and matter may be indirect contact with these phagocytic cells.Asir Surgery MCQs Bank. © 1422H-2002- first impression ©

This project was raised after an idia by Dr. Gharama Al-Shehri (consultant surgeon). Developed and typed by Dr. Ghazi Al-Shumrani (intern).

Page 65: Hepatobiliary & pancreatic surgery

Hepatobiliary & pancreatic surgery

102.The liver synthesizes key metabolites essential for the production of fuel substrates for other organs. These keymetabolites include:

a. Glucose-6-phosphate (G6P)b.Acetyl CoAc. Pyruvated.Oxaloacetate

Answer: a, b, c

Hepatic processes in the liver are essential for the production of fuel substrates for other organs. The liver, by virtue ofits terminal position in the portal system, is the organ that must regulate intestinally absorbed nutrients for tissueconsumption or storage. The liver accomplishes its task by synthesizing three key metabolites: -glucose-6-phosphate,pyruvate and acetyl CoA. G6P can be stored as glycogen or converted into glucose, pyruvate, or ribose-5-phosphate (anucleotide precursor). Pyruvate can be converted into lactate, alanine (and other amino acids), and acetyl CoA, or it canenter the tricarboxylic acid cycle. Acetyl CoA is converted to HMG-CoA (a cholesterol and ketone body precursor) orcitrate (for fatty acid and triglyceride synthesis), or it is degraded to carbon dioxide and water for energy.

103.The following statement(s) concerning hepatic bile formation/secretion is/are true.

a. The adult human liver secretes less than 1000 cc of bile dailyb.Most bile is secreted by hepatocytes (canalicular bile)c. Primary bile acids include cholic acid, chenodeoxycholic acid, and deoxycholic acidd.The enterohepatic circulation is tremendously efficient in reabsorption of intestinal bile acidse. Bile acids are the primary determinant of bile flow

Answer: b, d, e

The adult human liver secretes about 1.5 liters of bile daily. Eighty percent of this volume is secreted by the hepatocytes(canalicular bile) and 20% is secreted by the bile duct epithelial cells (ductular bile). Solutes constitute about 3% of bile.The major solutes are conjugated bile acids, phosphatidyl choline, cholesterol, protein and bilirubin. Bile acids are themain determinant of bile production, and canalicular bile flow is traditionally divided into bile acid-dependent and bileacid-independent components. Primary bile acids are synthesized from cholesterol in the liver and in humans consist ofcholic acid and chenodeoxycholic acid. Secondary bile acids are formed in the intestinal lumen by bacterialdehydroxylation and consist of deoxycholic acid and lithocholic acid derived from cholic acid and chenodeoxycholicacid, respectively. Essentially all primary and secondary bile acids are conjugated with the amino acids glycine ortaurine. The human liver synthesizes 300 to 400 mg per day of bile acids from cholesterol, or about 10% of the total bilesalt pool. Normally intestinal bile acids are efficiently (about 95%) taken up by the enterohepatic circulation. Luminalbile acids are transported by carrier proteins in the distal ileum and appear in the portal venous effluent. The hepatocyteextracts more than 95% of portal venous bile acids for resecretion into the bile.

104.The following statement(s) is/are true containing lipid metabolism in the liver.

a. Hepatic mitochondrial hydrolysis of fatty acids is a tremendous source of ATPb.Significant hepatic storage of triglyceride or fatty infiltration can cause hepatic fibrosis or necrosisc. Approximately 90% of cholesterol synthesis occurs in the liverd.Most cells in the body are capable of phospholipid synthesis, therefore the liver plays a minimal role in this

processAnswer: a, c

The liver has a number of important functions in the metabolism of lipids: 1) the synthesis of apolipoproteins, 2) thedegradation of fatty acids into energy substrates, 3) the synthesis of triglycerides from carbohydrates and proteins, and4) the synthesis of cholesterol and phospholipids from fatty acids. The mitochondrial hydrolysis of fatty acids is asource of large quantities of ATP. The conversion of stearic acid to CO2 and H2O, for instance, generates 136 ATPmolecules and demonstrates the highly efficient storage of energy in fat. In times of unrestrained lipolysis, such asstarvation, uncontrolled diabetes or other conditions of triglyceride mobilization from adipose tissue, the ability of theliver to perform beta-oxidation may be inadequate. Under these circumstances, significant hepatic storage oftriglycerides or fatty infiltration of the liver may occur. Triglyceride storage by itself does not appear to be a cause ofhepatic fibrosis or necrosis, but fatty infiltration may be a marker for derangement of normal processes by alcohol or

Page 66: Hepatobiliary & pancreatic surgery

drug toxicity, diabetes, chronic parenteral nutrition, or morbid obesity.Cholesterol is an important regulator of membrane fluidity and is a substrate for bile acid and steroid hormonesynthesis. Cholesterol may be available by dietary intake or by de novo synthesis. In mammals, about 90% of new

Asir Surgery MCQs Bank. © 1422H-2002- first impression ©

This project was raised after an idia by Dr. Gharama Al-Shehri (consultant surgeon). Developed and typed by Dr. Ghazi Al-Shumrani (intern).

Page 67: Hepatobiliary & pancreatic surgery

Hepatobiliary & pancreatic surgerycholesterol is synthesized by the liver from its precursor, acetyl CoA. Dietary cholesterol intake suppresses endogenoussynthesis by inhibiting the rate-limiting enzyme in cholesterol by a synthetic pathway, HMG-CoA reductase. There arethree major classes of phospholipids synthesized by the liver: the lecithins, the cephalins, and the sphingomyelins.Although most cells in the body are capable of some phospholipid synthesis, the liver produces 90%.

105.Hepatic biotransformation is defined as the intracellular metabolism of endogenous and exogenous organiccompounds. Which of the following is/are enzyme families responsible for hepatic bile transformation?

a. Cytochromes P-450b.UDB-glucuronyl transferasesc. Glutathione-transferasesd.Sulfotransferases

Answer: a, b, c, d

The liver contains enzyme systems that can expose functional groups such as hydroxyl ions and alter the size andsolubility of a wide variety of organic and inorganic compounds by conjugation with small polar molecules. Thegeneral strategy of the liver is to convert hydrophobic, potentially toxic compounds into hydrophilic conjugates that canthen be excreted into bile or urine. There are four general enzyme families responsible for hepatic bile transformation.The cytochromes P-450 catalyze reactions such as oxidation, hydroxylation, sulfoxide formation, oxidativedeamination, dealcoholization and dehalogination. Such reactions allow further phase II conjugation with polar groupssuch as glucuronate, glutathione and sulfate. Glucuronidation is the conjugation of UDB-glucuronic acid to a widevariety of xenobiotics by either esther or ether linkages. The glutathione transferases and sulfotransferases play a role inconjugation of P-450 derivatives. However, the glucuronyl transferase system is the predominant mechanism.

106.The following statement(s) is/are true concerning the differential diagnosis between an amoebic and apyogenic liver abscess.

a. The clinical presentation is often clearly distinguishableb.A history of travel or origin from a high risk area might suggest an amebic liver abscessc. Routine liver chemistries frequently can distinguish pyogenic from amoebic liver abscessd.Serologic testing for the presence of antibody to entamoeba histolyctica is the only specific and sensitive way

to confirm the diagnosis of amoebic liver abscesse. Distinguishing pyogenic from hepatic abscesses preoperatively is not important since surgical drainage is

imperative for bothAnswer: b, d

Distinguishing amoebic from pyogenic liver abscess can be a diagnostic challenge. It is of major importance, however,because effective medical therapy with metronidazole can obviate the need for either percutaneous or surgical drainagein most cases of amoebic abscess. The clinical presentation for both conditions with acute onset of fever, abdominalpain, and altered liver function tests are almost identical. Important features such as travel to or origin from a high riskarea is particularly important for amebic liver abscess. Routine liver chemistries and radiographic studies can rarelydistinguish between amoebic and pyogenic liver abscesses. Specific serologic tests for the presence of antibody to E.histolytica are specific and sensitive for amoebic hepatic abscess being positive in 95% of the cases, and therefore, arekey in distinguishing the two infections.

107.A patient is found to develop evidence of hepatitis approximately eight weeks after receiving bloodtransfusions during a surgical procedure. Which of the following statement(s) is/are true?

a. The virus responsible is most likely hepatitis Cb.A chronic carrier state will ultimately develop in most patientsc. There is no role for interferon in the treatment of chronic hepatitis C viral infectiond.Chronic infection with hepatitis C is not associated with an increased risk of developing hepatocellular

carcinomaAnswer: a, b

Hepatitis C virus is a virus that is responsible for more than 90% of post-transfusion hepatitis and most sporadic non-A,non-B hepatitis throughout the world. The most common identifiable sources of acquisition of hepatitis C virus are priortransfusion of blood or blood-derived products or a history of intravenous illicit drug use. The usual incubation period

Page 68: Hepatobiliary & pancreatic surgery

of post-transfusion hepatitis C viral infection is 5 to 10 weeks. An initial elevation of liver enzymes may be associatedwith little or no clinical disturbance. In some patients, acute hepatitis C viral infection does not progress to chronicinfection, however, chronic hepatitis C viral infection develops in up to 70% of patients with post-transfusion hepatitisAsir Surgery MCQs Bank. © 1422H-2002- first impression ©

This project was raised after an idia by Dr. Gharama Al-Shehri (consultant surgeon). Developed and typed by Dr. Ghazi Al-Shumrani (intern).

Page 69: Hepatobiliary & pancreatic surgery

Hepatobiliary & pancreatic surgeryC infection with many progressing to cirrhosis. Hepatitis C does not appear to alter life expectancy at least in the first15 years of infection. However, once cirrhosis and end stage liver disease develop, the clinical syndrome isindistinguishable from other forms of chronic liver disease with a predisposition to the development of hepatoma.Interferon alpha is the only FDA approved therapy for chronic hepatitis C viral infection. There is some evidence thatearly administration of interferon in acute hepatitis C viral infection may reduce the risk of progression to the chronicstate. As yet, there is no evidence that interferon alters the natural history of chronic hepatitis C viral infection orchanges the incidence.

108.A surgeon is suspected of having contacted hepatitis B virus via needle stick. Which of the followingstatement(s) is/are true concerning his diagnosis and outcome?

a. Incubation of hepatitis B virus is about two weeksb.Jaundice is the first serologic indicator of hepatitis B infectionc. The patient has about a 10% chance of developing a chronic carrier stated.All susceptible household or sexual contacts of the surgeon should receive hepatitis B viral vaccinee. The surgeon should receive hepatitis B immunoglobulin as soon as possible after the accidental needle stick

Answer: c, d, e

Hepatitis B viral infection is insidious. The incubation period of the virus is about eight weeks. The first serum indicatorof infection by hepatitis B virus is detection of the serum hepatitis B surface antigen (HBsAg) which may proceed theonset of jaundice. In most cases, hepatitis B infection is self-limited and does not progress to chronic hepatitis.However, some 10% of patients with acute hepatitis B viral infection, whether it is clinical or subclinical, will develop achronic carrier state. The carrier state is defined by the presence of HBsAg in serum for longer than six months. Thebest method of treatment of hepatitis B viral infection is primary prevention by vaccination. All susceptible householdor sexual contacts of a person with a positive serum test for HBsAg should be advised to receive a full course ofhepatitis B viral vaccine. Passive prophylaxis with hepatitis B immunoglobulin should be provided to any susceptiblecontact in whom there is recent potential parenteral exposure such as an accidental needle stick.

109.The following statement(s) is/are true concerning the diagnosis and treatment of hydatid cysts.

a. Percutaneous aspiration is an important aspect of diagnosis and treatment of a hydatid cystb.CT scan will oftentimes show the classic findings of a cystic liver lesion with a calcific rimc. At operation, care must be taken to protect the operative field from spillage of the cyst fluidd.The use of a scoleocide has become obsolete with current surgical techniques

Answer: b, c

Hydatid cysts are most commonly the result of infection with the tape worm, Echinococcus granulosis. Routinelaboratory tests in patients with hydatid cysts are normal or nonspecifically abnormal. Although routine chest orabdominal radiographs may show a mass with a calcific rim, sonography and CT scan are the favored means of imaginghydatid cysts. The presence of calcifications and daughter cysts within the parent cyst suggests Echinococcus.Percutaneous needling of a hydatid cyst is unwise unless precautions against anaphylaxis are undertaken. A cyst’s fluidis often under pressure, and needling may precipitate rupture with the potential for anaphylaxis or intraperitonealseating. The classic treatment of hydatid cysts is operative. The surgical aim is to remove the cyst or cysts withoutdissemination of the organism. At operation, the cyst is drained of fluid through a cannula after carefully protecting theoperative field from fluid leakage. If the aspirate is clear a parasiticidal fluid (ethyl alcohol or 20% sterile saline) isinjected into the cyst to kill any adherent scoleces. The cyst contents and the pericystic wall is then removed withcareful surgical dissection.

110.Which of the following statement(s) is/are true concerning treatment of pyogenic liver abscess?

a. Antibiotic therapy alone may be advisable in patients with multiple small abscessesb.Percutaneous drainage provides comparable results to surgical drainage in patients with unilocular large

abscessesc. Sufficient antibiotic coverage for most hepatic abscesses includes coverage for gram-positive aerobic bacteria

onlyd.In patients with a primary biliary origin for the hepatic abscess, treatment must also be addressed at underlying

biliary pathology such as choledocholithiasis or biliary ductal obstructionAnswer: a, b, d

Page 70: Hepatobiliary & pancreatic surgery

The preferred treatment of most patients with hepatic abscesses is broad-spectrum antibiotic coverage and drainage. Anumber of studies have demonstrated for most patients with large unilocular abscesses that percutaneous catheterAsir Surgery MCQs Bank. © 1422H-2002- first impression ©

This project was raised after an idia by Dr. Gharama Al-Shehri (consultant surgeon). Developed and typed by Dr. Ghazi Al-Shumrani (intern).

Page 71: Hepatobiliary & pancreatic surgery

Hepatobiliary & pancreatic surgerydrainage is as effective as surgical drainage. Bacteria that predominate in pyogenic liver abscesses are gram-negativeaerobes, streptococcal species, and anaerobes. Therefore, broad-spectrum antibiotic coverage is necessary. Antibioticcoverage alone may be advisable in occasional patients who have multiple small abscesses not accessible topercutaneous or surgical drainage. Since many of these patients have an underlying biliary pathology as the source ofthe hepatic abscess, correcting this underlying pathology, for example, establishing biliary drainage surgically ornonoperatively is important.

111.Which of the following statement(s) is/are true concerning acute, fulminant hepatic failure?

a. The most frequent cause of acute hepatic failure world-wide is hepatitis B infectionb.Higher grades of encephalopathy are associated with a worse prognosisc. Hypoglycemia is a common complication of all liver diseasesd.Liver transplantation would appear indicated in all patients with hepatic coma secondary to acute liver failure

Answer: b

The diagnosis of acute (fulminant) hepatic failure is based on the development of encephalopathy within eight weeks ofthe onset of symptoms. The overall prognosis is poor, but the hepatic lesions are potentially reversible, and recovery canlead to restoration of normal liver function. The most frequent cause of acute hepatic failure world-wide is non-A, non-B viral hepatitis. A variety of other viral agents and hepatotoxins can also cause this condition.No reliable criteria predict outcome and response to treatment. Higher grades of encephalopathy (depth of coma) onadmission are associated with the worst prognosis. Management should include general supportive measures andspecific treatment for hepatic encephalopathy, cerebral edema, electrolyte and metabolic disturbances, infection, andpain. Hypoglycemia is an unusual complication of most liver diseases except in patients with acute hepatic failure orhepatic neoplasms. The enormous reserve capacity of the liver accounts for the rarity of hypoglycemia except as apreterminal event. Bleeding is also a frequent cause of death in patients with acute hepatic failure secondary todepressed liver synthesis of clotting factors and qualitative or quantitative platelet disorders. The lack of a definitivemedical treatment for acute hepatic failure makes liver transplantation seem attractive especially for patients with littleor no chance of recovering normal liver function. Perhaps the most significant drawback to widespread acceptance ofliver transplantation for acute hepatic failure is the lack of criteria reliability to predict which patients are likely tobenefit from operation. Patients with mild to moderate degrees of coma are likely to recovery spontaneously without theneed for liver transplantation while rapid deterioration and neurologic status to grade III or grade IV coma areassociated in some centers with a mortality of 95%.

112.Which of the following statement(s) is/are true concerning the natural history and clinical features of alcoholiccirrhosis?

a. In patients with compensated cirrhosis, the probability of survival at 10 years approaches 50%b.The development of clinical evidence of hepatic decompensation reduces five year survival to less than 20%c. Continued consumption of alcohol worsens prognosisd.The risk of death after variceal hemorrhage depends more on the severity of underlying liver disease than the

type of therapyAnswer: a, b, c, d

Recent studies have analyzed the natural history of cirrhosis as a function of the degree of hepatic decompensation atthe time of diagnosis. A high proportion of patients with compensated cirrhosis remain well for many years afterdiagnosis. In these studies the probability of remaining compensated 10 years after diagnosis was 42%, and survivalprobability of compensated patients was 47%. The prognosis worsened considerably once patients developed clinicalevidence of hepatic decompensation (ascites, jaundice, encephalopathy, or gastrointestinal hemorrhage). Among thesepatients, the probability of five-year survival was only 16%. The risk of death from variceal hemorrhage depends muchmore of the severity of the underlying liver disease than on the type of therapy. It would also appear from naturalhistory studies that continued alcohol consumption does affect survival. In one study, the overall five-year survival is63% for abstainers versus 40.5% for those who continued to drink. Continued alcohol consumption may have less of aneffect on survival than the intensity of alcohol consumption. Furthermore, the degree of hepatic compensation at thetime of inclusion into the study may have also been an important factor.

113.Which of the following statement(s) is/are true concerning the morphologic and histologic findings ofcirrhosis?

Page 72: Hepatobiliary & pancreatic surgery

a. Micronodular cirrhosis is a pattern typical of chronic alcoholic liver diseaseb.Mallory bodies and megamitochondria are typical findings of alcoholic cirrhosis

Asir Surgery MCQs Bank. © 1422H-2002- first impression ©

This project was raised after an idia by Dr. Gharama Al-Shehri (consultant surgeon). Developed and typed by Dr. Ghazi Al-Shumrani (intern).

Page 73: Hepatobiliary & pancreatic surgery

Hepatobiliary & pancreatic surgeryc. Bile leaks caused by rupture of bile ducts with extravasation of bile into portal triads is a common finding in

post-necrotic cirrhosis secondary to hepatitisd.Large regenerating nodules separated by coarse irregular scars in piecemeal parenchyma necrosis is common

in liver disease secondary to chronic active hepatitisAnswer: a, b, d

Morphologic classification of cirrhosis includes micronodular, macronodular, and mixed forms. Micronodular cirrhosisis characterized by uniform nodules and scars. The nodules are usually less than 3 mm in diameter and are typicallyassociated with Laennec or nutritional cirrhosis in alcoholics. Post-necrotic cirrhosis is characterized by largeregenerating nodules separated by coarse irregular broad as well as thin scars. This pattern is frequently seen in patientswith viral hepatitis. Biliary cirrhosis is characterized by a coarsely granular macronodular liver. This condition resultsfrom long-standing cholestasis secondary to obstruction of intrahepatic or extrahepatic bile ducts. The most distinctivefeature of large duct obstruction is the presence of bile leaks caused by rupture of bile duct with extravasation of bileinto portal triads. Portal cirrhosis, which is typically observed in alcoholics, can generally be distinguishedhistologically by the presence of several specific hepatocellular alterations such as Mallory bodies andmegamitochondria.

114.The following statement(s) is/are true concerning the management of ascites associated with chronic liverdisease.

a. Spontaneous bacterial peritonitis is an insignificant complicationb.Large volume paracentesis is unsafe due to excessive volume loss from the intervascular spacec. Peritoneovenous shunting is a trivial surgical procedure with minimal perioperative morbidity and mortalityd.Transjugular intrahepatic portosystemic shunts (TIPS) can effectively treat ascites in patients refractory to

conventional medical therapyAnswer: d

The onset of ascites usually indicates the presence of advanced liver disease. Cirrhotic ascites is usually straw colored,clear, or greenish. Spontaneous bacterial peritonitis occurs as a complication of cirrhotic ascites in up to 10% ofpatients. Spontaneous bacterial peritonitis is defined as infected ascitic fluid without a demonstrable other site ofinfection. This is a serious complication with reported in-hospital mortality rates of 60% to 90%. The rational approachof therapy for ascites includes sodium and fluid restriction, the use of diuretics, and the use of therapeutic paracentesis.Several studies have shown that repeated paracentesis in stable cirrhotic patients may be safe and effective as medicaltherapy and shortens the length of hospitalization. Single, large volume paracentesis has been reported to be effectiveand safe. Up to 10 liters of ascites can be removed in one hour if salt-poor albumen is administered simultaneously. In asmall percentage of patients, surgical implantation of a peritoneovenous shunt may be advisable. The principalindication for use of peritoneovenous shunt is to stabilize ascites that is refractory to conventional medical therapy andtherapeutic paracentesis. Despite the simplistic nature of the device, postoperative mortality and morbidity rates of 20%to 60%, respectively have been reported. Precipitation of disseminated intravascular coagulopathy, varicealhemorrhage, or hepatic failure may complicate this procedure. Transintrahepatic portosystemic shunts (TIPS) have beendemonstrated to control ascites in one study in over 90% of patients with ascites refractory to medical management.However, patients with poor hepatic reserve in this study all died if orthotopic liver transplantation was not performed.This data suggests that TIPS is effective for refractive ascites in patients with good to moderate hepatic reserve but poorrisk cirrhotics require orthotopic liver transplantation to correct this problem.

115.Which of these statement(s) is/are true concerning the etiologic factors in the development of cirrhosis?

a. Viral hepatitis of any type (A, B, or non-A, non-B) can all progress to cirrhosisb.Acetaminophen can cause acute liver failure and necrosis but will not lead to cirrhosisc. Alcohol exerts toxic effects on the liver via reactive intermediates such as acetaldehyded.Long-standing congestive heart failure can lead to cirrhosis secondary to centrilobular congestion, hemorrhage,

and necrosisAnswer: c, d

Liver cells are sensitive to a variety of physical, microbiologic, and chemical agents, all of which may produce cellularinjury. The eventual development of cirrhosis is determined by the nature and severity of the cellular injury and theliver’s ability to regenerate. Most infectious hepatitides are viral in origin. There is no documentation that hepatitis Aprogresses to cirrhosis, however hepatitis B and non-A, non-B hepatitis do demonstrate a propensity to become chronicwith the development of cirrhosis. Chemical hepatotoxicity include direct and indirect actions. Acetaminophen under

Page 74: Hepatobiliary & pancreatic surgery

normal circumstances is detoxified chiefly by conjugation with glucuronic acid or sulfate. Saturation of glucuronicpathway with large doses of acetaminophen results in progressive depletion of intracellular glutathione stores,accumulation of toxic intermediate, and eventual cell necrosis which may progress to either acute liver failure orAsir Surgery MCQs Bank. © 1422H-2002- first impression ©

This project was raised after an idia by Dr. Gharama Al-Shehri (consultant surgeon). Developed and typed by Dr. Ghazi Al-Shumrani (intern).

Page 75: Hepatobiliary & pancreatic surgery

Hepatobiliary & pancreatic surgerychronic cirrhosis. Alcohol can affect liver cell function in a number of ways. Like many hepatotoxins, the toxic effectsof alcohol are caused indirectly by reactive intermediates. Acetaldehyde is the principal reactive compound generatedby alcohol metabolism. The hepatotoxicity of acetaldehyde is related to its binding two major constituents of cellularmembranes altering membrane integrity and enzymatic function frequently to the detriment of the cell. Earlymorphologic changes of long-standing cardiac decompensation and right-sided heart failure are central lobularcongestion, hemorrhage, and necrosis combined with phlebosclerosis of central veins and scars connecting centrizonalareas (cardiac cirrhosis).

116.Important spontaneous portosystemic collaterals which develop in the face of portal hypertension include:

a. The hemorrhoidal veinsb.Left renal veinc. The paraumbilical venous plexusd.The coronary, short gastric, and paraesophageal veins

Answer: a, b, c, d

If portal pressure is elevated, spontaneous portosystemic collaterals develop in an attempt to decompress the portalsystem. Such collaterals increase venous return to the heart and increase cardiac output. In humans, the most importantcollaterals develop as tributaries of the coronary, short gastric, and paraesophageal veins; intercostal, esophageal, andazygous veins; the superior, middle, and inferior hemorrhoidal veins; and the peraumbilical plexus. Retroperitonealveins and veins draining to the left renal vein from the splenic, adrenal, and gonadal veins may also serve as sites of thedevelopment of venous collaterals.

117.Which of the following statement(s) is/are true concerning the pathophysiology of variceal hemorrhage?

a. All patients with portal hypertension will develop esophageal varicesb.All patients with esophageal varices eventually bleedc. Variceal size can predict the incidence of variceal hemorrhaged.Control of acid secretion by H2 blockade can decrease the incidence of rebleeding after esophageal

hemorrhagee. None of the above

Answer: e

About two-thirds of patients with portal hypertension develop varices; of these only two-thirds subsequently experiencevariceal hemorrhage. A number of factors are important in the pathogenesis of variceal hemorrhage. These includeportal pressure, intravariceal pressure, variceal size and structure, and other factors. Variceal size alone is not predictiveof variceal hemorrhage. Evidence conflicts about whether erosive esophagitis is a cause of variceal rupture. Control ofacid reflux by H2 blockade has not been shown to decrease the incidence of rebleeding after esophageal hemorrhage.

118.Which of the following statement(s) is/are true concerning the use of transjugular intrahepatic portosystemicshunts (TIPS) in the treatment of variceal bleeding?

a. This procedure effectively creates an end-to-side portocaval shuntb.Procedure-related mortality is generally in excess of 20%c. TIPS has been used successfully as a pretransplant procedure to reduce portal pressured.The placement of a TIPS is not associated with the development of encephalopathy

Answer: c

Transjugular intrahepatic portosystemic shunts (TIPS) refer to an implantable, expandable metal stent placedradiologically through the hepatic parenchyma to establish a track between branches of the hepatic and portal veins.TIPS results in similar hemodynamics as a side-to-side portal systemic shunt. There is firm clinical data that TIPSprovides effective control of acute variceal hemorrhage and portal hypertension regardless of the etiology of theunderlying liver disease or the degree of hepatic decompensation. TIPS has also been used for preoperative portaldecompression to facilitate orthotopic liver transplantation. Pretransplant TIPS should reduce portal pressure therebyreducing operative time and blood loss. The major complications of TIPS include encephalopathy and stenosis orocclusion of this stent. Encephalopathy occurs in 10% to 20% of patients after TIPS. This complication appears tocorrelate with increasing age of the patient and increased shunt diameter and shunt flow.

Page 76: Hepatobiliary & pancreatic surgery

119.Hepatic encephalopathy is a common systemic manifestation of chronic liver disease. Which of the followingstatement(s) is/are true concerning this condition?

Asir Surgery MCQs Bank. © 1422H-2002- first impression ©

This project was raised after an idia by Dr. Gharama Al-Shehri (consultant surgeon). Developed and typed by Dr. Ghazi Al-Shumrani (intern).

Page 77: Hepatobiliary & pancreatic surgery

Hepatobiliary & pancreatic surgery

a. Blood ammonia levels correlate well with the stage of hepatic encephalopathyb.Alterations in central nervous system neurotransmitters such as the neurotransmitter g- aminobutyric acid

(GABA) have been proposed in the pathogenesis of hepatic encephalopathyc. Lactulose can be used to decrease intestinal ammonia absorptiond.Patients can be expected to have an increased sensitivity to benzodiazepines

Answer: b, c, d

Hepatic encephalopathy, a poorly explained neuropsychiatric syndrome, characterized by diverse neurologicabnormalities, is the pathologic evidence of nonspecific structural changes in neurons, and a variable prognosis. Severalhypotheses to explain the pathogenesis of hepatic encephalopathy have been proposed. Ammonia has been widelyimplicated in the pathogenesis of hepatic encephalopathy despite conflicting evidence. Blood ammonia levels correlatepoorly with the stage of encephalopathy, however, one of the mainstays of treatment is measures to decrease ammoniaabsorption from the gut including the oral administration of lactulose. Another hypothesis has implicated falseneurotransmitters in the pathogenesis of hepatic encephalopathy. Evidence suggests that activation of the GABA systemmay be important in the pathogenesis of hepatic encephalopathy. The GABA receptor binds several classes of ligandsincluding GABA and drugs such as benzodiazepines. For reasons that are unclear, hepatic failure appears to increase thebrain density of GABA receptors. This observation may explain the increased sensitivity to benzodiazepines and otherinhibitory neurotransmitters observed with patients with chronic liver disease.

120.Which of the following statement(s) is/are true concerning the management of gastroesophageal varicealhemorrhage?

a. Vasopressin decreases portal pressure through the process of splanchnic vasoconstrictionb.Somatostatin is as effective as vasopressin but without the cardiac side effectsc. Balloon tamponade provides good long-term control of bleeding esophageal varicesd.Endoscopic sclerotherapy is more effective than conservative medical therapy in the treatment of bleeding

esophageal varicese. Sclerotherapy, although excellent for the control of bleeding short-term, does not prolong overall survival

Answer: a, b, d

A number of invasive and noninvasive therapies exist for the treatment of bleeding esophageal varices. Vasopressin isfrequently used in the treatment of acute variceal hemorrhage and acts by decreasing portal venous pressure or flowthrough splanchnic vasoconstriction. Vasopressin alone has been reported to temporarily control variceal hemorrhage in50% to 75% of patients, however, it can be associated with cardiac and peripheral extremity cutaneous ischemia.Somatostatin also acts as a vasoconstrictor to reduce splanchnic flow, with trials demonstrating similar efficacy incontrolling acute hemorrhage when compared to vasopressin but without the cardiac side effects. Balloon tamponade isgenerally used for the temporary control of acute variceal hemorrhage unresponsive to vasopressin or sclerotherapy.Initial control of acute variceal hemorrhage occurs in about 80% of patients, but bleeding recurs promptly on deflationof the balloons in over 50%. Endoscopic sclerotherapy has become the primary treatment for bleeding esophagealvarices. A number of clinical trials demonstrate that emergent sclerotherapy is able to halt variceal bleeding that fails torespond to more conservative measures, with no increase in frequency or severity of complications. The long-termsurvival of patients treated with sclerotherapy continues to be debated. Metaanalysis has been performed on the datafrom seven randomized clinical trials evaluating the effect of repeated sclerotherapy on long-term survival. Thisanalysis demonstrates that sclerotherapy reduces the number of deaths by 25% therefore supporting the use ofsclerotherapy as an effective means of prolonging survival in patients who have experienced variceal hemorrhage.

121.Which of the following statement(s) is/are true concerning the surgical management of bleeding esophagealvarices.

a. A side-to-side portacaval shunt may be associated with the development of hepatofugal blood flowb.Selective shunts preserve prograde (hepatopedal) blood flow while decompressing esophageal varices or

reducing portal pressurec. The presence of intractable ascites is a contraindication to the Warren shuntd.If the patient is considered a liver transplant patient, an interposition mesocaval shunt is a suitable alternative

Answer: a, b, c, d

Although portosystemic shunts are the most effective therapy for preventing recurrent variceal hemorrhage, they areassociated with the increased incidence of encephalopathy. A number of types of shunts have been described, primarily

Page 78: Hepatobiliary & pancreatic surgery

to avoid the consequences of complete diversion of portal blood flow or to simplify the operation. End-to-sideportocaval shunts are hemodynamically unique in that all portal flow is diverted and the hepatic limb of the portal veinis ligated, thus preventing hepatofugal blood flow from the liver. With lateral side-to-side shunts, the hepatic limb of theAsir Surgery MCQs Bank. © 1422H-2002- first impression ©

This project was raised after an idia by Dr. Gharama Al-Shehri (consultant surgeon). Developed and typed by Dr. Ghazi Al-Shumrani (intern).

Page 79: Hepatobiliary & pancreatic surgery

Hepatobiliary & pancreatic surgeryportal vein remains patent. A greater compensatory increase in hepatic arterial flow occurs when the portal vein servesas an outflow track and the liver extracts oxygen and metabolites from the blood exiting through the patent limb of theportal vein (hepatofugal). The goal of selective shunts is to preserve prograde (hepatopedal) portal flow to the liverwhile selectively decompressing gastroesophageal varices (Warren shunt) or reducing portal pressures sufficiently toprevent variceal hemorrhage (small-diameter shunts). Most surgeons with a special interest in this field attempt toconstruct a selective shunt when the operation is elective, unless contraindications are present. The distal splenorenalshunt of Warren and the small-diameter interposition portocaval shunt of Sarfeh are the most common selective shuntsused. The Warren shunt is an ascitogenic operation and therefore the presence of ascites that is difficult to controlmedically is a contraindication of this operation. An interposition mesocaval shunt is frequently preferred in emergentsituations because the shunt is relatively safe to construct and promptly halts variceal hemorrhage. If future hepatictransplantation is contemplated, this shunt offers the advantages in that it does not involve dissection of the area of thehepatoduodenal ligament.

122.Which of the following statement(s) is/are true concerning the results of portosystemic shunting?

a. When comparing shunts with nonshunting procedures, only minor differences in long-term survival arereported, but the mode of death usually changes

b.Distal splenorenal shunts are associated with the development of less hepatic encephalopathyc. Survival statistics following distal splenorenal shunt in good risk patients (Child’s Class A) are in excess of

80%d.None of the above

Answer: a, b, c

Multiple randomized comparisons of distal splenorenal shunt with other types of portosystemic shunts have beenreported. Operative mortality and long-term prognosis are similar, but patients undergoing distal splenorenal shuntsdevelop less encephalopathy. Less encephalopathy is also seen with the selective Sarfeh shunt and after nonshuntingprocedures, such as sclerotherapy or devascularization. When comparing shunts with nonshunting procedures, onlyminor differences in long-term survival are reported, but the mode of death usually changes. With nonshuntingprocedures, a greater proportion of patients die of recurrent hemorrhage; after construction of a shunt, a greaterproportion die of hepatic failure. Class A patients undergoing distal splenorenal shunts can be predicted to have anactuarial survival including operative mortality in excess of 80%. This is a substantial improvement over previousexperience with portocaval shunts and is likely better than the results that can be obtained with hepatic transplantation.

123.Which of the following statement(s) is/are true concerning radiologic examinations used for the assessment ofhepatic neoplasms?

a. Magnetic resonance imaging is considered the test of choice for distinguishing hemangiomas from other masslesions

b.Dynamic CT scanning is the dominant imaging modality for routine screening and diagnosisc. Radionucleotide imaging plays an important role in modern screening and detection for liver lesionsd.CT angio-portography (CTAP) is the gold standard for early detection of metastatic hepatic lesions

Answer: a, b, d

In the Western world, the test most commonly used for screening, detection, and diagnosis of hepatic neoplasms aredynamic bolus-enhanced CT scan, MRI, and ultrasound. Dynamic CT scanning remains the dominant imaging modalityfor routine screening and diagnosis because it is sensitive, widely available, provides helpful anatomic information andallows the evaluation of other intraabdominal structures in the detection of extrahepatic disease. The most sensitive testfor the detection of liver masses is CTAP, however, like angiography is generally reserved for preoperative evaluationof resectability and not for screening. Hepatic MRI imaging is another useful tool in the detection and diagnosis of theliver lesions, with sensitivities equal to that of CT. MRI with T1 and T2-weighted images and gadilinium enhancementcan aid in the differential diagnosis of a mass. For example, MRI is now considered the test of choice for distinguishinghemangiomas from other mass lesions. Radionucleotide imaging is less sensitive and specific than CT, MRI, orultrasound, and has a high percentage of false-positive and false-negative results. Therefore, nuclear medicine scansplay a limited role in modern screening and detection, but they can help differentiate discrete masses.

124.A 45-year-old woman undergoes an ultrasound because of vague right upper quadrant pain and epigastricfullness. A 7 cm cystic lesion is detected. Which of the following statement(s) is/are true concerning the

Page 80: Hepatobiliary & pancreatic surgery

patient’s diagnosis and management?

a. Simple aspiration is indicated for treatment and diagnosisAsir Surgery MCQs Bank. © 1422H-2002- first impression ©

This project was raised after an idia by Dr. Gharama Al-Shehri (consultant surgeon). Developed and typed by Dr. Ghazi Al-Shumrani (intern).

Page 81: Hepatobiliary & pancreatic surgery

b.Bile stained fluid suggests underlying biliary pathologyc. The cyst is likely lined by cuboidal epitheliumd.Laparoscopic unroofing of the cyst can provide satisfactory treatment

Answer: c, d

Hepatobiliary & pancreatic surgery

A symptomatic solitary hepatic cyst may cause vague right upper quadrant discomfort or pain, a sensation of epigastricfullness or heaviness, and early satiety, however, most cases are asymptomatic. Complications are rare but includehemorrhage into the cyst, secondary bacterial infection, or obstructive jaundice from compression of extrahepatic ducts.In the absence of complications, laboratory abnormalities are uncommon. Cysts are somewhat more common infemales, are more common in the right lobe and are often multilocular rather than unilocular. They are lined withcuboidal epithelium resembling bile duct epithelium and are filled with fluid that may be clear, mucoid, bloody, orbilious.If the patient has no symptoms and the cyst was discovered incidentally and there is no evidence of infection ormalignancy, one may observe the patient. Neither percutaneous aspiration nor surgery is indicated. Cysts nearly alwaysrecur after simple aspiration. Treatment of symptomatic cysts is surgical. Indications for surgery include symptoms,rupture, hemorrhage, or infection. Asymptomatic, uninfected simple cysts are best treated by excision, if possible.Larger cysts may be inroofed with free peritoneal drainage unless there is a history of hemorrhage or evidence of biliarycommunication. A laparoscopic approach to the unroofing of cysts has recently been reported with excellent success. Ifthis cyst communicates with the biliary system (grossly by cholangiography) the leak may be oversewn or the cystdrained by a Roux-en-Y cystojejunostomy.

125.Which of the following statement(s) is/are true concerning biopsy techniques for hepatic masses?

a. A fine needle aspiration (FNA for cytology is contraindicated for patients with hypervascular masses)b.Percutaneous biopsy should be performed only if results may obviate the need for exploratory laparotomyc. Needle track seeding of tumor is not a risk associated with percutaneous biopsyd.Laparoscopy and biopsy play little role in the management of liver lesions

Answer: b

Biopsy of a liver mass can be done percutaneously (with or without CT or ultrasound guidance), laparoscopically, or atlaparotomy. The biopsy may be done for cytology only (FNA) or for histology (larger-core biopsy). Guided FNA has anoverall sensitivity of 77% to 94% and may allow a distinction between primary and secondary malignancy. The risksassociated with needle biopsy include bleeding, infection, needle track seeding of tumor, and sampling error.Hypervascular masses, coagulopathy, and ascites are contraindications to percutaneous core biopsy, however, FNAbiopsy is generally considered safe under these circumstances. In evaluation of any liver mass, percutaneous biopsyshould be performed only if it can reasonably be expected to obviate the need for exploratory laparotomy. Biopsy ofsuspected primary metastatic malignancy with clinical indications of unresectability may spare the patient anunnecessary laparotomy. Laparoscopy with biopsy may also be used to evaluate liver masses and to possibly avoidlaparotomy in patients considered to be borderline resectable.

126.A 55-year-old woman presents with vague right upper quadrant pain and a palpable liver. Laboratory tests arenormal and a noncontrast CT scan (patient has a history of contrast allergy) reveals an 8 cm right hepatic mass.Which of the following statement(s) is/are correct concerning the patient’s diagnosis and management.

a. A gadilinium-enhanced MRI would be indicated to define the extent of the lesion and confirm the diagnosis ofhemangioma

b.A fine needle aspiration should be performed regardless of radiographic workupc. Hepatic embolization is the treatment of choiced.The lesion should be resected because of concern for malignant degeneration

Answer: a

Cavernous hemangiomas of the liver are the most common benign hepatic tumor and are detected in some 2% to 7% ofautopsies. Overall, hemangiomas are exceeded only by hepatic metastases as the most common hepatic tumor.Cavernous hemangiomas consist histologically of cystically dilated, endothelium-lined vascular spaces. They occur inall ages but are observed more commonly in females. Hemangiomas are not premalignant. Less than half of affectedpatients have symptoms. Those that have symptoms usually have large masses. Symptoms usually occur includingvague right upper quadrant discomfort, pain, fullness, and early satiety. Physical examination may be notable forhepatomegaly, mass or bruit. There are no laboratory abnormalities in patients with hemangiomas. The most usefulradiologic test for diagnosing hemangiomas are MRI, CT and tagged red blood cell scanning. These tests have largely

Page 82: Hepatobiliary & pancreatic surgery

replaced angiography. CT with vascular contrast often demonstrates a diagnostically characteristic enhancementpattern. Gadilinium-enhanced MRI has recently been shown to be sensitive and specific in the diagnosis of hemangiomaand has better resolution than tagged red blood cell scans. FNA biopsy of suspected hemangiomas can be performed,

Asir Surgery MCQs Bank. © 1422H-2002- first impression ©

This project was raised after an idia by Dr. Gharama Al-Shehri (consultant surgeon). Developed and typed by Dr. Ghazi Al-Shumrani (intern).

Page 83: Hepatobiliary & pancreatic surgery

Hepatobiliary & pancreatic surgeryhowever this procedure should be avoided if the diagnosis is secure using noninvasive procedures. Given the naturalhistory of hemangiomas and its low risk of rupture, observation is indicated for asymptomatic patients, especially forlesions smaller than 4 cm. Surgical excision is the only consistently effective treatment for symptomatic masses andshould be performed if the lesion is localized and accessible with an acceptable operative risk. Embolization is indicatedonly for unresectable lesions and is only modestly effective.

127.Which of the following statement(s) is/are true concerning the pathogenesis of hepatocelluar carcinoma?

a. Hepatocellular carcinoma is associated with infection with hepatitis A, hepatitis B, and hepatitis Cb.The risk of developing hepatocellular carcinoma related to hepatitis B viral infection is related to the chronicity

of the infectionc. Hepatocellular carcinoma only develops in cirrhosis associated with hepatitisd.Alfatoxin B1 is a potent carcinogen in the development of hepatocellular carcinoma

Answer: b, d

Hepatocellular carcinoma has three well-known epidemiologic associations: hepatitis B infection, cirrhosis, and varioushepatotoxins, most notably aflatoxin B1 (a microtoxin from the fungus Aspergillus flavus). Hepatocellular carcinoma isstrongly associated with hepatitis B viral infection but there is no link between the neoplasm and the hepatitis A viralinfection. There does appear to be some link between hepatocellular carcinoma and hepatitis C virus. Cirrhosis,predominantly macronodular, is a frequent result of hepatitis B viral infection and is a risk factor for hepatocellularcarcinoma. Nevertheless, the micronodular form, which is more common in early alcoholic cirrhosis, will progress tohepatocellular carcinoma in about 2% to 3% of cases. Thus, hepatocellular carcinoma can develop in patients withhepatitis B viral infection and cirrhosis, hepatitis B viral infection alone, and with cirrhosis from any cause alone.

128.A 38-year-old woman with a 17 year history of oral contraceptive use presents with right upper quadrant pain.A CT scan demonstrates a 4 cm lesion in the right lobe of the liver. Which of the following statement(s) is/aretrue concerning the patient’s diagnosis and management?

a. The lesion is likely premalignantb.A 99mTc sulfur colloid scan will distinguish this benign lesion from a malignant hepatomac. The lesion, although benign, may be associated with life-threatening hemorrhaged.The lesion would be expected to be hypervascular on angiographic study

Answer: c, d

Hepatic adenomas are clearly linked with the use of oral contraceptives. The likelihood of developing a hepaticadenoma appears to be related to the duration and dosage of estrogen, and is greater at ages above 30 years. Themajority (75%) of these lesions occur in the right lobe of the liver. Although adenomas may be a symptomatic, mostpatients have symptoms, usually of abdominal pain (as many as 50%), and 10% to 33% of patients present with acutesigns and symptoms secondary to bleeding or rupture with intraperitoneal hemorrhage.No radiologic test is specific for adenomas. CT is the most useful preliminary test and often reveals areas ofhemorrhage and necrosis. Angiography may add to CT findings by demonstrating the hypervascular tumor with aperipheral blood supply. 99mTc sulfur colloid scans show a cold spot thereby distinguishing adenoma from focalnodular hyperplasia but not from other solid masses. For asymptomatic patients or patients with minimal symptoms,surgery is the treatment of choice given the tumors have the tendency to bleed and a small chance of coexistingmalignancy.

129.Which of the following statement(s) is/are true concerning focal nodular hyperplasia (FNH)?

a. The lesion predominantly affects young womenb.The lesion is associated with the use of oral contraceptives and other estrogensc. Radionucleotide scanning can be useful in the specific diagnosis of FNHd.Excisional biopsy is indicated in almost all cases because of the risk of bleeding

Answer: a, c

Focal nodular hyperplasia (FNH) should not be confused with a hepatic adenoma. Although FNH predominantly affectsyoung women, it is also found in men and children. Unlike adenomas, there is no clear relationship between oralcontraceptives and the development of FNH. FNH is most commonly asymptomatic and does not have a propensity tobleed or undergo malignant change. Histologically, FNH contains normal-appearing hepatocytes, bile ducts, and

Page 84: Hepatobiliary & pancreatic surgery

Kupffer cells in distinction to adenomas. Radionucleotide imaging can be useful in diagnosing FNH because FNH is theonly lesion that contains Kupffer cells and therefore appears isodense rather than a filling defect. Treatment of

Asir Surgery MCQs Bank. © 1422H-2002- first impression ©

This project was raised after an idia by Dr. Gharama Al-Shehri (consultant surgeon). Developed and typed by Dr. Ghazi Al-Shumrani (intern).

Page 85: Hepatobiliary & pancreatic surgery

Hepatobiliary & pancreatic surgeryasymptomatic patients is conservative when the diagnosis is clear. If there is doubt regarding the diagnosis, thenexcisional biopsy is indicated for small, easily removable lesions.

130.Which of the following statement(s) is/are true concerning the prognosis of patients with hepatic metastasesand colorectal carcinoma?

a. Over half of these patients will survive one year without treatmentb.Five year survivals following hepatic resection for an isolated metastasis is in excess of 25%c. Survival beyond five years after resection suggests a high probability of cured.Survival rates are improved with a margin of resection greater than 1 cme. The size of a liver metastasis is not a significant factor in predicting recurrence if adequate margins can be

obtainedAnswer: b, c, d, e

Studies consistently report five-year survival rates averaging 25% for hepatic resection for colorectal metastases. Thosewho survive beyond five years seem to do well with only an additional 5% dying of recurrence within the next fiveyears. Median survival of patients with untreated metastases is on the order of three to 10 months, with only 20%surviving past one year. Overall survival is significantly improved with surgical margins greater than 1 cm withdecreased survival in patients with positive margins or margins less than 1 cm in size. The number of metastasis is aless consistent but statistically significant factor. Patients with four or more metastases have a poorer prognosis. As forthe size of the metastasis, it is not a significant factor except that a larger total liver volume of metastasis requires alarger hepatic resection. Larger size may preclude adequate margins and indicate longer development of time with anincreased likelihood of micrometastases.

131.Which of the following statement(s) is/are true concerning the treatment and prognosis of hepatocellularcarcinoma?

a. The fibrolamellar variant of hepatocellular carcinoma has a distinctly better prognosis than other forms of thedisease

b.Patients with untreated hepatocellular carcinoma rarely survive a yearc. Multiagent chemotherapy is extremely effective in hepatocellular carcinoma and should be considered for the

treatment for most patientsd.Hepatic artery ligation or embolization has been demonstrated to be highly effective for hepatocellular

carcinomaAnswer: a, b

Hepatocellular carcinoma has a tendency for local and vascular invasion. Untreated, patients have a dismal prognosiswith a median survival of only three to four months after diagnosis with only rare survival beyond one year. Thefibrolamellar variant of hepatocellular carcinoma has a distinctly better prognosis with a five year survival afterresection of 50% to 60% versus 25% for other varieties. Surgical resection offers the only chance for cure for primaryhepatic malignancy, and survival is better if tumors are small and asymptomatic. The role of orthotopic livertransplantation for hepatocellular carcinoma continues to evolve. For patients with severe hepatic dysfunction or largeror centrally located tumors or bilobar tumors, this may be the preferred approach. Extrahepatic disease including lymphnode involvement is a contraindication to resection or transplantation. No single-or multiple-agent chemotherapy isparticularly effective in treating hepatocellular carcinoma. The rationale for treating vascular liver tumors with arterialobstruction is the fact that the nutritional blood supply of these tumors comes primarily from the hepatic artery. Hepaticartery ligation or embolization alone or in combination with regional chemotherapy has been shown to be ineffective forliver cancer. In addition, hepatic artery ligation has a high complication rate and cannot often be used in patients withcompromised liver function from cirrhosis.

132.The following statement(s) is/are true concerning the relationship of the biliary tree and the hepatic artery andportal vein.

a. The common hepatic and common bile duct lie immediately anterior to the portal veinb.The cystic artery, which usually arises from the right hepatic artery, crosses behind the hepatic duct in most

casesc. A replaced right hepatic artery arising from the superior mesenteric artery system will run to the right of the

common bile duct

Page 86: Hepatobiliary & pancreatic surgery

d.The arterial supply of the extrahepatic biliary ducts is derived from major trunks running along the medial andlateral walls of the common duct at the 3 o’clock and 9 o’clock position

Answer: a, b, c, dAsir Surgery MCQs Bank. © 1422H-2002- first impression ©

This project was raised after an idia by Dr. Gharama Al-Shehri (consultant surgeon). Developed and typed by Dr. Ghazi Al-Shumrani (intern).

Page 87: Hepatobiliary & pancreatic surgery

Hepatobiliary & pancreatic surgery

The common hepatic duct passes downward in the superior and lateral portions of the hepatoduodenal ligament and liesin front of the portal vein and to the right of the hepatic artery. The cystic artery which in most cases arises from theright hepatic artery usually crosses behind the hepatic duct (84%) of cases but is sometimes anterior to that structure(16%) of cases. A number of anatomic variations of the arterial supply of the liver can be seen. In patients in which theright hepatic artery arises from the superior mesenteric artery system, the "replaced" right hepatic artery usually runs tothe right of the bile duct and portal vein. The arteries to the extrahepatic biliary ducts anastomose freely within the ductwalls. The ductal arterial supply is derived primarily from the gastroduodenal and right hepatic arteries with majortrunks running along the medial and lateral walls of the common duct at the 3 o’clock and 9 o’clock positions.

133.An understanding of the anatomy of the extrahepatic biliary tree is essential in performing biliary tract surgery.Which of the following statement(s) is/are true concerning biliary ductal anatomy?

a. The majority of patients have the "classic" anatomical descriptionb.The common hepatic duct unites with the cystic duct to form the common bile ductc. An accessory right hepatic duct occurs in 5% of patientsd.A common channel or "Y" configuration of the distal bile duct and pancreatic ducts occur in approximately

70% of patientsAnswer: b, c, d

The classic anatomical description of the extrahepatic bile ducts and their arteries is present in only one-third ofindividuals. The left hepatic duct usually has a longer extrahepatic course than does the right hepatic duct. The commonhepatic duct is formed by the union of the right and left hepatic ducts close to the emergence from the liver. The ductpasses downward in the superior portion of the hepatoduodenal ligament and lies in front of the portal vein and to theright of the hepatic artery. The common hepatic duct unites with the cystic duct to form the common bile duct. Anaccessory right hepatic duct occurs in 5% of cases. The cystic duct passes downward, backward, and to the left in thehepatoduodenal ligament and usually unites with the main hepatic duct at an acute angle. Its course and mode ofinsertion into the common duct is highly variable. The common bile duct is formed by the union of the common hepaticand cystic ducts and is usually about 7 to 9 cm long. The junction of the distal common bile duct and pancreatic duct atthe ampulla can take one of three configurations. In approximately 70% of patients, there is a common channel of thebile and pancreatic ducts, thus a "Y" configuration. In approximately 20%, the common channel is nonexistent, and inanother 10%, the two ducts enter the duodenum via separate openings.

134.A 35-year-old female presents with typical biliary colic symptoms, however her sonogram shows nogallstones. Which of the following statement(s) is/are true concerning her diagnoses?

a. Chronic acalculous cholecystitis or gallbladder dyskinesia is seldom associated with classic biliary colicsymptoms

b.The most specific test for diagnosing gallbladder dyskinesia is CCK-enhanced cholescintigraphy withassessment of gallbladder ejection fraction

c. An ejection fraction greater than 75% is considered abnormal and indicative of gallbladder dyskinesiad.Cholecystectomy is not indicated for chronic acalculous cholecystitis

Answer: b

Motility abnormalities of the gallbladder and cystic duct present with symptoms suggesting gallstones. The mostcommon presentation for patients with gallbladder motility disorders such as chronic acalculous cholecystitis orgallbladder dyskinesia is recurrent biliary-type pain. Currently, the most specific test for diagnosing gallbladderdyskinesia is CCK-enhanced cholescintigraphy with assessment of gallbladder ejection fraction. CCK is infusedintravenously 15 to 30 minutes after ejecting an analogue of 99MTC imminodiacetic acid and calculating the ejectionfraction of the isotope by the contracting gallbladder. An ejection fraction of less than 35% is considered abnormal andcholecystectomy may be indicated. Most patients will have relief of symptoms following cholecystectomy.

135.The following statement(s) is/are true concerning the embryology of the biliary tree.

a. The primordial anlagen of the liver and biliary tract arises from the entodermb.Superior and inferior caudal buds form as the hepatic diverticulum developsc. The development of the liver is a separate process from that of the gallbladder and distal biliary treed.The biliary tree develops in association with the dorsal pancreas

Page 88: Hepatobiliary & pancreatic surgery

Answer: a, b

Asir Surgery MCQs Bank. © 1422H-2002- first impression ©

This project was raised after an idia by Dr. Gharama Al-Shehri (consultant surgeon). Developed and typed by Dr. Ghazi Al-Shumrani (intern).

Page 89: Hepatobiliary & pancreatic surgery

Hepatobiliary & pancreatic surgeryThe primordial anlagen of the liver, extrahepatic bile ducts, gallbladder, and ventral part of the pancreas develop as athickened area of entoderm on the ventral surface of the caudal portion of the foregut where it joins the midgut.Superior and inferior caudal buds form as the hepatic diverticulum grows out into the ventral mesogastrium. The solidmass of endodermal cells spreading with this cephalic bud forms the right and left lobes of the liver. The superiorgrowth of the cranial portion of the hepatic diverticulum, which extends from the duodenum to the liver, results in theformation of the hepatic, common hepatic, and common bile ducts. The caudal portion of the hepatic diverticulumdevelops into the gallbladder and cystic duct. The common bile duct is attached to the ventral aspects of the duodenumand is in close contact with the ventral pancreatic bud.

136.The following statement(s) is/are true concerning biliary motor function.

a. The contracted sphincter of Oddi impairs bile flow into the duodenum and directs it into the gallbladderb.In the postprandial state about 70% of hepatic bile flows into the gallbladder before reaching the duodenumc. During the interdigestive period, only a small fraction of gallbladder bile enters the duodenumd.Gallbladder emptying during fasting is associated with phase III of the interdigestive migrating motor complex

(MMC)e. After a meal, the gallbladder empties by a steady tonic contraction thought to be due to release of endogenous

motilin from the mucosa of the small intestineAnswer: a, b, c

As bile is secreted from the liver, it flows through the hepatic ducts into the common hepatic duct and continues throughthe common bile duct into the duodenum. With an intact and contracted sphincter of Oddi, bile flows directly into thegallbladder where it is concentrated and stored. In the postprandial state, about 70% of hepatic bile flows into thegallbladder before reaching the duodenum and entering the enterohepatic cycle. During the interdigestive phase, 90% ofbile from the liver enters the gallbladder, while only a small fraction of gallbladder bile enters the duodenum.Gallbladder emptying during fasting is associated with phase II of the interdigestive migrating motor complex (MMC).Motilin may account for this stimulatory effect since plasma elevations of motilin seem to correlate with the onset ofphase II waves. Following a meal, the gallbladder empties by a steady tonic contraction thought to be due to release ofendogenous cholecystokinin (CCK) from the mucosa of the small intestine.

137.The following statement(s) is/are true concerning the anatomy of the gallbladder.

a. The gallbladder lies between the right, left, and quadrate hepatic lobes or hepatic segments IV and Vb.The cystic duct contains the spiral valve of Heister which serves an important valvular function for the

gallbladderc. The cystic artery arises from the right hepatic artery in 95% of casesd.The cystic artery crosses anterior to the hepatic duct in the majority of cases

Answer: a, c

The gallbladder is a pear-shaped organ bound to a fossa on the right inferior surface of the liver located between theright, left, and quadrate hepatic lobes or hepatic segments IV and V. The gallbladder can be divided into four areas: thefundus, body, infundibulum, and neck. The body of the gallbladder extends from the fundus into the tapered portion, orneck, which curves backward and upward toward the transverse fissure of the liver and terminates in the cystic duct.The cystic duct lumen contains a thin mucosal septum, the spiral valve of Heister; the valve may make catheterizationto the cystic duct difficult but does not have true valvular function. The arteries of the gallbladder are derived from thecystic branch of the hepatic artery which in 95% of cases originates from the right hepatic artery. From its origin, thecystic artery usually crosses behind the hepatic duct (84% of cases) but is sometimes anterior to that structure. Thecystic artery proceeds to the neck of the gallbladder, where it divides into anterior and posterior divisions that supplythe corresponding areas of the gallbladder. The cystic veins empty into the right branch of the portal vein indirectly intothe liver.

138.The gallbladder plays an important role in altering bile composition by absorption and secretion. Which of thefollowing statement(s) is/are true concerning this mucosal function?

a. The absorption of water by the gallbladder can result in concentration of the solute components of bile from 2-to 10-fold

b.Gallbladder mucosal absorption can occur by both active and passive mechanismsc. Cyclic adenosine monophosphate (cAMP) stimulates NaCl-coupled transport and may also influence tight

Page 90: Hepatobiliary & pancreatic surgery

junction permeabilityd.Secretory products of the gallbladder include bicarbonate and glycoproteins

Answer: a, b, dAsir Surgery MCQs Bank. © 1422H-2002- first impression ©

This project was raised after an idia by Dr. Gharama Al-Shehri (consultant surgeon). Developed and typed by Dr. Ghazi Al-Shumrani (intern).

Page 91: Hepatobiliary & pancreatic surgery

Hepatobiliary & pancreatic surgery

The gallbladder rapidly absorbs water and solutes from bile and concentrates the solute components 2-to 10-fold. Thegallbladder has an active mucosa and is able to absorb water and solutes against significant concentration gradients.Water absorption is linked to the transport of ions. The two major mechanisms of absorption are active and passive. Inpassive absorption, sodium and chloride enter the gallbladder epithelial cells because of electrochemical gradients. Thisresults in an osmotic gradient, and water flows into the cell. Intracellular sodium is extruded across the basolateralmembrane into the lateral intercellular spaces by active transport. The active transport of sodium against anelectrochemical gradient is associated by an Na+-K+-ATPase pump. Cyclic AMP may inhibit NaCl-coupled transportand may also influence tight junction permeability. Other peptides, such as secretin, glucagon, and gastric inhibitorypeptide, have been shown to inhibit absorption. Secretion by the gallbladder occurs either by inhibiting net ion and fluidabsorption or with stimulation of bicarbonate secretory mechanisms. Gallbladder epithelium may also secrete mucinand nonmucin glycoproteins which may play a role in gallstone formation.

139.Abnormalities of the sphincter of Oddi have been recently recognized to cause symptoms which are referableto the biliary tree or pancreas. The following statement(s) is/are true concerning sphincter of Oddi motorfunction.

a. The sphincter’s basal resting pressure is 10 to 15 mm Hg above duodenal pressureb.Contraction of the sphincter occurs with CCK stimulationc. Vagal stimulation results in relaxation of the sphincterd.Manometry of the sphincter of Oddi may be performed at the time of ERCP to characterize basal pressure,

amplitude, frequency of contraction, and direction of propagation of contractile wavese. Stenosis of the sphincter of Oddi is characterized by abnormally elevated basal pressure on sphincter of Oddi

manometricsAnswer: a, c, d, e

The sphincter of Oddi is about 4 to 6 mm in length. The sphincter’s basal resting pressure is about 13 mm Hg aboveduodenal pressure. The sphincter exhibits phasic contractions at a frequency of 4 per minute and a duration of 8seconds. The regulation of bile flow is primarily controlled by the sphincter and not by the surrounding smooth muscleof the duodenum. Relaxation of the sphincter occurs with CCK stimulation leading to diminished amplitude of phasiccontractions and reduced basal pressure, allowing increased passive flow of bile into the duodenum. Parasympatheticstimulation also causes intermittent relaxation of the sphincter, and sympathetic splanchnic stimulation causes increasedpressure. Abnormalities of the sphincter of Oddi may cause symptoms which are referable to the biliary tree orpancreas. Manometry of the sphincter of Oddi may be performed at the time of ERCP to characterize its basal pressure,the amplitude and frequency of contractions, and the direction of propagation of contractile waves. Stenosis of thesphincter of Oddi is characterized by abnormally elevated basal pressure (> 40 mm Hg) whereas dyskinesia ischaracterized by abnormalities of other manometric parameters.

140.A 32-year-old woman with symptomatic gallstones wishes to discuss nonsurgical options for her gallstones.Which of the following statement(s) are true?

a. The best commercially available oral dissolution agent, ursodeoxycholic acid, is associated with a completedissolution rate of less than 50%

b.If the gallstones dissolve, there is minimal risk of gallstone recurrencec. Contact dissolution is applicable regardless of stone typed.Extracorporial shock wave lithotripsy (ESWL) in combination with oral dissolution agents is an appropriate

technique for most patients and can result in complete stone fragment clearance in over 90% of patients by one yearAnswer: a

Prior to the introduction of laparoscopic cholecystectomy, there was tremendous enthusiasm for a number ofnonsurgical techniques for managing gallstones. Dissolution of existing gallstones with pharmacologic agents has beenaddressed by several national cooperative studies. Ursodeoxycholic acid is the best, safest, and most effectivecommerically available drug currently available. However, it still has a rate of complete dissolution of only 40%. It isalso estimated that only 10% of patients will be suitable candidates for this therapy. Furthermore, gallstone recurrence isa major problem. Actuarial life table analysis indicates that the risk of gallstone recurrence in patients who haveundergone dissolution of gallstones with oral bile acid therapy is 50% by five years.The addition of extracorporial shock wave lithotripsy increases the efficiency of gallstone clearance and in selectedpatients complete fragment clearance can be obtained in over 90% of patients by one year. However, optimal results canbe obtained only by setting relatively strict criteria for inclusion. In applying such criteria, less than 20% of patients in

Page 92: Hepatobiliary & pancreatic surgery

the United States would be considered eligible for ESWL. Similar problems with gallstone recurrence have beenobserved with this technique. Finally, contact dissolution primarily with the ether solvent methyl tert-butyl ether(MTBE) is extremely effective in dissolving cholesterol gallstones. This technique, however, will work only in stonesAsir Surgery MCQs Bank. © 1422H-2002- first impression ©

This project was raised after an idia by Dr. Gharama Al-Shehri (consultant surgeon). Developed and typed by Dr. Ghazi Al-Shumrani (intern).

Page 93: Hepatobiliary & pancreatic surgery

Hepatobiliary & pancreatic surgerywhich are composed of cholesterol and the patient must have a demonstrably patent cystic duct before considering thistreatment. As with oral dissolution and lithotripsy, gallstone recurrence will remain a problem with this technique.

141.A 48-year-old woman presents with several hours of acute right upper quadrant pain, low grade fever, andnausea and vomiting. Which of the following statement(s) is/are true concerning her diagnosis andmanagement?

a. A mild elevation of her bilirubin (< 3 mg/dl) would strongly suggest a common bile duct stoneb.A positive bile culture can be expected in virtually 100% of patients with this scenarioc. Laparoscopic cholecystectomy is clearly contraindicatedd.Appropriate antibiotic coverage should include coverage for gram-negative aerobes

Answer: d

Acute cholecystitis occurs in about 10% to 20% of patients with symptomatic gallstones. Although the clinicalmanifestations of biliary colic and acute cholecystitis may overlap and clinical distinction is often difficult, persistenceof pain beyond a matter of hours and fever would strongly suggest acute cholecystitis. The primary events in thedevelopment of acute cholecystitis are chemical in nature with bacterial infection playing a minor role in the genesis ofthe disease. In normal healthy subjects without gallstones, incidence of positive bile cultures is essentially zero. Incontrast, between 30 and 70% of patients with the clinical diagnosis of acute cholecystitis will have positive bilecultures. The incidence of positive bile cultures who undergo cholecystectomy increases significantly with age.Septic complications continue to be a significant source of morbidity after cholecystectomy for acute cholecystitis.These septic complications can best be prevented by the judicious use of appropriate antimicrobial agents. The goal ofantimicrobial therapy should be establishment of adequate serum and tissue levels of antibiotic rather than selection ofan antibiotic that is excreted into the bile. Given the bacteriology that is typical in patients with uncomplicatedcholecystitis, an appropriate antibiotic regimen should provide for adequate coverage of gram-negative aerobes.Although technically more difficult, laparoscopic cholecystectomy can be completed safely in the majority of patientswith acute cholecystitis. Significant experience and good judgment, however, is essential in insuring optimal results.Laboratory data are often nonspecific with acute cholecystitis. Mild jaundice may be present in up to 20% of patientsand is typically due to inflammation as opposed to bile duct obstruction secondary to stones.

142.Laparoscopic cholecystectomy has become the procedure of choice for the management of symptomaticgallstones. Which of the following statement(s) is/are true concerning laparoscopic cholecystectomy?

a. Injury to biliary tree occurs more frequently with laparoscopic cholecystectomy than open cholecystectomyb.Laparoscopic cholecystectomy is contraindicated in patients with acute cholecystitisc. Bile duct injuries are more likely to occur during the surgeon’s early operative experience with the procedured.Previous upper abdominal surgery is an absolute contraindication to laparoscopic cholecystectomy

Answer: a, c

Laparoscopic cholecystectomy was first reported in 1988 and in recent years has supplanted open cholecystectomy asthe treatment of choice in the management of calculous biliary tract disease. As experience with the technologyincreases, recommendations regarding contraindications for laparoscopic cholecystectomy have evolved. Absolutecontraindications include the inability to perform laparoscopic cholecystectomy because of inadequate training orequipment, poor candidate for general anesthesia, uncontrolled coagulopathy, peritonitis, or suspected gallbladdercarcinoma. Factors previously considered relative contraindications have been identified but with appropriate skill andjudgment do not preclude the laparoscopic technique. These include acute cholecystitis, morbid obesity, previous upperabdominal surgery, cirrhosis, portal hypertension, and pregnancy. Laparoscopic cholecystectomy for acute cholecystitisis clearly more difficult and while the conversion rate of elective laparoscopic cholecystectomy to an open procedure isless than 5% in the setting of acute cholecystitis, the rate is approximately 20–30%.The incidence of bile duct injury in large surveys in this country and abroad suggest that the incidence of bile ductinjury with open cholecystectomy is less than 0.2%. Initially much higher, (approaching 1%), several series oflaparoscopic cholecystectomy have documented that injuries to the bile ducts currently occur at the rate of 0.2–0.6% ofcases. There is a definite learning curve associated with performing laparoscopic cholecystectomy with most bile ductinjuries likely to occur during a surgeon’s early operative experience with this procedure.

143.Which of the following statement(s) is/are true concerning gallstone ileus?

a. The diagnosis may be suggested by plain abdominal radiograph

Page 94: Hepatobiliary & pancreatic surgery

b.Primary surgical management consists of relief of obstruction and cholecystectomyc. Gallstone ileus accounts for less than 5% of all causes of intestinal obstructiond.Typical patients are elderly with long-standing gallstone disease

Asir Surgery MCQs Bank. © 1422H-2002- first impression ©

This project was raised after an idia by Dr. Gharama Al-Shehri (consultant surgeon). Developed and typed by Dr. Ghazi Al-Shumrani (intern).

Page 95: Hepatobiliary & pancreatic surgery

Answer: a, c, dHepatobiliary & pancreatic surgery

Cholecystoenteric fistulas are the most common form of gallbladder perforation accounting for 40% of allcases.Depending on the size of the fistulous communication, a gallstone may pass through this tract. If the stone is largeenough (> 2 cm), it may become lodged in a portion of the gastrointestinal tract and cause mechanical small bowelobstruction. This condition, called gallstone ileus, is relatively rare and accounts for fewer than 5% of cases of intestinalobstruction. The diagnosis of gallstone ileus may be suggested by the presence of intrahepatic biliary air in the settingof a small bowel obstruction seen on abdominal radiography. Patients with gallstone ileus are best managed as if theyhad a mechanical small bowel obstruction with aggressive fluid resuscitation, broad spectrum antibiotics, and earlylaparotomy. The primary goals at laparotomy are correction of the obstruction and removal of the offending gallstone.Since many of the patients are elderly and ill, cholecystectomy and takedown of the biliary enteric fistula may not beappropriate. Enterolithotomy alone, without cholecystectomy, has an associated mortality of 5% in contrast to 15%mortality rate in patients who undergo both procedures at the same time.

144.The pathogenesis of cholesterol gallstones is multifactorial. A number of key processes, however, appear tointeract closely in the formation of cholesterol gallstones. These include:

a. Cholesterol supersaturationb.Gallbladder stasisc. Accelerated nucleationd.High bilirubin concentration

Answer: a, b, c

The development of gallstones is a complex multifactorial process. Although a number of factors may contribute togallstone formation, a number of steps are considered critical in the formation of gallstones. These include cholesterolsupersaturation whereby the amount of cholesterol present is in excess of the ability of the micelles and vesicles totransport cholesterol in a soluble state. In addition, the process by which cholesterol crystals form and agglomerate isreferred to as nucleation. It is likely that the process of nucleation is altered by a number of biliary proteins or othercompounds which may promote or inhibit the nucleation process. Gallbladder stasis provides the proper environmentfor gallbladder crystal precipitation and stone formation. The stagnant pool of bile within the gallbladder may facilitateboth cholesterol supersaturation, the nucleation process, as well as altering the gallbladder’s absorptive or secretoryfunctions. Although bilirubin has been suggested to play a small role in cholesterol gallstone formation, perhaps ineither nucleation or solubilization of cholesterol, excessive production of bilirubin as seen in hemolytic disorders isassociated primarily with the formation of pigment gallstones.

145.Which of the following statement(s). is/are true concerning the diagnosis of biliary tract disease?

a. Nonvisualization of the gallbladder on oral cholecystogram is diagnostic of biliary calculous diseaseb.Ultrasonography has a diagnostic accuracy and sensitivity for cholelithiasis in excess of 95%c. Ultrasonography is the preferred test to distinguish chronic from acute cholecystitisd.Hepatobiliary scintigraphy is primarily indicated to confirm the clinical diagnosis of acute cholecystitis

Answer: b, d

A number of radiographic studies can be used to diagnose cholelithiasis. Visualization of gallstones on plain abdominalradiographs is possible in about 20% of patients whose stones are grossly calcified. Traditionally, oral cholecystogramhas been the gold standard for the diagnostic evaluation of patients with suspected gallstones. Although the accuracy fororal cholecystography has been reported to be as high as 95%, several conditions preclude satisfactory examination,including acute illness; poor patient compliance; inability to absorb the tablets as the result of emesis, malabsorption, ordiarrhea; and jaundice or hepatic dysfunction. Abdominal ultrasonography has therefore become the preferred test forevaluation of patients with suspected gallstones. Most large series suggest that diagnostic accuracy and sensitivity forcholelithiasis exceeds 95%. In addition to detecting gallstones, information is provided concerning the size and shape ofthe gallbladder, gallbladder wall thickness, presence of pericholcystic fluid, or a sonographic Murphy’s sign. The latterhas been suggested to have an 85% accuracy rate in patients with acute cholecystitis. Nonetheless, while abdominalsonography is most helpful in identifying the presence or absence of gallstones, it is of limited use in distinguishingchronic from acute cholecystitis. Hepatobiliary scintigraphy provides information on the patency of the bile ducts,including the cystic duct. Recognition that cystic duct obstruction is the sine qua non of acute cholecystitis, failure ofthe gallbladder to visualize during hepatobiliary scintigraphy is highly suggestive of acute cholecystitis. Although anumber of false positive exams can occur, nonvisualization of the gallbladder during hepatobiliary scanning in the

Page 96: Hepatobiliary & pancreatic surgery

appropriate clinical setting is highly diagnostic of acute cholecystitis.

146.Which of the following statement(s) is/are true concerning the solubilization of cholesterol in bile?Asir Surgery MCQs Bank. © 1422H-2002- first impression ©

This project was raised after an idia by Dr. Gharama Al-Shehri (consultant surgeon). Developed and typed by Dr. Ghazi Al-Shumrani (intern).

Page 97: Hepatobiliary & pancreatic surgery

a. Cholesterol is highly soluble in both serum and bileb.Mixed micelles are the primary transport mechanism for biliary cholesterolc. Most cholesterol found in bile is the result of excretion from serumd.Biliary vesicles are composed primarily of biliary phospholipid

Answer: d

Hepatobiliary & pancreatic surgery

Bile is secreted by the liver and is composed primarily of water, electrolytes, and organic solutes. Bile salts, cholesterol,and phospholipids are the main solutes found within bile and account for about 80% of the dry weight of bile. Most ofthe cholesterol found in bile is synthesized de novo in the liver. Cholesterol is an organic molecule that is virtuallyinsoluble in an aqueous medium such as bile. Therefore, mechanisms for maintaining cholesterol in solution have beenevolved. For decades, the mixed micelle which is composed of the amphiphatic bile salts and phospholipid wasconsidered the primary carrier of bile. More recently, it has been demonstrated that up to 70% of the total amount ofcholesterol normally found in gallbladder bile is transported and solubilized in the vesicular form. Bile vesicles arecomposed primarily of phospholipid of which in the human, lecithin accounts of 90% of the phospholipid content.

147.Appropriate options for management of common bile duct stones identified at laparoscopic cholecystectomyinclude:

a. Conversion to open cholecystectomy and common duct explorationb.Transcystic duct dilatation and explorationc. Laparoscopic choledochotomyd.Complete the laparoscopic cholecystectomy with postoperative ERCP and stone removal

Answer: a, b, d

Common bile duct stones found unexpectedly during laparoscopic procedures presents the surgeon with a different setof choices from that of stones found during an open procedure where one would invariably proceed to common ductexploration. The practice of converting to an open procedure for the purposes of common duct exploration is acceptableand should be considered based on the surgeon’s experience and expertise and other options available. Transcystic ductdilatation and exploration with stone removal is being performed in increasing numbers and appears to be safe andeffective. Although experience with laparoscopic choledochotomy and common duct exploration has been reported withlimited experience, this technique is probably not appropriate for the average surgeon. Many surgeons, depending ontheir clinical situation, may opt to complete the laparoscopic removal of the gallbladder and proceed to postoperativemanagement of the stone using endoscopic technique.

148.Risk factors associated with development of gallstones include:

a. Increasing ageb.Obesityc. Rapid weight lossd.Cirrhosise. Diabetes mellitus

Answer: a, b, c, d, e

Incidence of gallstones varies widely throughout the world} In the United States, about 10% of the population hascholelithiasis. Although the physiologic explanation for increasing incidence of gallstone disease in the elderly isunclear, epidemiologic studies have demonstrated a relationship between increasing age and the prevalence ofcholelithiasis in both males and females. Gallstone disease is a much more common in women than men, however withthe higher incidence of cholelithiasis in women persisting until the seventh or eighth decade of life, at which time theincidence approaches 20% in both men and women. Clinical evidence suggests a two to three-fold increase in theincidence of cholelithiasis among morbidly obese patients compared to age matched normal weight subjects.Interestingly, rapid weight loss has also been associated with secretion of cholesterol saturated bile and increasedincidence of gallstones. Patients with diabetes mellitus have a two-fold increase of gallstone disease when compared tonon-diabetics. Although the influence of alcohol on gallstone formation remains poorly defined, autopsy studiessuggests that the incidence of cholelithiasis is significantly increased in patients with alcoholic cirrhosis.

149.Which of the following statement(s) is/are true concerning a 35-year-old woman found to have asymptomaticgallstones?

Page 98: Hepatobiliary & pancreatic surgery

a. The patient should undergo attempts at medical dissolutionb.Cholecystectomy should only be performed if the laparoscopic technique is an available option

Asir Surgery MCQs Bank. © 1422H-2002- first impression ©

This project was raised after an idia by Dr. Gharama Al-Shehri (consultant surgeon). Developed and typed by Dr. Ghazi Al-Shumrani (intern).

Page 99: Hepatobiliary & pancreatic surgery

Hepatobiliary & pancreatic surgeryc. The patient has less than 10% chance of developing significant symptoms over the next five yearsd.The risk of gallbladder cancer with stones detected at such a young age, warrants cholecystectomy

Answer: c

The optimal management of individual patients with asymptomatic gallstones has been debated for years. Mostevidence, although still somewhat controversial, would indicate that the natural history of asymptomatic gallstones isbenign and that early or prophylactic cholecystectomy, either laparoscopic or otherwise, is rarely indicated. It issuggested that less than 10% of patients with asymptomatic gallstones will develop significant symptoms over a fiveyear period. Similarly, medical dissolution with oral agents, extracorporial biliary lithotripsy, or contact dissolution, isnot indicated in these patients. The risk of gallbladder cancer is so low as to not warrant cholecystectomy inasymptomatic patients.

150.In which of the following clinical situations is pigment rather than cholesterol gallstones a more commonproblem?

a. Alcoholic cirrhosisb.Morbid obesityc. Diabetes mellitusd.Hereditary spherocytosise. Biliary tract infection

Answer: a, d, e

World wide, pigment stones are the most common type of calculus found in the gallbladder. In the United States,although cholesterol calculi predominate, pigment stones constitute about 30% of all gallbladder stones. Pigment stonesare characterized by a high concentration of bilirubin and a low cholesterol content. The high incidence of pigmentgallstones in patients with hemolytic disorders probably results from the excessive loads of bilirubin being presented tothe liver for excretion. Infection has been thought to be a key factor in the pathogenesis of pigment gallstones in thatunconjugated bilirubin, produced by bacterial deconjugation, is insoluble in water, and combines with calcium in bile toproduce a calcium bilirubin matrix. Specific alterations in bile acid metabolism have been proposed to explain theincreased incidence of pigment gallstones found in cirrhotics. Gallstones associated with morbid obesity and rapidweight loss appear to be primarily due to the increased hepatic secretion of cholesterol leading to a cholithogenic state.

151.The following statement(s) is/are true concerning the mode of spread of gallbladder cancer.

a. The most common mode of spread for gallbladder cancer is to the liver by hematogenous spreadb.Lymphatic drainage of the gallbladder can extend from the cystic duct lymph node to periportal lymph nodes

and the celiac and superior mesenteric lymph nodesc. Gallbladder cancers rarely extend past the cystic ductd.Direct extension to the liver, particularly liver segments IV and V, is the most common mode of spread for

gallbladder cancere. Most patients with gallbladder cancer present with a very limited extent of disease

Answer: b, d

The mode of spread of gallbladder cancer can be predicted by the lymphatic and venous drainage of the gallbladder.Venous drainage is into the venules that drain directly into the adjacent liver. The most common spread of gallbladdercancer is through direct extension into the liver, particularly liver segments IV and V. The lymphatic drainage of thegallbladder is to the cystic duct lymph node, to periportal lymph nodes, and then to celiac and superior mesentericlymph nodes. These tumors can also spread into and around the cystic duct and may extend into the common bile duct,causing biliary obstruction. Besides direct extension and lymphatic spread of the tumor, distant metastases is possiblebut is less commonly seen. A staging system has been developed which takes into consideration lymphatic and venousdrainage of the gallbladder. The most advanced stage, stage V, includes distant spread, either into the liver or adjacentorgans, which unfortunately is the case at the time of presentation for most patients.

152.A 55-year-old woman undergoes cholecystectomy for symptomatic gallstones. The final pathology returns oneweek after the operation showing gallbladder carcinoma. Which of the following statement(s) is/are trueconcerning this patient’s prognosis and management?

a. If the gallbladder carcinoma is limited to the mucosa and/or submucosa, cholecystectomy alone is adequate

Page 100: Hepatobiliary & pancreatic surgery

treatmentb.If the carcinoma involves deeper layers of the gallbladder wall, the prognosis will be significantly worsec. If reoperation is performed, the chance of finding residual disease will be low

Asir Surgery MCQs Bank. © 1422H-2002- first impression ©

This project was raised after an idia by Dr. Gharama Al-Shehri (consultant surgeon). Developed and typed by Dr. Ghazi Al-Shumrani (intern).

Page 101: Hepatobiliary & pancreatic surgery

Hepatobiliary & pancreatic surgeryd.If a full thickness tumor had been recognized at the time of cholecystectomy, an "extended cholecystectomy"

may have been appropriateAnswer: a, b, d

Gallbladder cancers with the best prognosis are those that are found incidentally at the time of cholecystectomy forsymptomatic gallstone disease. This point emphasizes the importance of opening all gallbladders at the time ofcholecystectomy so that any suspicious lesions can be immediately examined histologically. When gallbladder cancer islimited to mucosa and submucosa, cholecystectomy is adequate treatment and has a good prognosis, with up to 100%five year survival. When the cancer involves the deeper layers of the gallbladder wall, the prognosis is more grim. A64% five year survival was reported in patients who had carcinoma confined to the mucosa and submucosa, whereasnone of the patients with cancer involving all layers of the gallbladder wall survived more than 2.5 years. Even thoughthese tumors are relatively localized at the time of cholecystectomy, cholecystectomy alone was not adequate treatmentfor long-term survival. In hopes of improving these survival rates and considering the lymphatic and venous drainage ofthe gallbladder, it has been recommended that gallbladder cancers be treated by cholecystectomy with wide resection ofthe liver around the gallbladder bed (liver segments IV and V) and regional lymphadenectomy. This procedure has beentermed an "extended cholecystectomy." Although this treatment has prolonged median survival, an overallimprovement in five year survival has not been observed. When a gallbladder cancer has been recognized incidentallyat the time of pathology examination after routine cholecystectomy, reexploration will likely show evidence of residualdisease in the lymph nodes or adjacent liver and therefore resection of this residual disease may improve survival. Moreradical excision including hepatic resection and bile duct resection and sometimes even pancreaticoduodenectomy hasbeen reported. However, the associated increased morbidity and mortality of these operations has not resulted in animprovement in survival.

153.Gallbladder cancer accounts for 3% to 4% of all gastrointestinal tumors. Which of the following statement(s)is/are true concerning the incidence of gallbladder cancer?

a. Gallbladder cancer is much more frequent in menb.Southwest Native Americans, Alaskans, Mexicans, and Hispanics have a greater incidence of gallbladder

cancer than the general populationc. The majority of patients with gallbladder cancer have gallstonesd.The association of gallstones with gallbladder cancer is greater in patients with multiple small stones

Answer: b, c

Gallbladder cancer accounts for 3% to 4% of all gastrointestinal tract tumors and about 2% of all biliary procedures aredone for gallbladder cancers. Gallbladder cancer is much more frequent in women, with a female/male ratio of 3:1. Thedisease is most commonly seen in elderly women with a mean age of 65 at diagnosis. Southwest Native Americans,Alaskans, Mexicans, and Hispanics living in the United States have an estimated five to six times greater incidence ofgallbladder cancer than the general population. On the other hand, gallbladder cancer is much less common in blacks.The association of gallstones with gallbladder cancer is well known. Seventy to ninety percent of all patients withgallbladder cancers have gallstones, and about 0.4% of all patients with gallstones have gallbladder cancer. Theassociation of gallstones with cancer may be related to gallstone size; larger stones have a greater cancer risk. There is a10-fold increase in the incidence of gallbladder cancer in patients who have gallstones that are larger than 3 cm indiameter.

154.The following statement(s) is/are true concerning the prognosis for gallbladder cancer.

a. Average survival is in the range of one yearb.Five year survival rates approach 50%c. The combination of postoperative adjuvant radiation and chemotherapy have been associated with overall

improved survivald.For most patients, the goal of treatment is palliation

Answer: d

The prognosis for gallbladder cancer remains poor, with an average survival in the range of six months. Less than 5% ofpatients survive five years because 90% of gallbladder cancer patients present with stage V disease. For stage V disease,the goal of treatment is palliation. If these patients present with obstructive jaundice, a major goal of treatment is reliefof jaundice and its attendant symptoms such as pruritus and cholangitis. Although radiation and chemotherapeuticregimens have been tried, none have been associated with a good response.

Page 102: Hepatobiliary & pancreatic surgery

155.Which of the following statement(s) is/are true concerning gallbladder polyps?

Asir Surgery MCQs Bank. © 1422H-2002- first impression ©

This project was raised after an idia by Dr. Gharama Al-Shehri (consultant surgeon). Developed and typed by Dr. Ghazi Al-Shumrani (intern).

Page 103: Hepatobiliary & pancreatic surgery

Hepatobiliary & pancreatic surgerya. Sonographic findings of a gallbladder polyp include a filling defect which does not move with change in

positionb.Benign gallbladder polyps can cause symptoms similar to those caused by gallstonesc. Ultrasonography can readily distinguish benign from malignant gallbladder lesionsd.An asymptomatic gallbladder polyp 1 cm in size can usually be followed with serial ultrasounds and does not

require cholecystectomyAnswer: a, b

Benign gallbladder polyps cause symptoms similar to those caused by cholelithiasis. Biliary symptoms including rightupper quadrant pain and discomfort, fatty food intolerance, nausea, vomiting, and an increase in flatulence are commoncomplaints in patients with symptomatic benign gallbladder polyps. Gallbladder tumors, when diagnosedpreoperatively, may be visualized by ultrasound, oral cholescintography, or less commonly, computed tomography. Afilling defect that does not move with changes in position is probably a benign or malignant gallbladder polyp.Symptomatic patients who have lesions in the gallbladder should have a cholecystectomy. Since neither ultrasound norcholecystography can distinguish benign from malignant lesions, all gallbladders that contain polypoid lesions shouldbe removed.

156.The following statement(s) is/are true concerning the surgical therapy of cholangiocarcinomas.

a. A resectable distal bile duct carcinoma is best treated by a Whipple procedureb.Klatskin tumors may require hepatic resection in attempt for potential curec. Regardless of the surgical resection for proximal biliary tumors, stenting of the biliary anastomosis is

importantd.Complete hepatic resection with hepatic transplantation has been associated with overall good results

Answer: a, b, c

The surgical treatment of a cholangiocarcinoma is dependent upon its location. For lesions of the middle third of thebile duct, resection of the bile duct tumor with reanastomosis is the procedure of choice when possible. For distallesions, the Whipple procedure (pancreaticoduodenectomy) is often necessary. Overall, both middle and lower thirdlesions have a better prognosis than tumors in the hilum. The prognosis for patients with hilar bile duct cancer (Klatskintumors) is extremely poor with mortality rates of 80% to 90% in five years. Most patients are unresectable at the time ofdiagnosis. Unless contraindicated for other reasons, however, surgical exploration should be performed in all patientswhose tumors are potentially resectable. Hepatic lobectomy is indicated for potential cure if the bile duct lesion extendsinto the hepatic parenchyma. Regardless of the surgical therapy, stenting of the biliary anastomosis is important sincepostoperative strictures or recurrent tumors are common and long-standing stents allow for cholangiographic followupand for dilatation should strictures arise. These stents have also been used for placement of iridium seeds used for localirradiation. More aggressive surgical therapy including complete hepatic resection followed by hepatic transplantationhave been performed. However, the results have been associated with a very high rate of recurrence in the post-transplantation period and therefore at the present time this treatment is not recommended.

157.The following statement(s) is/are true concerning carcinoma of the bile ducts.

a. Carcinoma of the bile ducts is more common than gallbladder carcinomab.Similar to gallbladder cancer, bile duct cancer is more common in femalesc. Unlike gallbladder cancer, there is no association of bile duct cancer and gallstonesd.Ulcerative colitis is associated with an increased incidence of bile duct cancere. Choledochal cysts may be associated with the development of bile duct cancer

Answer: d, e

Cancer of the bile ducts is even less common than gallbladder carcinoma and is seen in 0.01% to 0.46% of all autopsies.Unlike gallbladder cancer, which is more frequent in females, males have a higher incidence of bile duct cancer. Theaverage age range of diagnosis between is 50 and 70 years. Similar to gallbladder cancer, there is an associationbetween bile duct cancer and gallstones. This association is, however, not as direct as for gallbladder cancer with only25% to 57% of patients with bile duct cancer having gallstones. Biliary tract infection is associated with these tumors;for example, patients with Clonorchis sinensis infection and chronic typhoid carriers have a higher incidence of bileduct cancer than the general population. Choledochal cysts are also associated with bile duct cancer. Patients withulcerative colitis have a marked increase incidence of bile duct tumors which tend to follow a more aggressive course.

Page 104: Hepatobiliary & pancreatic surgery

158.Which of the following statement(s) is/are true concerning the association of gallstones with gallbladdercancer?

Asir Surgery MCQs Bank. © 1422H-2002- first impression ©

This project was raised after an idia by Dr. Gharama Al-Shehri (consultant surgeon). Developed and typed by Dr. Ghazi Al-Shumrani (intern).

Page 105: Hepatobiliary & pancreatic surgery

Hepatobiliary & pancreatic surgerya. The association of gallbladder cancer with gallstones is such that young patients with asymptomatic gallstones

should undergo routine cholecystectomyb.Calcification of the wall of the gallbladder is associated with an increased incidence of gallbladder cancerc. Larger gallstones are associated with a greater risk of gallbladder cancer than smaller gallstonesd.The 20 year risk of developing gallbladder carcinoma in patients in the general population with gallstones is

approximately 1%Answer: b, c

The association of gallstones with gallbladder cancer is well known. Seventy to ninety percent of patients withgallbladder cancer have gallstones. The association of gallstones with cancer may be related to gallstone size, withlarger stones having a greater cancer risk. The 20-year risk of developing gallbladder carcinoma in patients withgallstones has been estimated to be in the range of 0.13% for the general population. It is evident that this small risk ofdeveloping gallbladder cancer in patients with gallstones, even over the long-term, does not warrant routinecholecystectomy in all patients with gallstones. Calcification of the wall of the gallbladder, the so-called porcelaingallbladder, is associated with a 25% to 60% incidence of gallbladder cancer. In the presence of a porcelain gallbladder,a cholecystectomy should be performed unless contraindicated for other reasons.

159.A 65-year-old man presents with obstructive jaundice. The patient’s workup begins with a CT scan. Which ofthe following statement(s) is/are true concerning his diagnosis?

a. A CT scan demonstrating intrahepatic biliary obstruction with a decompressed gallbladder and a nondilatedextrahepatic biliary tree will be consistent with a Klatskin tumor

b.The presence of biliary obstruction seen on the CT scan requires further evaluation with invasivecholangiography either percutaneously or endoscopically

c. Percutaneous transhepatic cholangiography would be the preferred technique for a suspected proximalcholangiocarcinoma in that it will better visualize the proximal extent of the tumor in the biliary tree

d.The placement of a transhepatic biliary catheter can prove useful in surgical management of proximal bile ductcancers

e. There is little role for angiography in the evaluation of a patient with suspected cholangiocarcinomaAnswer: a, b, c, d

Jaundice is the most frequent presenting symptom in patients with cholangiocarcinoma, occurring in 90% of patients.The diagnosis begins with abdominal imaging to detect ductal dilatation. A CT appearance of a proximalcholangiocarcinoma or a Klatskin tumor will show a dilated intrahepatic biliary tree with a decompressed gallbladderand normal caliber distal bile duct. When biliary obstruction is present, further visualization of the biliary tree isrequired either through percutaneous transhepatic cholangiography (PTC) or endoscopic retrogradecholangiopancreatography (ERCP). PTC is preferred for more proximal lesions because ERCP may fail to adequatelyvisualize the proximal portion of the biliary tree. For lower bile duct lesions, ERCP may be the preferred route. Themanagement of hilar tumors can often be facilitated by the placement of transhepatic percutaneous catheters which canfacilitate both resection, reconstruction, and may serve as a route for administration of local radiation therapy. Thesetumors are often found to be unresectable because of direct vascular invasion into the portal vein or hepatic arteries.Therefore, selective celiac angiography can be helpful preoperatively to determine if these major adjacent vascularstructures are involved with the tumor.

160.Benign gallbladder and bile duct tumors are extremely rare. Which of the following statement(s) is/are trueconcerning benign biliary tumors?

a. Cholesterol polyps are due to foamy macrophages filled with cholesterolb.Inflammatory polyps and cholesterol polyps are not considered to be premalignantc. Gallbladder adenomas, like adenomas in other gastrointestinal organs, are considered premalignantd.Most gallbladder adenomas are associated with gallstonese. An adenomyoma is a mucosal lesion of the gallbladder not considered to be premalignant

Answer: a, b, c

Benign gallbladder and bile duct tumors are extremely rare and can include pseudotumors or hyperplastic conditionswhich are thought to result from inflammatory states, or adenomas, which are most likely premalignant lesions.Cholesterolosis, or a strawberry gallbladder, is manifested by yellow spots visible on the surface of the gallbladdermucosa. This proliferation of foamy macrophages filled with cholesterol in the lamina propria may also result in theformation of polyps called cholesterol polyps. These polyps are thought to result from disturbance in cholesterol

Page 106: Hepatobiliary & pancreatic surgery

metabolism. Other pseudotumors of the gallbladder are inflammatory polyps which are composed of vascularconnective tissue stalk with a single layer columnar epithelial cells and have a chronic inflammatory infiltrate. Theselesions are not considered to be premalignant and are felt to be the result of chronic inflammation. Adenomas with

Asir Surgery MCQs Bank. © 1422H-2002- first impression ©

This project was raised after an idia by Dr. Gharama Al-Shehri (consultant surgeon). Developed and typed by Dr. Ghazi Al-Shumrani (intern).

Page 107: Hepatobiliary & pancreatic surgery

Hepatobiliary & pancreatic surgeryhyperplasia of the epithelial layer of the gallbladder, like adenomas in other gastrointestinal organs can be sessile orpapillary. Carcinoma in situ has been reported in these lesions, and they are thought to be premalignant. The cause ofadenomas of the gallbladder is unknown; but most are not associated with the presence of gallstones. An adenomyomaof the gallbladder is a rare intramural mass or nodule. This lesion is characterized by a proliferation of the mucosalepithelium and hypertrophy of the mucosal layers of the gallbladder. Histologically in addition to muscularhypertrophy, invagination of the epithelial mucosa between muscle layers is evidenced. These lesions are also notconsidered premalignant.

161.Which of the following statement(s) concerning bile duct strictures due to chronic pancreatitis is/are true?

a. Most patients present with progressive jaundiceb.Strictures are classically long and tapered involving the entire intrapancreatic bile ductc. Patients may be asymptomatic and diagnosed only by persistent elevation of serum alkaline phosphatased.An excellent option for surgical management is choledochoduodenostomy

Answer: b, c, d

The clinical presentation of patients with common bile duct strictures secondary to chronic pancreatitis is variable. Alarge number of patients are asymptomatic with a diagnosis of bile duct strictures suggested only by abnormal liverfunction tests. The serum alkaline phosphatase appears to be the most sensitive laboratory finding and is elevated inover 80% of patients. Although in most cases, transient jaundice may occur, progressive jaundice is rare.Cholangiography will demonstrate a classic long smooth gradual tapering of the common bile duct throughout its entireintrapancreatic segment. Biliary reconstruction is the appropriate management for most patients.Choledochoduodenostomy is preferred by many surgeons because it does not divert bile from the duodenum, istechnically easier to perform, and leaves the jejunum intact for any associated procedures that may be required fordecompression of the obstructed gastrointestinal tract or pancreatic duct.

162.If a bile duct injury is suspected at laparoscopic cholecystectomy, appropriate management includes which ofthe following?

a. Conversion to open cholecystectomy and intraoperative cholangiographyb.Small ducts (< 3 mm) demonstrated by cholangiography to drain a single liver segment can be ligatedc. If the injured segment is greater than 1 cm. in length an end-to-end ductal anastomosis is the procedure of

choiced.Postoperative external drainage should be avoided

Answer: a, b

In many cases, proper initial management of a bile duct injury recognized at the time of cholecystectomy can avoid thedevelopment of a bile duct stricture. Unfortunately recognition of a bile duct injury is uncommon during either open orlaparoscopic cholecystectomy. It must be emphasized that should bile leakage be noted or if "a typical" anatomy isencountered during laparoscopic cholecystectomy, early conversion to an open technique and prompt cholangiographyis imperative. If a segment of accessory duct less than 3 mm has been injured, and cholangiography demonstratessegmental or sub-segmental drainage of the injured ductal system, simple ligation of the injured duct is indicated. If theinjured duct is 4 mm or larger, however, it is likely to drain multiple hepatic segments or the entire right or left lobe andthus requires operative repair. If the injured segment of the bile duct is short (< 1 cm) and the two ends can be opposedwithout tension, an end-to-end anastomosis can be performed with placement of a T-tube through a separatecholedochotomy either above or below the anastomosis. For proximal injuries, or if the injured segment of bile duct isgreater than 1 cm in length, an end-to-end bile duct anastomosis should be avoided because of the excessive tension thatusually exists in these situations. The use of Roux-en-Y jejunal limb is preferable for creation of the anastomosis.Regardless of the type of anastomosis, all repairs at the time of initial operation should involve some form of externaldrainage either with a T-tube or intraoperatively-placed transanastomotic stent.

163.Which of the following statement(s) is/are true concerning the incidence of bile duct injury followingcholecystectomy?

a. Data from the pre-laparoscopic cholecystectomy era would suggest the incidence of bile duct injury duringopen cholecystectomy to be 0.1–0.2%

b.The current incidence of bile duct injury during laparoscopic cholecystectomy is greater than 1%c. The experience of the surgeon performing laparoscopic cholecystectomy can be correlated with the incidence

Page 108: Hepatobiliary & pancreatic surgery

of bile duct injuryd.Intraoperative cholangiography during laparoscopic cholecystectomy will prevent bile duct injury in virtually

all casesAsir Surgery MCQs Bank. © 1422H-2002- first impression ©

This project was raised after an idia by Dr. Gharama Al-Shehri (consultant surgeon). Developed and typed by Dr. Ghazi Al-Shumrani (intern).

Page 109: Hepatobiliary & pancreatic surgery

Answer: a, cHepatobiliary & pancreatic surgery

A number of large surveys encompassing thousands of patients undergoing open cholecystectomy would suggest theincidence of bile duct injury to be 0.1–0.2%. Although a number of early individual series of laparoscopiccholecystectomy reported bile duct injuries in 1% of patients, as larger series have been reported and surveys includingthousands of patients have appeared, the true incidence would appear to be 0.3–0.6%. A number of factors areassociated with bile duct injury during laparoscopic cholecystectomy including the experience of the surgeon. Thisreflects the steep learning curve with this procedure. Although strongly debated, there is no evidence that intraoperativecholangiography prevents bile duct injury during laparoscopic cholecystectomy. The use of intraoperativecholangiography may however detect the injury early in the course of the procedure and thus minimize the extent ofinjury.

164.Primary sclerosing cholangitis has a number of treatment options—both medical and surgical. Which of thefollowing statement(s) is/are true?

a. A number of immunosuppressive oral agents can provide specific effective treatment for primary sclerosingcholangitis

b.Biliary reconstruction with long-term transanastomotic stents can be useful in selected patients with focalstrictures at the hepatic duct bifurcation

c. Biliary reconstruction should be reserved only for patients with established biliary cirrhosisd.Hepatic transplantation for primary sclerosing cholangitis can be associated with survival rates similar to other

indications for transplantationAnswer: b, d

There is no known specific, effective medical therapy for primary sclerosing cholangitis. Although encouraging resultsfrom a prospective, randomized, placebo-controlled trial suggests that ursodeoxycholic acid significantly improvesserum liver function tests and clinical symptoms. Because of the lack of effective medical therapy, an aggressivesurgical approach is indicated for most symptomatic patients with primary sclerosing cholangitis. One surgicalapproach, in patients with a predominant stricture at the hepatic duct bifurcation, is resection of the bifurcation andlong-term transhepatic stenting with silastic stents. Results in patients without established cirrhosis are excellent.However, in those patients with secondary biliary cirrhosis present before surgery, perioperative morbidity andmortality have been high and long-term results poor. Patients with established secondary biliary cirrhosis should bereferred for hepatic transplantation. Recent reviews of the experience with hepatic transplantation for primary sclerosingcholangitis would suggest survival to be similar to those reported for hepatic transplantation for patients with anydiagnosis.

165.The following statement(s) regarding the elective repair of a bile duct stricture is/are true:

a. A transanastomotic stent is necessary for a successful resultb.Long-term stenting for approximately one year is necessary for an anastomosis performed at the distal

common hepatic ductc. A Roux-en-Y hepaticojejunostomy provides the best route for restoring biliary-enteric continuityd.Preoperatively-placed biliary catheters facilitate dissection and identification of the stricture and are useful in

placement of transanastomotic stents when employedAnswer: c, d

Several principles are associated with successful repair of a biliary stricture. Although many surgeons favor the use oftransanastomotic stents, a number of series have reported successful results without the use of such stents. The length ofstenting depends upon the location of the stricture. If the injury involves the common bile duct or common hepatic ductat least 2 cm distal to the hepatic duct bifurcation, and adequate proximal bile duct mucosa can be defined, the use oflong-term biliary stents is not necessary. In these situations transanastomotic stenting for 4–6 weeks postoperatively isadequate. When adequate proximal bile duct is not available for a good mucosa-to-mucosa anastomosis, long-termstenting of the biliary-enteric anastomosis with silastic transanastomotic stents for at least a year is recommended. Forestablished strictures, simple excision and end-to-end anastomosis or repair of the damaged duct can rarely beaccomplished because of the invariable loss of duct length as a result of fibrosis associated with injury. Thus in almostall cases, hepaticojejunostomy constructed to a Roux-en-Y limb of jejunum is the preferred procedure. The use ofpreoperatively-placed transhepatic biliary catheters can aid in the dissection and identification of the biliary treeespecially in patients with prior attempts at repair where scarring and fibrosis may be significant. In addition, the biliarycatheters can assist in the placement of long-term transanastomotic stents.

Page 110: Hepatobiliary & pancreatic surgery

Asir Surgery MCQs Bank. © 1422H-2002- first impression ©

This project was raised after an idia by Dr. Gharama Al-Shehri (consultant surgeon). Developed and typed by Dr. Ghazi Al-Shumrani (intern).

Page 111: Hepatobiliary & pancreatic surgery

Hepatobiliary & pancreatic surgery166.The vast majority of benign bile duct strictures occur following operations in or near the right upper quadrant.

Other causes of benign bile duct strictures include:

a. Chronic pancreatitisb.Ulcerative colitisc. Primary sclerosing cholangitisd.Intrahepatic arterial infusion of 5-fluorouracil

Answer: a, c, d

Most bile duct strictures are postoperative in nature with over 80% occurring after injury to the bile duct duringcholecystectomy. A number of inflammatory conditions can also result in strictures of the biliary tree. The chronicinflammation and fibrosis associated with chronic pancreatitis can result in a stricture of the intrapancreatic bile duct.Primary sclerosing cholangitis is an idiopathic disease believed to be autoimmune in nature which is characterized byintrahepatic and extrahepatic inflammatory strictures of the biliary tree. Although primary sclerosing cholangitis isfrequently associated with ulcerative colitis, this colonic disease has no direct causal relationship with benign bile ductstrictures. A rare cause of benign bile duct strictures in both the intrahepatic and extrahepatic biliary tree has been theuse of intrahepatic arterial infusion of 5-fluorouracil used to treat hepatic metastasis from colorectal carcinoma.

167.A 37-year-old female presents with obstructive jaundice due to a mid-bile duct stricture four months afterlaparoscopic cholecystectomy. Which of the following statement(s) are true:

a. Surgical reconstruction is the only option for management of this patientb.Excellent long-term results can be expected in approximately 80% of patients following surgical biliary

reconstructionc. One year follow-up after successful repair is satisfactory regardless of the method of managementd.Surgical reconstruction offers a better chance of long-term success than either percutaneous or endoscopic

dilatationAnswer: b, d

Excellent long-term results can be achieved in 70–90% of patients who undergo surgical repair of bile duct strictures.The definition of satisfactory results in most series requires patients with no symptoms of jaundice or cholangitis.Length of follow-up is important in analyzing results, however, because recurrent strictures can occur up to 20 yearsafter the initial procedure. About two-thirds of restrictures are evident within two years and 90% within seven years.Although operative management of bile duct strictures in most cases can result in excellent results, the nonoperativeapproaches of percutaneous or endoscopic dilatation are suitable alternatives in many patients. Although comparisonsbetween techniques are difficult, two retrospective comparative studies from single institutions have suggested thatsurgical reconstruction offers a better chance of long-term success than either percutaneous or endoscopic management.

168.Most patients with postoperative bile duct strictures after cholecystectomy present early after their initialoperation. Patients may present in which of the following manner(s)?

a. Obstructive jaundiceb.An external biliary fistulac. Progressive accumulation of bile in the peritoneal cavity (bile ascites)d.Biliary cirrhosis

Answer: a, b, c

Most patients with benign postoperative bile duct strictures present early after their initial operation. Those patientssuspected of having postoperative bile duct stricture within days to one week of an initial operation usually present inone of two ways. One mode of presentation is the progressive elevation of liver function tests, particularly serumbilirubin and alkaline phosphatase. These changes can be seen as early as the second or third postoperative day. Thesecond mode of early presentation is with leakage of bile from the injured bile duct. Bilious drainage from theoperatively-placed drains or through the wound after cholecystectomy is abnormal and represents some form of biliaryinjury. In those patients without drains, or in whom the drains have been removed, the bile may leak into the peritonealcavity as bile ascites or may loculate as a collection. Patients with markedly delayed diagnosis of bile duct stricture maypresent with advanced biliary cirrhosis and other evidence of liver dysfunction.

169.Nonoperative dilatation, performed either endoscopically or percutaneously, can be successfully employed in

Page 112: Hepatobiliary & pancreatic surgery

which of the following etiologies of bile duct strictures?

Asir Surgery MCQs Bank. © 1422H-2002- first impression ©

This project was raised after an idia by Dr. Gharama Al-Shehri (consultant surgeon). Developed and typed by Dr. Ghazi Al-Shumrani (intern).

Page 113: Hepatobiliary & pancreatic surgery

Hepatobiliary & pancreatic surgerya. Postoperative bile duct strictures following a hepaticojejunostomy used for reconstruction during a Whipple

procedureb.Complete transection of the bile duct during laparoscopic cholecystectomy (the so-called "classic laparoscopic

cholecystectomy injury")c. Primary sclerosing cholangitisd.Oriental cholangiohepatitis

Answer: a, c

Nonoperative management of bile duct strictures is an available option at most institutions, however, it has sometechnical limitations due to the anatomic situation. In the so-called "classic laparoscopic bile duct injury" however,complete bile duct transection and discontinuity of the biliary tree eliminates the possibility of nonoperativemanagement. Percutaneous dilatation of biliary-enteric anastomosis has been shown in a number of series to have asuccess rate approaching that of surgical reconstruction. Although limited experience with either percutaneous orendoscopic dilatation in primary sclerosing cholangitis has been reported, this alternative may provide at leasttemporary improvement in symptoms and radiologic appearance. Oriental cholangiohepatitis is an unusual infection ofthe biliary tree frequently associated with Clonorchis sinensis and other parasites. Cholangiography will demonstratemultiple strictures of both the intrahepatic and extrahepatic biliary tree with bile ducts filled with sludge and stones.Surgical management consisting of cholecystectomy and improving biliary drainage with either a Roux-en-Ycholedochojejunostomy or choledochoduodenostomy is necessary in almost all patients. Access to the biliary tree forpostoperative management of intrahepatic stones or sludge should be maintained, however, with transhepatic biliarystents.

170.The gold standard for evaluation of patients with bile duct strictures is cholangiography. The two routes forcholangiography are percutaneous transhepatic cholangiography (PTC) or endoscopic retrogradecholangiography (ERC). Which of the following statement(s) is/are true?

a. PTC is generally more valuable than ERC in defining the proximal biliary tree to be used in reconstructionb.ERC is technically easier in patients with bile leaks because the biliary tree is usually not dilatedc. Parenteral antibiotics should be administered prior to either procedure to prevent cholangitisd.Biliary stents can be placed via either technique to control biliary leaks

Answer: a, b, c, d

The gold standard for evaluation of patients with a bile duct stricture is cholangiography. PTC is generally morevaluable than ERC in that it defines the anatomy of the proximal biliary tree that is used in the surgical reconstruction.ERC is often less useful than PTC because the discontinuity of the extrahepatic biliary tree usually prevents adequatefilling of the proximal biliary tree. However, in patients with biliary fistulas, the proximal biliary tree is often notdilated making PTC somewhat technically more challenging. Parenteral antibiotics should be administered prior toeither procedure to decrease the risk of cholangitis. Biliary stents used either to temporarily control biliary leaks or tostent a stricture after nonoperative dilatation can be placed by either the percutaneous or endoscopic route.

171.The management of a suspected bile duct injury depends on a number of factors, most importantly the modeand timing of presentation. Which of the following statement(s) is/are true concerning a patient presenting witha suspected bile leak after laparoscopic cholecystectomy?

a. Laparotomy should be performed immediatelyb. Cholangiography should be performed to determine the nature of the injuryc. Operatively-placed drains should be removed to allow the fistula to closed. The patient should be discharged to home to allow the leak to close spontaneously

Answer: b

Patients presenting in the early postoperative period with a biliary leak may be septic with either cholangitis or intra-abdominal bile collections. Sepsis must be controlled first with broad-spectrum parenteral antibiotics, cholangiographywith percutaneous biliary drainage and percutaneous or operative drainage of biliary leaks. Once sepsis is controlled,there is no hurry in proceeding with surgical reconstruction of the bile duct stricture. The combination of proximalbiliary decompression and external drainage allows most biliary fistulas to be controlled or even closed. At that time theexternal drains may be removed. The patients can then be discharged to home to allow several months to lapse forresolution of the inflammation in the periportal region and recovery of overall health status.

Page 114: Hepatobiliary & pancreatic surgery

Asir Surgery MCQs Bank. © 1422H-2002- first impression ©

This project was raised after an idia by Dr. Gharama Al-Shehri (consultant surgeon). Developed and typed by Dr. Ghazi Al-Shumrani (intern).


Recommended